Sie sind auf Seite 1von 272

AP® NEW FORMAT TEST BANK

WORLD CIVILIZATIONS
THE GLOBAL EXPERIENCE
Revised AP® Edition
SEVENTH EDITION

Peter N. Stearns
George Mason University

Michael Adas
Rutgers University

Stuart B. Schwartz
Yale University

Marc Jason Gilbert


Hawaii Pacific University

AP® is a trademark registered and/or owned by The College Board, which was not involved in the production of,
and does not endorse, this product.
Copyright © 2017, 2015, 2011, 2007 Pearson Education, Inc. One Lake St., Upper Saddle River, NJ 07458.
All rights reserved. Manufactured in the United States of America. This publication is protected by Copyright, and
permission should be obtained from the publisher prior to any prohibited reproduction, storage in a retrieval
system, or transmission in any form or by any means, electronic, mechanical, photocopying, recording, or likewise.
To obtain permission(s) to use material from this work, please submit a written request to Pearson Education, Inc.,
Permissions Department, One Lake St., Upper Saddle River, NJ 07458, or you may fax your request to 201-236-
3290.

AP® is a trademark registered and/or owned by The College Board, which was not involved in the production of,
and does not endorse, this product.

10 9 8 7 6 5 4 3 2 1

ISBN 10: 0-13-445637-8


ISBN 13: 978-0-13-445637-9
CONTENTS

Period 1 Technological and Environmental Transformations (to c. 600 B .C.E.) 1

Period 2 Organization and Reorganization of Human Societies


(c. 600 B .C.E. to c. 600 C .E.) 24

Period 3 Regional and Interregional Interactions (c. 600 C .E. to c. 1450) 55

Period 4 Global Interactions (c. 1450 to c. 1750) 108

Period 5 Industrialization and Global Integration (c. 1750 to 1900) 162

Period 6 Accelerating Global Change and Realignments (1900 to the Present) 217

iii
iii
Copyright
Copyright © © 2017,
2017, 2015, 2015, 2011,Education,
2011 Pearson 2007 Pearson Education,
Inc., Upper Saddle River,Inc. All rights
NJ 07458. reserved.
All rights reserved.
Period 1
Technological and Environmental Transformations
(to c. 600 B.C.E.)
Section I
Multiple-Choice Questions
Questions 1.1–1.3 refer to the map below.

The Spread of Agriculture

1.1. What were the demographic and social effects of the Neolithic Revolution?
(A) Population grew as a result of more food, which led, in turn, to more complex economic
systems and more complex social systems.
(B) Hunter-gathers increased in number in order to defend their land and to keep their place in
society.
(C) As a result of a decrease in the supply of food, agriculturalists began to compete for space.
(D) The effects of the Neolithic Revolution were minimal in Mesopotamia but increased as the
Neolithic Revolution spread.

Answer: A
Topic: Diffusion of domesticated plants and animals; Development of agriculture, pastoralism, and
associated technological innovations

1
Copyright © 2017, 2015, 2011, 2007 Pearson Education, Inc. All rights reserved.
Theme: Interaction Between Humans and the Environment; Creation, Expansion, and Interaction of
Economic Systems
Learning Objective: ENV–2; ECON–10; ECON–12
Historical Thinking Skill: Causation; Analyzing Evidence: Content and Sourcing
Key Concept: 1.2.I.A

1.2. How did pastoralists impact the environment?


(A) Pastoralists cleared land to pasture their sheep and goats.
(B) Pastoralists adopted fire-stick farming to clear land.
(C) Pastoralists overgrazed fragile grasslands leading to erosion.
(D) Pastoralists created irrigation systems, such as the qanats, and cleared land.

Answer: C
Topic: Neolithic Revolution; Pastoralism; Agriculture
Theme: Interaction Between Humans and the Environment; Creation, Expansion, and Interaction of
Economic Systems
Learning Objective: ENV–2; ECON–1
Historical Thinking Skill: Analyzing Evidence: Content and Sourcing; Causation
Key Concept: 1.2.I.D

1.3. What were the environmental effects of the Neolithic Revolution?


(A) As people began to farm, more land under cultivation led to more environmental diversity.
(B) Hunter-gathers lived alongside agriculturalists and merged their communities.
(C) Women began to have more children and society became more patriarchal.
(D) Agricultural practices drastically impacted environmental diversity.

Answer: D
Topic: Neolithic Revolution; Pastoralism; Agriculture
Theme: Interaction Between Humans and the Environment; Creation, Expansion, and Interaction of
Economic Systems
Learning Objective: ENV–2; ECON–5
Historical Thinking Skill: Causation; Analyzing Evidence: Content and Sourcing
Key Concept: 1.2.I.C

2
Copyright © 2017, 2015, 2011, 2007 Pearson Education, Inc. All rights reserved.
Questions 1.4–1.6 refer to the images below.

Neolithic Woman Spinning a Dried


Excavation of the Ancient Settlement at Çatal Hüyük Yucca Stalk Against a Fire-Starter

1.4. How did agriculture and pastoralism transform human societies during this period?
(A) Increased population led to the rise of matriarchal societies across most settled lands.
(B) Increased population led to job specialization, large settlements and new classes of artisans,
warriors, and elites.
(C) Population declined because of drought and increasing desertification, which led to
increased warfare.
(D) Agriculture and pastoralism did not have any significant effect on human societies during
this period.

Answer: B
Topic: Labor specialization; Development of elites
Theme: Development and Transformation of Social Structures; State Building, Expansion, and Conflict
Learning Objective: SOC–2; SB–1
Historical Thinking Skill: Causation; Synthesis
Key Concept: 1.2.II.A

1.5. Which technological innovations led to improvements in agricultural production, trade, and
transportation?
(A) pottery, silks and woven textiles, and metallurgy
(B) plows, pottery, silks and woven textiles, and wheeled vehicles
(C) compound bows, elementary gunpowder, woven textiles, and plows
(D) pottery, plows, woven textiles, wheeled vehicles, and metallurgy

Answer: D
Topic: Agriculture, artisans, specialization; Agriculture and pastoralism
Theme: Creation; Expansion, and Interaction of Economic Systems; Interaction Between Humans and
the Environment
Learning Objective: ECON–5, ENV–2
Historical Thinking Skill: Analyzing Evidence: Content and Sourcing

3
Copyright © 2017, 2015, 2011, 2007 Pearson Education, Inc. All rights reserved.
Key Concept: 1.2.II.B

1.6. What forms of social organization developed in agrarian and pastoralist societies?
(A) Patriarchal forms of social organizations developed.
(B) Matriarchal forms of social organizations developed
(C) Society turned to polygamy as populations decreased.
(D) Dictatorships developed to run the empires.

Answer: A
Topic: Development of elites; Patriarchy and social hierarchies
Theme: State Building, Expansion, and Conflict; Development and Transformation of Social Structures
Learning Objective: SB–1; SOC–1
Historical Thinking Skill: Analyzing Evidence: Content and Sourcing
Key Concept: 1.2.II.C

4
Copyright © 2017, 2015, 2011, 2007 Pearson Education, Inc. All rights reserved.
Questions 1.7–1.9 refer to the images below.

Statue Known to the West as the Sphinx and to the Arabs as the Father of Sumerian Clay Tablet with Cuneiform
Terror Characters

Nineveh

1.7. How did rulers of states in Mesopotamia and the Nile Valley, who mobilized surplus labor to
create monumental architecture and who had the support of the military, justify their rule?
(A) survival of the fittest and strongest
(B) hereditary rule
(C) mandate of heaven
(D) divine connections to power

Answer: D
Topic: First states
Theme: State Building, Expansion, and Conflict
Learning Objective: SB–1

5
Copyright © 2017, 2015, 2011, 2007 Pearson Education, Inc. All rights reserved.
Historical Thinking Skill: Argumentation
Key Concept: 1.3.II.A

1.8. Which kingdom conquered Egypt about 1000 B.C.E and ruled it for centuries?
(A) the Hittites in Anatolia
(B) the Hebrews
(C) the kingdom of Kush
(D) the kingdom of Axum

Answer: C
Topic: Egyptian civilization; Characteristics of first states
Theme: State Building, Expansion, and Conflict
Learning Objective: SB–1
Historical Thinking Skill: Argumentation
Key Concept: 1.3.II.

1.9. As the first states emerged within core civilizations in Mesopotamia and the Nile Valley, what role
did pastoralists play?
(A) Pastoralists were migrant workers who assisted in the building of pyramids and of ziggurats
in the valleys.
(B) Pastoralists were the developers and disseminators of new weapons and of new modes of
transportation that transformed warfare.
(C) Pastoralists, who brought Islam to Mesopotamia, were the connection between the regions
known as Upper and Lower Egypt.
(D) Pastoralists brought chariot racing to Mesopotamia and to the Nile Valley in order to
provide entertainment for the masses in the form of bread and circuses.

Answer: B
Topic: Pastoralists as disseminators of technology; Transportation and warfare
Theme: Interaction Between Humans and the Environment; Creation, Expansion and Interaction of
Economic Systems
Learning Objective: ENV–6; ECON–10
Historical Thinking Skill: Analyzing Evidence: Content and Sourcing; Causation
Key Concept: 1.3.II.C

6
Copyright © 2017, 2015, 2011, 2007 Pearson Education, Inc. All rights reserved.
Questions 1.10–1.12 refer to the images below.

Statue Known to the West as the Sphinx and to the Arabs as the Father of
Terror Early Chinese Script on Stone

Capture of Babylon

1.10. What role did culture play in unifying states?


(A) States had natural boundaries and the people were unified through language and customs.
(B) States were unified through laws, language, literature, religion, myths, and monumental art.
(C) States were unified through strong militaries; culture played no significant role.

7
Copyright © 2017, 2015, 2011, 2007 Pearson Education, Inc. All rights reserved.
(D) States faced continual warfare and were unified through common goals to defeat their
enemies.

Answer: B
Topic: Role of art in unifying early cultures
Theme: Development and Interaction of Cultures
Learning Objective: CUL–9
Historical Thinking Skill: Synthesis; Causation
Key Concept: 1.3.III.C

1.11. How did rulers and religious elites maintain power?


(A) by sponsoring monumental art and architecture, such as ziggurats, pyramids, and temples
(B) by commanding powerful armies to keep control over aristocrats
(C) by permitting frequent revolts and peasant uprisings to spur the formation of dictators
(D) by paying workers on large projects significant amounts of money for their labor

Answer: A
Topic: Monumental architecture; Characteristics of first states
Theme: Development and Interaction of Cultures; State Building, Expansion, and Conflict
Learning Objective: CUL–8; SB–2; SB–5
Historical Thinking Skill: Analyzing Evidence: Content and Sourcing; Synthesis
Key Concept: 1.3.III.A

1.12. What helped facilitate the rule of early state governments over the people?
(A) diverse populations and rulers who employed the millet system
(B) constant warfare and the reliance of people on kings for protection
(C) aristocrats who sent their children to be educated and to learn to read and write cuneiform
(D) development of legal codes, such as the Code of Hammurabi and the Code of Ur-Nammu

Answer: D
Topic: Legal codes developed
Theme: State Building, Expansion, and Conflict
Learning Objective: SB–1
Historical Thinking Skill: Analyzing Evidence: Content and Sourcing
Key Concept: 1.3.III.A

8
Copyright © 2017, 2015, 2011, 2007 Pearson Education, Inc. All rights reserved.
Questions 1.13–1.15 refer to the maps below.

China in the Shang and Zhou Eras

9
Copyright © 2017, 2015, 2011, 2007 Pearson Education, Inc. All rights reserved.
India in the Age of Harappa and the Early Aryan Migrations

10
Copyright © 2017, 2015, 2011, 2007 Pearson Education, Inc. All rights reserved.
Mesopotamia in Maps

1.13. What evidence supports the claim that trade expanded and interregional trade networks existed
at this time?
(A) Remnants of silk have been found in Viking graves.
(B) The Harappa left detailed accounts of their trade networks across the region.
(C) Precious stones from China and Southeast Asia have been found at the Harappan site.
(D) Maize from the Americas has been found in burial sites of the Shang era.

Answer: C
Topic: Trade and cultural interaction; Expanding trade routes, from local to regional; Local, regional, and
interregional trade
Theme: Development and Interaction of Cultures; State Building, Expansion, and Conflict; Creation,
Expansion, and Interaction of Economic Systems
Learning Objective: CUL–2; SB–9; ECON–12
Historical Thinking Skill: Analyzing Evidence: Content and Sourcing
Key Concept: 1.3.III.E

1.14. What was the effect of increased trade between Mesopotamia and Egypt?
(A) As people came into contact with others, marriages increased and new social classes arose.
(B) Goods, technological ideas, and cultural ideas were exchanged across the regions.
(C) Egyptians adopted the Code of Hammurabi as the basis of their political system.
(D) Mesopotamians begin to grow potatoes and maize as a result of increased trade.

Answer: B

11
Copyright © 2017, 2015, 2011, 2007 Pearson Education, Inc. All rights reserved.
Topic: Trade and cultural interaction; Expanding trade routes, from local to regional; Local, regional, and
interregional trade
Theme: Development and Interaction of Cultures; State Building, Expansion, and Conflict; Creation,
Expansion, and Interaction of Economic Systems
Learning Objective: CUL–2; SB–9; ECON–12
Historical Thinking Skill: Analyzing Evidence: Content and Sourcing
Key Concept: 1.3.III.E

1.15. Which new religions that emerged at this time continued to have strong influences in later
periods?
(A) animism, Hebrew monotheism, Christianity, and Confucianism
(B) Vedic religions in India, Hebrew monotheism, and Zoroastrianism
(C) Hebrew monotheism, Zoroastrianism, and Mahayana Buddhism
(D) Vedic religions in India, Mahayana Buddhism, and Sunni and Shi’a Islam

Answer: B
Topic: New religious beliefs
Theme: Development and Interaction of Cultures
Learning Objective: CUL–1
Historical Thinking Skill: Analyzing Evidence: Content and Sourcing
Key Concept: 1.3.III.D

12
Copyright © 2017, 2015, 2011, 2007 Pearson Education, Inc. All rights reserved.
Section II
Short-Answer Question
1.16 Use the image and map below and your knowledge of world history to answer all parts of the
question that follows.

Basalt Olmec Head

Early Centers of Civilization

(A) Discuss the political structures of the civilizations in the Americas.


(B) Identify the social structures of the civilizations in the Americas.
(C) Explain the interconnected economic structures of the early civilizations in the Americas.

13
Copyright © 2017, 2015, 2011, 2007 Pearson Education, Inc. All rights reserved.
Sample Answers

(A) Students might discuss that the Olmecs were the first civilization in the Americas. They
developed in Central America along the coast of the Gulf of Mexico. Their society and
government would lay a foundation that others would follow; for example, the Mayans
would build upon Olmec achievements. The Olmecs are believed to be the first culture in
the Americas to build cities and to have had large religious centers. The Chavin civilization
developed in the Andean region. However, the Olmecs controlled larger regions in
Mesoamerica than did the Chavin in the Andes. Overall, as both civilizations grew, the focus
was on temple complexes in the cities, where monumental architecture played a prominent
role. This led to the increased importance of religion in ceremonies. Both developed political
bureaucracies and religious hierarchies. Large carved Olmec heads are believed to have
been used in Olmec religious centers. Artistic styles and beliefs spread across the regions,
with jaguars and snakes being common themes.

(B) Students might explain that hunter-gather societies evolved into agricultural societies. As
settlements became permanent and local farming communities grew into villages,
specialization of labor resulted due to the food surplus. Because their economies were
based on sedentary agriculture, the surplus of goods may have resulted in increasing social
stratification and complexity. Over time, with the increase in population, social hierarchies,
with priests, priesthoods, merchants, and a warlike nobility, evolved. Both societies were
increasingly patriarchal. Large Olmec heads are thought to represent rulers or people with
hereditary elite status due to the amount of labor required to create and transport the
sculptures. Agricultural workers were the commoners, and kinship groups were also
important. Economic life based on sedentary agriculture that required irrigation may have
led to increasing social complexity and more sophisticated political forms.

(C) Students might mention that both the Olmec and the Chavin cultures engaged in trade. The
Olmecs developed irrigation techniques for agriculture, growing food such as maize, beans,
squash, and peppers, which they traded across the Central American region, especially for
jade stones. The Chavin developed irrigation techniques, and maize was introduced from
Central America as a result of trade networks. The population increased when potatoes and
quinoa were adapted for farming at higher elevation levels leading to increased
specialization of labor. In addition to trading llama products, Chavin peoples were known for
their pottery, which was traded across the region. Quipu was used for record keeping by the
Chavin; the Olmecs devised a system of writing for record keeping. Overall, early civilizations
in the Americas developed a surplus of goods that may have led both to the unequal
distribution of goods and to trade, which resulted in more contact with other peoples.
Olmec art, which has been found across Central America, is testimony to extensive Olmec
trade networks. Large urban centers developed as populations increased.

Topic: Trade and cultural interaction; Patriarchy; Early civilizations; Monumental architecture;
Development of elites; Expansion of trade; Urban hierarchies; Agriculture, artisans, specialization; Social
structures; Patriarchy and social hierarchies
Theme: Development and Interaction of Cultures; State Building, Expansion, and Conflict; Creation,
Expansion, and Interaction of Economic Systems; Development and Transformation of Social Structures
Learning Objective: CUL–2; CUL–3; CUL–4; SB–5; SB–6; SB–10; ECON–2; ECON–5; ECON–8; SOC–1
Historical Thinking Skill: Causation; Synthesis; Analyzing Evidence: Content and Sourcing

14
Copyright © 2017, 2015, 2011, 2007 Pearson Education, Inc. All rights reserved.
Key Concept: 1.3.I; 1.3.III.B; 1.2.I.A; 1.2.I.B

15
Copyright © 2017, 2015, 2011, 2007 Pearson Education, Inc. All rights reserved.
Section III
Long Essay Question
Question 1.17: Compare how the environment contributed to the cultural and technological
achievements and to the religious beliefs of two of the following river valley civilizations, during the
period from c. 8000 B.C.E. to c. 600 BCE.

Chinese—————Indus—————Mesopotamian

Sample Answer

This question is asking students to compare how the environment contributed to the cultural,
technological, and religious achievements/beliefs in two river valley civilizations. Students should
understand that compare also means contrast and that they are required to provide evidence and
analysis for both comparative and contrasting viewpoints but not necessarily evenly.

River valley civilizations had many similarities, including developing successful agricultural societies as a
result of their geography and location along rivers. China’s civilization along the Huang He River and the
Indus civilization along the Indus River were both similar in that each river carried rich soils that enriched
the farmland downstream: loess in China and rich soils in the Indus Valley that came from the
mountains. Although all three civilizations experienced flooding, flooding could not be predicted with
accuracy, and, as a result, people began to organize projects to dredge rivers (China) and work on
technological improvements, including irrigation systems and flood-control projects. Mesopotamia
arose along the Tigris and Euphrates.

While the rivers brought potential devastation, they were also responsible for the rise of each
civilization. As a result of a favorable environment to grow crops, each river valley civilization increased
its agricultural output, which led, in turn, to significant population increase. China had wheat and rice
agricultural regions; various grains were grown in the Indus Valley; and a variety of crops, such as barley,
wheat, beans, were grown in Mesopotamia. In addition to food crops, the Indus Valley had a large
granary, which most likely served as the center of political authority. The Indus Valley civilization also
grew cotton and developed an early textile industry.

In each civilization, agricultural surpluses led to job specialization, to the rise of class distinctions, such as
warriors, nobles, priests, rulers, etc., and to the development of rich cultural traditions and large urban
areas. Kings in Mesopotamia began to claim descent from the gods to justify their rule. As men became
the main providers, a patriarchal society evolved in all three civilizations, and women in Mesopotamia
began to wear the veil. Also, as populations increased due to the agricultural surplus, there were more
levels of society to manage, which led to the evolution of law codes that helped create cultural unity in
India and in Mesopotamia (Code of Hammurabi). (Writing had been developed for record keeping.)
Writing also led to a more elaborate culture. The Zhou in China ruled by decree and believed in a
mandate from heaven to justify their rule. Floods, famines, and other natural disasters could undermine
the ruling mandate and a new dynasty would arise.

As populations increased, large urban cultural centers arose in all three civilizations. Because
Mesopotamia does not have geographic barriers, Mesopotamian authorities organized labor to build

16
Copyright © 2017, 2015, 2011, 2007 Pearson Education, Inc. All rights reserved.
defensive walls, in addition to building ziggurats and temples, in a manner similar to the organization of
labor in China to construct flood projects. In the Indus Valley, large cities included homes with running
water, in addition to a large pool in the city complex thought to be for religious purposes. Control of
surplus goods was important, and, in Mesopotamia, a large cult of priests and cultural centers arose.
Priests would intervene with the gods to ensure good fortune and good harvests and to make
astronomical calculations needed for irrigation systems. Regarding belief systems, in the Indus Valley,
worship of trees and animals was important, because they represented vital forces. The early Hebrews
in Mesopotamia, similarly, recognized nature spirits in trees, rocks, and mountains.

The proximity of minerals to craft copper, tin, and iron, and the creation of new tools, weapons, wheels,
and technology, led to more power and wealth in all three civilizations. In China the ruling elites
controlled bronze production, which helped them maintain their power. When iron technology spread
to China, the Zhou did not control it as successfully. Their subordinates who established iron works
impacted their power. Because of their locations along rivers, transportation technology was developed
for shipping and trade. India had access to precious stones, which were traded across the region,
bringing great wealth.

To gain the thesis point, the student must address all parts of the question, qualifying comparisons and
contrasts in the given timeframe. Qualifying categories for comparison and contrast are the roles that
the environment played in the development of culture, technology, and religious beliefs during this
period.

A strong essay will not only provide evidence of comparison of environmental effects but also include
analysis of the impacts relative to similarities and differences.

In this question, the targeted historical thinking skill of comparison is accomplished by providing specific
evidence to compare and contrast two regions. Students must describe similarities and differences
about historical developments, relating developments to environmental factors while analyzing the
reasons for similarities and/or differences. Students must evaluate the significance of the effects to gain
two points.

Two points can be gained through extensive use of evidence (eight pieces of evidence minimum) that
supports the thesis and that establishes clear linkages between the evidence and the thesis.

Students must make direct comparisons and address both parts of the question but do not have to
address them evenly.

There are many ways that the synthesis point can be earned. Students may discuss connections to
economics or to another field of discipline that supports their thesis in order to earn the point. This
question offers the student many options.

Topic: Agriculture; Iron use; Environmental settings; First states; Architecture; River civilizations and the
first states; Agricultural diversity; New religious beliefs; Patriarchy; Legal codes developed; Favorable
environmental factors; Social structures; Patriarchy and social structures
Theme: Creation, Expansion, and Interaction of Economic Systems; Development and Transformation of
Social Structures; Interaction Between Humans and the Environment; Development and Interaction of
Cultures; State Building, Expansion, and Conflict
Learning Objective: ECON–8; SOC–1; ENV–1; ENV–2; ENV–4; ENV–5; CUL–1; CUL–3; SB–1; SB–4

17
Copyright © 2017, 2015, 2011, 2007 Pearson Education, Inc. All rights reserved.
Historical Thinking Skill: Comparison, Contextualization, Synthesis, Analyzing Evidence: Content and
Sourcing
Key Concept: 1.2; 1.2.I.D; 1.2.II.A; 1.2.II.B; 1.2.II.C; 1.3.I; 1.3.II.A; 1.3.II.B; 1.3.II.C; 1.3.III.A; 1.3.III.B;
1.3.III.C; 1.3.III.D; 1.3.III.E; 1.3.III.F

18
Copyright © 2017, 2015, 2011, 2007 Pearson Education, Inc. All rights reserved.
Section IV
Document-Based Question
Directions: The following question is based on the accompanying documents. The documents have been
edited for the purpose of this exercise.

In your response, you should do the following:

• State a relevant thesis that directly addresses all parts of the question.

• Support the thesis or a relevant argument with evidence from all, or all but one, of the
documents.

• Source and analyze the significance of at least four of the documents on the following basis:
intended audience, author’s purpose, historical context, format or medium, and/or point of view.

• Develop context by relating your argument to broader historical events or processes.

• Synthesize the elements above into a persuasive essay that extends your argument by connecting
it to

• a different historical period or geographical region


• another course theme or historical approach that is not the focus of the essay (such as
political, social, economic, or intellectual history)
• another perspective from a different academic discipline (such as economics, art history,
anthropology, or government)

19
Copyright © 2017, 2015, 2011, 2007 Pearson Education, Inc. All rights reserved.
Question 1.18: Using the documents and your knowledge of world history, identify the characteristics of
ancient Chinese culture that led to its success.

Historical Background: Early river valley civilizations rose in China along the Huang He River. Struggles
for power led to the Period of Warring States. China was rich with mineral deposits and produced
agricultural surpluses, leading to an increased population and power.

Document 1
Confucianism: Government and the Superior, 551–479 B.C.E.
IDEAL GOVERNMENT

The Master said, “When rulers love to observe the rules of propriety, the people respond readily to the
calls on them for service.”

The Master said, “If the people be led by laws, and uniformity sought to be given them by punishments,
they will try to avoid the punishment, but have no sense of shame.

“If they be led by virtue, and uniformity sought to be given them by the rules of propriety, they will have
the sense of shame, and moreover will become good.”

The Master said, “He who exercises government by means of his virtue may be compared to the north
polar star, which keeps its place and all the stars turn towards it.”

The duke Ai asked, saying, “What should be done in order to secure the submission of the people?”
Confucius replied, “Advance the upright and set aside the crooked, then the people will submit. Advance
the crooked and set aside the upright, then the people will not submit.”

Ji Kang asked how to cause the people to reverence their ruler, to be faithful to him, and to go on to
nerve themselves to virtue. The Master said, “Let him preside over them with gravity; — then they will
reverence him. Let him be filial and kind to all; — then they will be faithful to him. Let him advance the
good and teach the incompetent; — then they will eagerly seek to be virtuous.” Ji Kang asked Confucius
about government. Confucius replied, “To govern means to rectify. If you lead on the people with
correctness, who will dare not to be correct?”

The Master said, “If a minister makes his own conduct correct, what difficulty will he have in assisting in
government? If he cannot rectify himself, what has he to do with rectifying others?”

The Master said, “If good men were to govern a country in succession for a hundred years, they would
be able to transform the violently bad, and dispense with capital punishments.” True indeed is this
saying!

20
Copyright © 2017, 2015, 2011, 2007 Pearson Education, Inc. All rights reserved.
Document 2
China from the Later Zhou Era to the Han Era

Document 3
The Great Wall

21
Copyright © 2017, 2015, 2011, 2007 Pearson Education, Inc. All rights reserved.
 

 
Document  4  
Li  Si,  “Burn  the  Books!”    
In  earlier  times  the  empire  disintegrated  and  fell  into  disorder,  and  no  one  was  capable  of  unifying  it.  
Thereupon  the  various  feudal  lords  rose  to  power.  In  their  discourses  they  all  praised  the  past  in  order  
to  disparage  the  present  and  embellished  empty  words  to  confuse  the  truth.  Everyone  cherished  his  
own  favorite  school  of  learning  and  criticized  what  had  been  instituted  by  the  authorities.  But  at  present  
Your  Majesty  possesses  a  unified  empire,  has  regulated  the  distinctions  of  black  and  white  and  has  
firmly  established  for  yourself  a  position  of  sole  supremacy.  And  yet  these  independent  schools,  joining  
with  each  other,  criticize  the  codes  of  laws  and  instructions.  Hearing  of  the  promulgation  of  a  decree,  
they  disapprove  of  it  in  their  hearts;  going  out  they  criticize  it  in  the  thoroughfare.  They  seek  a  
reputation  by  discrediting  their  sovereign;  they  appear  superior  by  expressing  contrary  views,  and  they  
lead  the  lowly  multitude  in  the  spreading  of  slander.  If  such  license  is  not  prohibited,  the  sovereign  
power  will  decline  above  and  partisan  factions  will  form  below.  It  would  be  well  to  prohibit  this.  
 
Your  servant  suggests  that  all  books  in  the  imperial  archives,  save  the  memoirs  of  Qin,  be  burned.  All  
persons  in  the  empire,  except  members  of  the  Academy  of  Learned  Scholars,  in  possession  of  the  Book  
of  Odes,  the  Book  of  History,  and  discourses  of  the  hundred  philosophers  [Confucian  scholars]  should  
take  them  to  the  local  governors  and  have  them  indiscriminately  burned.  Those  who  dare  to  talk  to  each  
other  about  the  Book  of  Odes  and  the  Book  of  History  should  be  executed  and  their  bodies  exposed  in  
the  market  place.  Anyone  referring  to  the  past  to  criticize  the  present  should,  together  with  all  
members  of  his  family,  be  put  to  death.  Officials  who  fail  to  report  cases  that  have  come  under  their  
attention  are  equally  guilty.  After  thirty  days  from  the  time  of  issuing  the  decree,  those  who  have  not  
destroyed  their  books  are  to  be  branded  and  sent  to  build  the  Great  Wall.  Books  not  to  be  destroyed  
will  be  those  on  medicine  and  pharmacy,  divination  by  the  tortoise  and  milfoil,  and  agriculture  and  
arboriculture.  People  wishing  to  pursue  learning  should  take  the  officials  as  their  teachers.  
 
Document  5   Document  6  
Bronze  Axe  Head  from  the  Shang  Dynasty   Bronze  Vessel  from  the  Shang  Dynasty  

   

22  
Copyright  ©  2017,  2015,  2011,  2007  Pearson  Education,  Inc.  All  rights  reserved.  
Sample Answer

An acceptable thesis would consist of one or more congruent sentences in the opening paragraph that
present a plausible analysis of how the characteristics of ancient Chinese culture led to its success. For
instance, students could organize the documents around the contribution of ancient law and
government, the impact of China’s barriers, its massive construction projects, its bronze metallurgy,
and/or Confucianism. An excellent thesis would include sophisticated analysis relating to the period of
warring states, the role that belief systems played, the power of the empire to control large numbers of
people, and the advantages that bronze metallurgy brought to the empire culturally, politically,
technologically, militarily, and economically.

To achieve all four points in the evidence category of the rubric, the student will need to relate at least
five of the documents to the thesis and analyze each by addressing historical context, audience,
purpose, or point of view of each document. It is always wise to use all documents by relating the
evidence back to the thesis and (1) addressing the intended audience and point of view or purpose or (2)
placing the document in the context of what was happening. For instance, Document 1 could be used in
different ways. Students might discuss how the contributions to ancient government and law ended
political and social instability associated with the period. They might also connect Document 1 with
Document 4 with an analysis of the role legalism played. Students might reference Document 1 in
regards to the Confucian belief in responsibility and civil service. The audience for Documents 1 and 4
would be the Chinese population, with the purposes of demonstrating good government and
encouraging proper behavior. References to Document 2 could include the positive impact of China’s
barriers, as in protecting the people from invaders and in growing the empire, but could also note the
impact that barriers had in creating a sense unity among the Chinese people while developing its unique
artistic styles and culture. References to Document 3 might note the authority the Chinese government
commanded through the control of large amounts of labor for building projects. Students could group
this with Document 2, regarding barriers, or mention that bronze technology required a large labor force
to mine and refine ore, as large labor forces to build monumental architecture were similarly controlled.
Documents 5 and 6 could easily be grouped, as both images demonstrate the wealth, artistic expression,
and technology the empire enjoyed, along with its high level of metalworking ability. Students might
discuss the unique artwork and connect this uniqueness back to the role that geographic barriers played
(Documents 2 and 3). An excellent comment would include outside information relating to the control
the empire held over bronze metalworking and the access to resources.

There are plenty of outside examples students could use to support their thesis. The synthesis point
could be gained by discussing the characteristics of Chinese culture and unity. Students might connect
the increase of agriculture and agricultural surplus to the rise of larger urban centers and to state
building. They might discuss the role geography played in the expansion of empires or the role that
trade played in the economy and rise of the empires.

Topic: Architecture; New religious beliefs; Early civilizations; Role of art in unifying early urban cultures;
Monumental architecture
Theme: Interaction Between Humans and the Environment; Development and Interaction of Cultures;
State Building, Expansion, and Conflict
Learning Objective: ENV–2; CUL–1; CUL–4; CUL–9; SB–2; SB–5
Historical Thinking Skill: Analyzing Evidence: Content and Sourcing; Contextualization; Synthesis
Key Concept: 1.3.III.A; 1.3.III.C; 1.3.III.D

23
Copyright © 2017, 2015, 2011, 2007 Pearson Education, Inc. All rights reserved.
Period 2
Organization and Reorganization of Human Societies
(c. 600 B.C.E. to c. 600 C.E.)
Section I
Multiple-Choice Questions
Questions 2.1–2.4 refer to the image below.

Eighteenth-Century Artist’s Rendering of Confucius, Laozi, and the Buddha

2.1. How does Confucianism differ from Daoism and Buddhism?


(A) Confucianism is regarded, not as a religion, but as more of a secular belief system.
(B) Confucianism is based on the teachings of a central figure.
(C) Confucianism has gained many adherents outside of East Asia.
(D) Confucianism is not a belief system native to China.

Answer: A
Topic: Religion and Culture
Theme: Development and Interaction of Cultures
Learning Objective: CUL–2

24
Copyright © 2017, 2015, 2011, 2007 Pearson Education, Inc. All rights reserved.
Historical Thinking Skill: Comparison
Key Concept: 2.1.I; 2.1.II

2.2. Which of the following statements constitutes an accurate comparison between Judaism and
Hinduism?
(A) Both religions were tolerant of other faiths and often combined belief systems seamlessly.
(B) Both religions employed strict social and religious hierarchies that defined society.
(C) Both religions adhered to reincarnation and the multiple manifestation of gods.
(D) Both religions organized their beliefs through the codification of holy scriptures.

Answer: D
Topic: The Formation of Hinduism; Judaism
Theme: Development and Interaction of Cultures
Learning Objective: CUL–4
Historical Thinking Skill: Comparison
Key Concept: 2.1.I

2.3. Christianity and Buddhism gained their greatest number of converts because of __________.
(A) their underlying teachings of militarism and conquest
(B) their adoption by powerful politicians
(C) their recognition of reincarnation as a religious tenet
(D) their belief in the separation of religion and society

Answer: B
Topic: Patterns of Roman History; The Mauryan Dynasty
Theme: State Building, Expansion, and Conflict
Learning Objective: SB–2
Historical Thinking Skill: Comparison
Key Concept: 2.1.II

2.4. Buddhism and Christianity offered an alternative to existing social structures by __________.
(A) allowing women to be ordained as priests
(B) teaching that women and men were equals in marriage
(C) allowing men to practice monasticism
(D) teaching that women, not only men, should study and interpret holy scriptures

Answer: C
Topic: Religion and Culture
Theme: Development and Interaction of Cultures; Development and Transformation of Social Structures
Learning Objective: CUL–2; SOC–5
Historical Thinking Skill: Comparison
Key Concept: 2.1.III

25
Copyright © 2017, 2015, 2011, 2007 Pearson Education, Inc. All rights reserved.
Questions 2.5–2.7 refer to the image below.

Battle Between Alexander the Great and King Darius, Roman Mosaic, House of Faun, Macedonia

2.5. The popular classical-period artistic medium used to create The Battle Between Alexander the
Great and King Darius is best described as __________.
(A) the technique of mural painting executed upon freshly laid plaster
(B) the technique of carving an image into the surface of a block of wood
(C) the technique of shaping a ceramic material by hand or by dry-pressing it into an object
(D) the technique of assembling an image from small pieces of glass or stone

Answer: D
Topic: Religion and Culture
Theme: Development and Interaction of Cultures
Learning Objective: CUL–8; CUL–9
Historical Thinking Skill: Contextualization
Key Concept: 2.1.II

2.6. In what way was Persian society different from Greek society?
(A) Persian society rejected widespread participation in politics.
(B) Persian society rejected warfare as a means of expansion.
(C) Persian society rejected the incorporation of roads to facilitate movement.
(D) Persian society rejected toleration of foreign languages and cultures.

Answer: A
Topic: The Persian Tradition
Theme: State Building, Expansion, and Conflict
Learning Objective: SB–1; SB–4; SB–6; SB–8

26
Copyright © 2017, 2015, 2011, 2007 Pearson Education, Inc. All rights reserved.
Historical Thinking Skill: Comparison
Key Concept: 2.2.II

2.7. What belief did followers of Zoroastrianism, which strongly influenced Judaism, Christianity, and
Islam, share?
(A) a belief in many gods
(B) a belief in a day of judgment
(C) a belief in heaven, but not hell
(D) a belief in animal sacrifice

Answer: B
Topic: The Persian Tradition
Theme: Development and Interaction of Cultures
Learning Objective: CUL–2
Historical Thinking Skill: Comparison
Key Concept: 2.1.I; 2.1.II

27
Copyright © 2017, 2015, 2011, 2007 Pearson Education, Inc. All rights reserved.
Questions 2.8–2.10 refer to map below.

Greece and Greek Colonies of the World, c. 431 B.C.E.

2.8. Which Greek city-state dominated maritime trade and warfare in the Aegean Sea?
(A) Athens
(B) Epirus
(C) Sparta
(D) Persia

Answer: A
Topic: Patterns of Greek History
Theme: Interaction Between Humans and the Environment; State Building, Expansion, and Conflict
Learning Objective: ENV–4; SB–5
Historical Thinking Skill: Interpretation
Key Concept: 2.2.I; 2.2.IV

2.9. The Athenian government in the fifth century B.C.E. was __________.
(A) a theocracy
(B) a republic
(C) a direct democracy
(D) an oligarchy

Answer: C
Topic: Greek and Roman Political Institutions
Theme: State Building, Expansion, and Conflict

28
Copyright © 2017, 2015, 2011, 2007 Pearson Education, Inc. All rights reserved.
Learning Objective: SB–1; SB–2
Historical Thinking Skill: Argumentation
Key Concept: 2.2.II

2.10. Which of the following statements best describes the result of the Peloponnesian War?
(A) The Spartan victory ushered in a common military tradition across the Greek peninsula.
(B) Both Sparta and Athens were badly damaged and unable to build on past successes.
(C) The Athenian victory led to a new age of artistic, political, and cultural supremacy.
(D) Neither Sparta nor Athens was able to claim victory; existing alliances, therefore, persisted.

Answer: B
Topic: Patterns of Greek History
Theme: State Building, Expansion, and Conflict
Learning Objective: SB–4; SB–5; SB–8
Historical Thinking Skill: Causation
Key Concept: 2.2.I; 2.2.IV

29
Copyright © 2017, 2015, 2011, 2007 Pearson Education, Inc. All rights reserved.
Questions 2.11–2.13 refer to the image below.

A Roman Warship

2.11. What had to occur before Rome was able to dominate the Mediterranean Sea?
(A) The Romans had to defeat the Germanic tribes to procure enough timber for shipbuilding.
(B) The Romans had to defeat their Carthaginian adversaries in North Africa.
(C) The Romans had to enslave Greek shipbuilders to acquire necessary technologies and skills.
(D) The Romans had to acquire Greek fire from Byzantine alchemists to suppress naval rivals.

Answer: B
Topic: Patterns of Roman History
Theme: State Building, Expansion, and Conflict
Learning Objective: SB–10
Historical Thinking Skill: Causation
Key Concept: 2.2.I

2.12. In the Roman upper classes, women __________.


(A) commanded little influence and power in the household
(B) held less authority and respect than did women in Chinese civilization
(C) gained respect through their religious influence
(D) were held inferior under the law and in the culture

Answer: D

30
Copyright © 2017, 2015, 2011, 2007 Pearson Education, Inc. All rights reserved.
Topic: Rome
Theme: Development and Transformation of Social Structures
Learning Objective: SOC–1; SOC–3; SOC–4; SOC–5
Historical Thinking Skill: Comparison
Key Concept: 2.1III; 2.2.III

2.13. Identify a cause of the decline and collapse of the Roman Empire.
(A) the shift of the capital from Rome to Avignon
(B) the construction of trade networks
(C) the issues associated with imperial succession
(D) the retreat of Gothic peoples from the north

Answer: C
Topic: Patterns of Roman History
Theme: State Building, Expansion, and Conflict
Learning Objective: SB–3; SB–4; SB–8
Historical Thinking Skill: Causation
Key Concept: 2.2.IV

31
Copyright © 2017, 2015, 2011, 2007 Pearson Education, Inc. All rights reserved.
Questions 2.14–2.16 refer to the two poems below.

Poems by Two Buddhist Nuns


MUTTA
So free am I, so gloriously free,
Free from three petty things —
From mortar, from pestle and from my twisted lord,
Freed from rebirth and death I am,
And all that has held me down
Is hurled away.

SUMANGALAMATA
A woman well set free! How free I am,
How wonderfully free, from kitchen drudgery.
Free from the harsh grip of hunger,
And from empty cooking pots,
Free too of that unscrupulous man,
The weaver of sunshades.
Calm now, and serene I am,
All lust and hatred purged.
To the shade of the spreading trees I go
And contemplate my happiness.
Used by permission from Uma Chakravarti and Kumkum Roy.

2.14. What can be inferred about the freedom described in each poem?
(A) that the woman is now free to choose her own husband
(B) that the woman is now the equal of men
(C) that the woman is now free from the roles imposed on her by a patriarchal society
(D) that the woman is now free to pursue occupations of her own choosing

Answer: C
Topic: Inequality as the Social Norm
Theme: SOC–1; SOC–3; SOC–5
Learning Objective: Development and Transformation of Social Structures
Historical Thinking Skill: Interpretation
Key Concept: 2.1.I; 2.1.III; 2.2.III

2.15. How were Indian and Chinese societies different?


(A) Indian society was tolerant of several religions whereas Chinese society held a single belief
system.
(B) Merchants had higher status in Indian society than allowed in Chinese society.
(C) Chinese society was based on a more rigid social hierarchy than Indian society.
(D) Chinese society was highly patriarchal whereas Indian society had less gender bias.

Answer: B
Topic: China and India Compared

32
Copyright © 2017, 2015, 2011, 2007 Pearson Education, Inc. All rights reserved.
Theme: Development and Transformation of Social Structures
Learning Objective: SOC–2
Historical Thinking Skill: Comparison
Key Concept: 2.2.III

2.16. Identify a difference between the Gupta and Mauryan dynasties of classical India.
(A) The Gupta dynasty was less centralized than the Mauryan.
(B) The Gupta rulers moved away from the strict caste system implemented by the Mauryan.
(C) The Gupta dynasty was able to build a more extensive bureaucracy than the Mauryan.
(D) The Gupta rulers were able to extend their borders beyond the subcontinent whereas the
Mauryan could not.

Answer: A
Topic: Political Institutions
Theme: State Building, Expansion, and Conflict
Learning Objective: SB–2
Historical Thinking Skill: Comparison
Key Concept: 2.2.I; 2.2.II; 2.2.III; 2.2.IV

33
Copyright © 2017, 2015, 2011, 2007 Pearson Education, Inc. All rights reserved.
Questions 2.17–2.20 refer to the map below.

Patterns of Trade in the Indian Ocean Trade Network

2.17. What items would the Mauryan dynasty have most likely donated to the Indian zone of the Indian
Ocean trade network?
(A) spices and cotton
(B) silk and porcelain
(C) gold and tapestries
(D) silver and paper

Answer: A
Topic: The Pattern of Trade in the Ancient Eurasian World
Theme: Creation, Expansion, and Interaction of Economic Systems
Learning Objective: ECON–10; ECON–12
Historical Thinking Skill: Contextualization
Key Concept: 2.3.I; 2.3.III

2.18. The influence of classical India on classical China was most visible in what area?
(A) in maritime trade
(B) in art and religion
(C) in military incursion
(D) in agriculture and science

Answer: B
Topic: Indian Influence and Comparative Features
Theme: Development and Interaction of Cultures

34
Copyright © 2017, 2015, 2011, 2007 Pearson Education, Inc. All rights reserved.
Learning Objective: CUL–2; CUL–6; CUL–9
Historical Thinking Skill: Contextualization
Key Concept: 2.1.I; 2.1.II; 2.3.III

2.19. The Kamasutra was a fourth-century manual that detailed __________.


(A) aspects of etiquette for higher-caste males
(B) procedures for intermarriage between castes
(C) efficient navigation methods for Indian Ocean trade
(D) administrative techniques for controlling landlords

Answer: A
Topic: Buddhism
Theme: Development and Transformation of Social Structures
Learning Objective: SOC–5
Historical Thinking Skill: Argumentation
Key Concept: 2.1.I; 2.1.II

2.20. Which society provided the fewest opportunities for social mobility in the classical world?
(A) Greece
(B) China
(C) Egypt
(D) India

Answer: D
Topic: Inequality as the Social Norm
Theme: Development and Transformation of Social Structures
Learning Objective: SOC–3; SOC–4
Historical Thinking Skill: Comparison
Key Concept: 2.2.III; 2.2.IV

35
Copyright © 2017, 2015, 2011, 2007 Pearson Education, Inc. All rights reserved.
Questions 2.21–2.25 refer to the image below.

The Great Wall of China

2.21. One of the following statements about the Great Wall is false. Which statement is false?
(A) The Great Wall was constructed by a strong centralized government.
(B) The Great Wall was built by soldiers enslaved in wars with other Asian empires.
(C) The Great Wall was located in the north of China to keep nomadic invaders at bay.
(D) The Great Wall was originally an earthen structure that was improved upon over time.

Answer: B
Topic: The Qin Dynasty
Theme: State Building, Expansion, and Conflict
Learning Objective: SB–4; SB–6
Historical Thinking Skill: Argumentation
Key Concept: 2.2.II; 2.2.IV

2.22. How did Confucianism work to bring harmony and balance to Chinese society?
(A) by instituting harsh legal codes
(B) by defining roles and relationships
(C) by using religion to unify social classes
(D) by consolidating power within the aristocracy

Answer: B
Topic: Religion and Culture
Theme: Development and Transformation of Social Structures
Learning Objective: SOC–3
Historical Thinking Skill: Argumentation
Key Concept: 2.1.II

2.23. What did the Han dynasty use Confucianism to do?

36
Copyright © 2017, 2015, 2011, 2007 Pearson Education, Inc. All rights reserved.
(A) to form strong interregional trade relationships
(B) to create an extensive and efficient bureaucracy
(C) to demonstrate the importance of a strong military
(D) to continue the social reforms instituted by the Qin

Answer: B
Topic: Political Institutions
Theme: State Building, Expansion, and Conflict
Learning Objective: SB–2; SB–4
Historical Thinking Skill: Argumentation
Key Concept: 2.2.II

2.24. Which of the following statements accurately describes how the Zhou dynasty differed from the
Qin and Han dynasties?
(A) Literacy rates were higher under the Zhou dynasty than under either the Qin or the Han
dynasty.
(B) The Zhou dynasty ruled through a system of local alliances unlike either the Qin or the Han
dynasty.
(C) The Zhou dynasty was more technologically advanced than either the Qin or the Han
dynasty.
(D) The territory under the Zhou dynasty was more vast than the territory of either the Qin or
the Han dynasty.

Answer: B
Topic: Patterns in Classical China
Theme: State Building, Expansion, and Conflict
Learning Objective: SB–1
Historical Thinking Skill: Patterns of Continuity and Change over Time
Key Concept: 2.2.II

2.25. Which of the following belief systems is not native to China?


(A) Daoism
(B) Legalism
(C) Buddhism
(D) Confucianism

Answer: C
Topic: Religion and Culture
Theme: Development and Interaction of Cultures
Learning Objective: CUL–1
Historical Thinking Skill: Argumentation
Key Concept: 2.1.II; 2.1.III

37
Copyright © 2017, 2015, 2011, 2007 Pearson Education, Inc. All rights reserved.
Questions 2.26–2.28 refer to the passage below.

I humbly propose that all historical records but those of Qin be burned. If anyone who is not a court
scholar dares to keep the ancient songs, historical records or writings of the hundred schools, these
should be confiscated and burned by the provincial governor and army commander. Those who in
conversation dare to quote the old songs and records should be publicly executed; those who use old
precedents to oppose the new order should have their families wiped out; and officers who know of
such cases but fail to report them should be punished in the same way.

If thirty days after the issuing of this order the owners of these books have still not had them destroyed,
they should have their face tattooed and be condemned to hard labor at the Great Wall. The only books
which need not be destroyed are those dealing with medicine, divination, and agriculture. Those who
want to study the law can learn it from the officers. The emperor sanctioned this proposal.
Records of the Grand Historian [Translated by Yang Hsien-yi and Gladys Yang in 1961], c. 85 B.C.E.

2.26. With which classical Chinese school of thought would one expect the passage from the Records of
the Grand Historian to be associated?
(A) Buddhism
(B) Confucianism
(C) Daoism
(D) Legalism

Answer: D
Topic: The Qin Dynasty
Theme: State Building, Expansion, and Conflict
Learning Objective: SB–1; SB–2; SB–4
Historical Thinking Skill: Interpretation
Key Concept: 2.2.I; 2.2.II; 2.2.III; 2.2.IV

2.27. The Chinese literary tradition was __________.


(A) influenced by Shinto teachings from Japan
(B) rooted in political theory that did not focus on poetry
(C) based on new interpretations of earlier classical literature
(D) lacking in comparison with its Indian and Greek counterparts

Answer: C
Topic: Literature, Art, and Science
Theme: Development and Interaction of Cultures
Learning Objective: CUL–8
Historical Thinking Skill: Causation
Key Concept: 2.1.III

2.28. On what were the most important technological advancements in classical China focused?
(A) on trade, to fulfill aristocratic desires for foreign goods
(B) on increasing agricultural yields, to support the population
(C) on mining iron, for the manufacture of weapons
(D) on papermaking, to satisfy the growing record-keeping needs of the bureaucracy

38
Copyright © 2017, 2015, 2011, 2007 Pearson Education, Inc. All rights reserved.
Answer: B
Topic: Trade and Technology
Theme: Creation, Expansion, and Interaction of Economic Systems
Learning Objective: ECON–12
Historical Thinking Skill: Contextualization
Key Concept: 2.3.II

39
Copyright © 2017, 2015, 2011, 2007 Pearson Education, Inc. All rights reserved.
Questions 2.29–2.30 refer to the map below.

Trade Routes at the End of the Classical Era

2.29. Which of the following statements about the function of nomadic intermediaries along the
mapped trade routes is false?
(A) Nomads provided caravans with protection from people threatening to cut off trade.
(B) Nomads were important producers of manufactured goods sold in urban areas.
(C) Nomads were instrumental in the exchange of religion and aspects of culture.
(D) Nomads supplied draft animals to transport the goods of merchants.

Answer: B
Topic: Nomads and Cross-Civilization Contacts and Exchanges
Theme: Creation, Expansion, and Interaction of Economic Systems
Learning Objective: ECON–10
Historical Thinking Skill: Contextualization
Key Concept: 2.3.III

2.30. Identify an aspect of Japanese innovation that differed from innovations of other classical Afro-
Eurasian societies.
(A) Japan developed a character-based language that was not related to other Asian languages.
(B) Japan established an entirely secular belief system that focused on social harmony.
(C) Japan constructed a largely urbanized society that relied more on trade than on agriculture.
(D) Japan began developing iron tools without first utilizing bronze and copper.

Answer: D
Topic: Japan and Northern Europe

40
Copyright © 2017, 2015, 2011, 2007 Pearson Education, Inc. All rights reserved.
Theme: Creation, Expansion, and Interaction of Economic Systems
Learning Objective: ECON–12
Historical Thinking Skill: Comparison
Key Concept: 2.3.II

2.31. Which of the following statements is an accurate description of Mesoamerican society during the
classical period?
(A) Mesoamericans established the ability to create large monuments that rivaled those of
Egypt.
(B) Mesoamericans largely demonstrated nomadic tendencies and little social stratification.
(C) Mesoamericans proved to be excellent maritime traders who made regular contact with
Polynesia.
(D) Mesoamerica was the most densely populated region of the classical world.

Answer: A
Topic: The Americas
Theme: Creation, Expansion, and Interaction of Economic Systems; Development and Transformation of
Social Structures
Learning Objective: ECON–3; SOC–2
Historical Thinking Skill: Comparison; Contextualization
Key Concept: 2.2.III

41
Copyright © 2017, 2015, 2011, 2007 Pearson Education, Inc. All rights reserved.
Questions 2.32–2.34 refer to the image below.

Icon of the Emperor Constantine Accompanied by the Bishops of the First Council of Nicaea

2.32. How did the Council of Nicaea demonstrate a difference between Christianity and the eastern
religions of Hinduism and Buddhism?
(A) Christianity held little toleration for diversity in religious doctrine.
(B) Eastern religions were more closely tied to the state than was Christianity.
(C) Christianity was more open to variations of faith than were eastern religions.
(D) Eastern religions were more highly centralized than was Christianity.

Answer: A
Topic: Christianity Gains Ground
Theme: State Building, Expansion, and Conflict; Development and Interaction of Cultures
Learning Objective: SB–2; CUL–4

42
Copyright © 2017, 2015, 2011, 2007 Pearson Education, Inc. All rights reserved.
Historical Thinking Skill: Comparison
Key Concept: 2.1.I; 2.1.II

2.33. Which of the following statements is an accurate description of Christian society in the classical
period?
(A) Christianity prohibited unity among different social groups and classes.
(B) Christianity preached that men and women were social and political equals.
(C) Christianity was spurned by political leaders and social elites because of its disorganization.
(D) Christianity taught equality of the souls of both sexes and allowed them to worship
together.

Answer: D
Topic: Christianity Gains Ground
Theme: Development and Transformation of Social Structures
Learning Objective: SOC–3; SOC–5
Historical Thinking Skill: Patterns of Continuity and Change over Time
Key Concept: 2.1.II

2.34. Identify a cause of the collapse of the Roman Empire.


(A) Advanced desertification in Egypt degraded the Roman tax base.
(B) Mortality rates in the Roman Empire lagged far behind birth rates.
(C) Christian popes openly called for the overthrow of Roman rulers.
(D) The programmed succession of emperors created political stability.

Answer: A
Topic: The Decline and Fall of Rome
Theme: State Building, Expansion, and Conflict
Learning Objective: SB–3; SB–4
Historical Thinking Skill: Causation
Key Concept: 2.2.IV

43
Copyright © 2017, 2015, 2011, 2007 Pearson Education, Inc. All rights reserved.
Section II
Short-Answer Question
2.35. Use the image, map, and passage below and your knowledge of world history to answer all parts
of the question that follows.

Seventh-Century Buddhist Major Religions of the Modern World –This map shows the contemporary geography of the
Pilgrim major religions.

44
Copyright © 2017, 2015, 2011, 2007 Pearson Education, Inc. All rights reserved.
Behold, these are the tools of the spiritual craft, which, if they be constantly employed day and night,
and duly given back on the Day of Judgement, will gain for us from the Lord that reward which He
Himself has promised—“which eye hath not seen, nor ear heard; nor hath it entered into the heart of
man to conceive what God hath prepared for them that love him.” And the workshop where we are to
labour diligently at all these things is the cloister of the monastery, and stability in the community.

OF OBEDIENCE
The first degree of humility is obedience without delay. This becomes those who hold nothing dearer to
them than Christ, and who on account of the holy servitude which they have taken upon them, and for
fear of hell, and for the glory of life everlasting, as soon as anything is ordered by the superior, just as if it
had been commanded by God Himself, are unable to bear delay in doing it. It is of these that the Lord
says: “At the hearing of the ear he hath obeyed me.” And again, to teachers he saith: “He that heareth
you heareth me.”

OF THE RECEPTION OF GUESTS


Let all guests that come be received like Christ Himself, for He will say: “I was a stranger and ye took me
in.” And let fitting honour be shown to all, especially, however, to such as are of the household of the
faith and to pilgrims.

Excerpts from the Rule of St. Benedict, Sixth Century

(A) Both Buddhism and Christianity were born out of preexisting faiths. Briefly explain how both
Buddhism and Christianity differed from their predecessors. Provide one example for each
religion.
(B) Briefly explain the roles of monasticism in Buddhism and Christianity.
(C) Briefly explain one similarity in the rapid expansion of Buddhism and Christianity during the
classical period.

Sample Answers

(A) Students might include the following differences in their answers:


(1) Differences Between Buddhism and Hinduism
Belief in a central figure (Siddhartha Gautama/Buddha)
A new set of teachings (the Four Noble Truths)
A new avenue for salvation (escaping the cycle of reincarnation with Nirvana)
Opposition to the caste system (more equal treatment of genders and classes)
Belief in the advocacy of deceased holy people (bodhisattvas)

(2) Differences Between Christianity and Judaism


Belief in a new central figure (Jesus of Nazareth)
A new set of teachings (the New Testament)
A new avenue for salvation (escaping death with the belief in Jesus as the messiah)
Equal treatment of men and women in the eyes of God (both sexes could worship
together)
Belief in the advocacy of deceased holy people (saints)

(B) Students might include the following roles of monasticism in their answers:

45
Copyright © 2017, 2015, 2011, 2007 Pearson Education, Inc. All rights reserved.
Monasticism helped to spread the faith.
Establishment of monastic religious communities centered on prayer and meditation.
Monasteries provided earthly examples of aestheticism and holiness.
Monasteries became centers of sacred and secular teaching and learning.
Monasticism offered men and women an avenue to withdraw from society.

(C) Students might include the following similarities in the rapid expansion of Buddhism and
Christianity in their answers:
Both religions developed devoted and determined groups of disciples.
Merchants carried the faiths along overland trade routes like the Silk Road.
The Mediterranean Sea and Indian Ocean trade networks provided further efficient diffusion
of these religions.
The established networks of roads in the Roman and Gupta empires provided efficient
diffusion.
Both religions had the acceptance of powerful emperors (e.g., Ashoka/Constantine).

Topic: Religion and Culture; Buddhism; Christianity Gains Ground


Theme: Development and Interaction of Cultures; Development and Transformation of Social
Structures; State Building, Expansion, and Conflict
Learning Objective: CUL–2; CUL–4; SOC–5; SB–2
Historical Thinking Skill: Comparison; Synthesis; Analyzing Evidence: Content and Sourcing
Key Concept: 2.1.I; 2.1.II; 2.1.III

46
Copyright © 2017, 2015, 2011, 2007 Pearson Education, Inc. All rights reserved.
Section III
Long Essay Question
Question 2.36: Compare the techniques that states utilized to unify their populations during the
classical period (c. 600 B.C.E. to c. 600 C.E.) in two of the following regions.

Southwest Asia—————South Asia—————East Asia

Sample Answer

The question is asking students to identify and compare the techniques that states utilized to unify their
populations within their territories. Students may compare Southwest Asia with South Asia, Southwest
Asia with East Asia, or South Asia with East Asia. Students should understand that compare also means
contrast and that they are required to provide evidence and analysis for both, but not necessarily
evenly. To gain the thesis point the student must address all parts of the question by qualifying
comparisons and contrasts within the given timeframe. Qualifying categories for comparison and
contrast must associate evidence related to unifying techniques utilized in two of the provided regions.
A strong essay will not only provide evidence related to unifying techniques utilized by states but also
analyze the impact of this evidence on these areas.

Points gained through the use of evidence shall be determined by the amount of evidence used. To gain
the maximum two points, students need to cite at least five pieces of evidence. Examples of evidence
and analysis that students might utilize to gain points are described in the paragraphs that follow.

The Persians, Mauryan, Qin, and to a lesser extent the Han all used authoritarian regimes to claim large
swaths of territory in order to establish empires. Mauryan, Qin, and Persian rulers were very
authoritarian in nature and rejected wide participation in politics, which curtailed political diversity.

All of these regimes used a variety of techniques to unify the diverse populations of people, facilitate
good relations, and quell uprisings. The Persians, Qin, Han, and Mauryan created centralized
governments and established capital cities from where attempts to unify their populations would flow.
In order to maintain their rule, Persian emperors divided their realm into regions and appointed local
rulers, or satraps, to enforce the wishes of the state. The capital would regularly send spies to more
distant regions of the empire to ensure that satraps were loyal and not allied with local political forces.
Qin Shi Huangdi realized that the decentralized nature of China permitted regional landlords to exercise
too much power. In order to curtail their power, he ordered nobles to leave their regional strongholds
and spend extended periods of time at court. He was then able to send loyal officials to oversee the
whole of China.

Qin Shi Huangdi used brutal but effective Legalist tactics to quell internal disorder. Legalism led to a
regimented Chinese society where nonconformists were met with harsh punishments. The Han dynasty
did not use the brutal tactics of the Qin. Rather than Legalism they promoted Confucianism at an
unprecedented level. The Confucian belief system permeated every level of Chinese society, which led
to an exceptional level of unity amongst the Chinese people. The Han continued many of the unification
processes of their predecessors and grew the centralized bureaucracy to a level that was not matched
until modern history. Other leaders of this time period also utilized belief systems as a means of

47
Copyright © 2017, 2015, 2011, 2007 Pearson Education, Inc. All rights reserved.
unification. The Persians used the religion of Zoroastrianism, which spread widely during the late Persian
period. Zoroastrianism proved to be an effective technique to unify the people of the empire. Persia was
also known for being a tolerant state that embraced a host of languages and cultures in addition to
Zoroastrianism. Toleration may seem to be a technique that argues against unification, but pleasing the
masses does, ultimately, ensure loyalty and suppresses the urge of people to revolt. The Mauryan
emperor Chandragupta and the Gupta rulers used the preexisting Hindu religion, with its strict ethical
and social codes, to coalesce their realms. In contrast, Ashoka, the grandson of Chandragupta,
eventually converted to Buddhism. He used the religion to pacify and unite the diverse people of India.
While honoring Hindu traditions, Ashoka propagated Buddhism throughout the known reaches of Asia.
The Zhou also promoted orderly religious practices and discouraged primitive practices. This led to more
unified forms of worship in early China.

The Gupta and Zhou featured decentralized political structures. Through joint military operations with
powerful regional families, they were able to solidify rule over central areas of South and East Asia,
respectively. The Gupta state did little to unify India to the degree that the Mauryans did. They
preferred to negotiate with local princes and intermarry with local families to loosely fuse the regions of
their empire. The Guptas ruled a smaller area of the subcontinent and employed a smaller bureaucracy
than their predecessors as well. The Zhou dynasty, which ruled from preclassical times through the year
256 B.C.E., solidified their influence between the Huanghe and the Yangzi, which became China’s core—
often called the Middle Kingdom. As the Zhou expanded beyond this area, the dynasty had to rely on the
loyalty of regional supporters. Perhaps the greatest aspect of unity under the Zhou came in the form of
the Mandarin language. The spread of this dialect by the Zhou dynasty resulted in the largest single
language group in the world at the time. Educated officials, who began to rely on Mandarin, used the
unifying power of a single language to achieve a more efficient bureaucratic model, which would have
far-reaching implications for China. Oral stories and epics also began to be recorded in written form as a
result of this common cultural currency instituted by the Zhou. The Han utilized this common language
base to further centralize China. The Han instituted judicial systems, civil service exams, and the keeping
of historical records. This placed the state in a position to be the primary unifying force and the keeper
of Chinese beliefs, which contributed to a united heritage.

Regardless of which realm or reign, each of these heads of state created an intricate network of roads
and highways to facilitate trade, diplomacy, tax collection, and the movement of the military.
Chandragupta and Ashoka seem to have incorporated Hellenistic and Persian models of military
organization. The Mauryan maintained large armies, with thousands of chariots and elephant-borne
troops, and pressed claims to the whole of the subcontinent except the very tip of India. The Persians
also unified their empire through use of an imposing and diverse military that utilized infantry, cavalry,
and chariots, among other types of forces. The Qin and Han were able to mobilize armies as well. They
crushed regional resistance and extended their borders. In addition to the military acumen of the Qin
and Han, these dynasties were able to demonstrate unity through their ability to conscript labor to build
and maintain, not only the Great Wall but other public works, such as canals, palaces, and granaries.

In this question, one point can be gained through the students’ use of the targeted historical thinking
skill of comparison. This can be accomplished by describing similarities and differences between two of
the regions specified in the question. An additional one point can also be secured if students use specific
examples of similarities and differences or analyze reasons for similarities and/or differences.

There are many ways that students can earn one point for synthesis. One way is for students to make
connections to other state-driven techniques used to unify populations by political entities in other

48
Copyright © 2017, 2015, 2011, 2007 Pearson Education, Inc. All rights reserved.
periods and regions. A good example of this would be to compare the durable political and cultural
traditions first seen in classical Persian and Han states with their regional descendants—Iran and the
Tang dynasty. Students may also choose to discuss additional qualifiers that explicitly figured into
techniques used for unification (e.g., politics, religions, society, or the environment).

Topic: Patterns in Classical China; Political Institutions; Patterns in Classical India; The Persian Tradition
Theme: State Building, Expansion, and Conflict
Learning Objective: SB–1; SB–2; SB– 4; SB–5; SB–6; SB–8; SB–9; SB–10
Historical Thinking Skill: Comparison
Key Concept: 2.1.II; 2.2.I; 2.2.II; 2.2.III; 2.2.IV

49
Copyright © 2017, 2015, 2011, 2007 Pearson Education, Inc. All rights reserved.
Section IV
Document-Based Question
Directions: The following question is based on the accompanying documents. The documents have been
edited for the purpose of this exercise.

In your response, you should do the following:

• State a relevant thesis that directly addresses all parts of the question.

• Support the thesis or a relevant argument with evidence from all, or all but one, of the
documents.

• Source and analyze the significance of at least four of the documents on the following basis:
intended audience, author’s purpose, historical context, format or medium, and/or point of view.

• Develop context by relating your argument to broader historical events or processes.

• Synthesize the elements above into a persuasive essay that extends your argument by connecting
it to:

• a different historical period or geographical region


• another course theme or historical approach that is not the focus of the essay (such as
political, social, economic, or intellectual history)
• another perspective from a different academic discipline (such as economics, art history,
anthropology, or government)

50
Copyright © 2017, 2015, 2011, 2007 Pearson Education, Inc. All rights reserved.
Question 2.37: Using the documents and your knowledge of world history, describe what caused the
Roman Empire in the West to fall.

Historical Background: The decline of the Roman Empire was already evident by 180 C.E. and was due to
a wide-range of factors that can be blamed on large, all- encompassing forces that would have been hard
for any society to control. The last western Roman emperor was removed in 476.

Document 1
Excerpt from Strayer, Gatzke, and Harbison, The Course of Civilization, 1961
The basic trouble was that very few inhabitants of the empire believed that the old civilization was
worth saving . . . the overwhelming majority of the population had been systematically excluded from
political responsibilities. They could not organize to protect themselves; they could not serve in the
army . . . Their economic plight was hopeless. Most of them were serfs bound to the soil, and the small
urban groups saw their cities slipping into uninterrupted decline.

Document 2
Excerpt from Edward Gibbon, The Decline and Fall of the Roman Empire, 1776–1788
The decline of Rome was the natural and inevitable effect of immoderate greatness (large size). . . . The
introduction . . . of Christianity, had some influence on the decline and fall of the Roman Empire. The
clergy successfully preached the doctrine of patience; the active virtues of society were discouraged;
and the last remains of military spirit were buried in the cloister; a large portion of public & private
wealth was consecrated to the . . . demands of charity and devotion.

Document 3
Excerpt from Herbert J. Muller, Uses of the Past, 1952
First the economic factor . . . [w]hile the empire was expanding, its prosperity was fed by plundered
wealth and by new markets in the semi-barbaric provinces. When the empire ceased to expand,
however, economic progress soon ceased. . . .The abundance of slaves led to growth of the latifundia,*
the great estates that . . . came to dominate agriculture and ruin the free coloni (farmers) who drifted to
the cities, to add to the unemployment there. The abundance of slaves kept wages low.

*latifundia—a large landed estate or ranch in ancient Rome typically worked by slaves

Document 4
Excerpt from Indro Montanelli, Romans Without Laurels, 1962
Rome, like all great empires, was not overthrown by external enemies but undermined by internal
decay. . . .The military crisis was the result of . . . proud old aristocracy’s . . . shortage of children.
(Consequently) foreigners poured into the . . . Roman army [which was] composed entirely of Germans.

51
Copyright © 2017, 2015, 2011, 2007 Pearson Education, Inc. All rights reserved.
Document 5
Roman Empire, c. 180 C.E.

Document 6
Excerpt from Sidonius Apollinaris (Bishop of Clermont, c. 430–485), Rome’s Decay and a Glimpse of the New Order, c. 470.
TRANSLATION: Finley Hooper and Matthew Schwartz, eds., Roman Letters: History from a Personal Point of View (Detroit, MI:
Wayne State University, 1991), 272–277.

Rumour has it that the Goths have occupied Roman soil. . . . It is our fate to furnish fuel to the fire of a
peculiar hatred, for, by Christ’s aid, we are the sole obstacle to the fulfilment of their ambition to extend
their frontier to the Rhone. . . . Their menacing power has long pressed us hard; it has already
swallowed up whole tracts of territory round us, and threatens to swallow more. We mean to resist with
spirit, though we know our peril and the risks which we incur. But our trust is not in our poor walls
impaired by fire, or in our rotting palisades, or in our ramparts worn by the breasts of the sentries, as
they lean on them in continual watch.

Sample Answer

An acceptable response would utilize five or six documents (all or all but one of the documents
provided) to deliver an analysis of the decline of the western Roman Empire. The thesis, which should
consist of one or more congruent sentences in the opening paragraph or in the conclusion, should
present various reasons for the decline. An example of a good thesis might assert that, although there
are wide-ranging causes for the decline of the western Roman Empire, these documents assert that the
empire became too large to control, the introduction of Christianity changed the values of Roman

52
Copyright © 2017, 2015, 2011, 2007 Pearson Education, Inc. All rights reserved.
society, the composition of the military no longer produced a viable force, and the structural shift in the
agricultural system led to economic decline.

Analysis of the documents should present evidence to support the thesis. Students might organize the
documents in a variety of ways around the factors cited in the thesis that contributed to the decline of
the western Roman Empire. To achieve all four points in the evidence category of the rubric, students
will need to relate at least five of the documents to the thesis and analyze each by addressing historical
context, audience, purpose, and/or the point of view of each document. For example, in Document 1,
students might focus on the fact that citizens gave up devotion to the old civilization and, because
people felt disenfranchised from political and military processes, didn’t believe that the empire was
worth saving. In this same document it is noted that the economic situation, both urban and rural, is
also deemed hopeless. Document 2 corroborates that the military was in decline and that new Christian
teachings shifted beliefs away from support of empire. The sheer size of the empire is also mentioned as
a contributing factor to decline as well. Muller, in Document 3, argues that once the expansion of the
empire ceased, so did its prosperity. The large privately owned estates were now being worked by the
large slave population, which led now-unemployed farmers to flock to urban areas where they could not
find work. Large numbers of unemployed farmers in urban areas placed a large strain on already limited
resources. Document 4 notes that the internal factors of decline outweighed external forces. The
military was now composed of mercenary foreigners, and a distant ruling class was not capable of
managing the empire nor of taking care of the empire’s inhabitants. The map in Document 5 illustrates
the expansive nature of the Roman Empire in the year 180 C.E., which students may tie to an inability to
govern such a large territory effectively. Lastly, the tone of Sidonius Apollinaris, in Document 6, is that of
a Christian bishop witnessing the final agonizing years of the western Roman Empire. He discusses his
willingness to resist foreign invaders but also provides a glimpse into the decaying infrastructure that
surrounds him.

Strong essays will not simply list descriptions or analysis of the documents but rather make connections
between the documents in order to provide arguments that support the thesis statement. Some
examples of these connections that are consistent with the sample thesis provided above are (1)
comparison of how ineffective the military had become due to an influx of foreigners not loyal to Rome
as discussed in Documents 1, 2, 4, and 6; (2) aspects contributing to the decline of the economy, such as
the growth of slavery and unemployment, as evident in Documents 1, 2, and 3; (3) the unmanageable
size of the western Roman Empire as can be inferred using Documents 2 and 5; and (4) the effect of
Christianity on the declining empire as can be concluded using Documents 1, 2, and 6.

When writing the response to this document-based question, it is necessary for students to provide
outside examples and knowledge to support their thesis. An example of outside knowledge that isn’t
discussed in the documents provided is the recurrent theme of succession and the intervention of the
army in these decisions. Another example that students often and correctly use is that of plagues
leading to a declining population, which, in turn, affected the empire’s tax revenues.

The contextualization point could be earned by commenting on how the end of the western Roman
Empire was more severe than the decline of both classical China and India, because Rome had not
produced an enduring belief system nor a bureaucratic model that might have allowed for revival or
even elements of unity amid political fragmentation.

53
Copyright © 2017, 2015, 2011, 2007 Pearson Education, Inc. All rights reserved.
An example that students commonly use to gain the synthesis point is to discuss how the empire’s
political, cultural, and economic power had been passed to Constantinople and the Byzantine Empire,
which would exist for almost another thousand years.

Topic: The Decline and Fall of the Roman Empire


Theme: State Building, Expansion, and Conflict; Creation, Expansion, and Interaction of
Economic Systems; Development and Interaction of Cultures
Learning Objective: SB–1; SB–2; SB–3; SB–4; SB–8; ECON–2; ECON–5; ECON–6; CUL–1; CUL–5
Historical Thinking Skill: Causation; Comparison; Contextualization; Argumentation; Analyzing Evidence:
Content and Sourcing; Interpretation; Synthesis
Key Concept: 2.1.II; 2.1.III; 2.2.II-IV; 2.3.III

54
Copyright © 2017, 2015, 2011, 2007 Pearson Education, Inc. All rights reserved.
Period 3
Regional and Interregional Interactions
(c. 600 C.E. to c. 1450)
Section I
Multiple-Choice Questions
Questions 3.1–3.4 refer to the image below.

Conversion of the Russia’s King Vladimir I, 988

3.1. Based on your knowledge of world history, for which major reason would a leader, such as
Vladimir I, have his people convert to a single religion?
(A) to unify a geographically large populace
(B) to ensure the salvation of his people
(C) to enhance trade relations with regions practicing the same religion
(D) to gain favor with refugees looking to settle in new areas

Answer: A
Topic: Russia Turns to Christianity
Theme: State Building, Expansion, and Conflict; Development and Interaction of Cultures
Learning Objective: SB–4; CUL–4
Historical Thinking Skill: Causation
Key Concept: 3.1.III

55
Copyright © 2017, 2015, 2011, 2007 Pearson Education, Inc. All rights reserved.
3.2. What action is taking place in the image of King Vladimir I?
(A) an Islamic profession of faith
(B) a Jewish bar mitzvah
(C) a Christian baptism
(D) a Daoist wedding

Answer: C
Topic: Russia Turns to Christianity
Theme: Development and Interaction of Cultures
Learning Objective: CUL–4
Historical Thinking Skill: Interpretation
Key Concept: 3.1.III

3.3. During the reign of Vladimir I, the prosperous trading center of Kiev was best known for which
commodities?
(A) silk and cotton
(B) furs and wood
(C) precious metals
(D) exotic animals

Answer: B
Topic: New Patterns of Trade
Theme: Creation, Expansion, and Interaction of Economic Systems
Learning Objective: ECON–2
Historical Thinking Skill: Comparison
Key Concept: 3.1.I

3.4. In the twelfth century, which of the following contributed least directly to the fall of the Kievan
principality?
(A) reduced manufacturing capability of luxury items
(B) royal disagreement over succession
(C) rapid decline of Byzantium as a trade partner
(D) invasions from Asia

Answer: A
Topic: Kievan Decline
Theme: State Building, Expansion, and Conflict; Development and Interaction of Cultures
Learning Objective: SB–9; CUL–4
Historical Thinking Skill: Causation
Key Concept: 3.3.II

56
Copyright © 2017, 2015, 2011, 2007 Pearson Education, Inc. All rights reserved.
Questions 3.5–3.8 refer to the image below.

One of the Earliest Known Artistic Renderings of a Ninth-Century Arab Army at War

3.5. Why did Islamic armies begin to avoid mass conversion of conquered people?
(A) Leaders believed that any further conversions might dilute religious teachings.
(B) Muslims believed that only a finite number of people could receive salvation.
(C) The number of soldiers in the Arab army was already too large to support.
(D) Converts would have been entitled to a share of the booty collected.

Answer: D
Topic: Motives for Arab Conquests
Theme: State Building, Expansion, and Conflict
Learning Objective: SB–3; SB–10
Historical Thinking Skill: Causation
Key Concept: 3.1.III; 3.2.I

3.6. Why did the Christian Copts and Nestorians prefer Arab invaders to their former Byzantine
overlords?
(A) because the Arabs did not tax these Christian sects
(B) because the Arabs were more tolerant of these Christian sects than were the Byzantines
(C) because the Arabs allowed these Christian sects to trade from Mediterranean ports
(D) because the Arabs promoted members of these Christian sects to high positions within the
Umayyad administration

57
Copyright © 2017, 2015, 2011, 2007 Pearson Education, Inc. All rights reserved.
Answer: B
Topic: Weaknesses of the Adversary Empires
Theme: Development and Interaction of Cultures
Learning Objective: CUL–4
Historical Thinking Skill: Contextualization
Key Concept: 3.1.I

3.7. Which statement about entitlements bestowed on women under Islam is false?
(A) Islam forbade female infanticide.
(B) Islam permitted women to take up to four husbands.
(C) Islam strengthened the legal rights of women in matters of divorce and inheritance.
(D) Under Islam, bride prices were paid to future wives instead of future husbands.

Answer: B
Topic: Marriage and Family in Pre-Islamic Arabia
Theme: Development and Interaction of Cultures
Learning Objective: CUL–5
Historical Thinking Skill: Patterns of Continuity and Change over Time
Key Concept: 3.3.III

3.8. What element of faith or religious practice did Islam have in common with Judaism and
Christianity?
(A) monotheism
(B) prayer five times a day
(C) fasting during the holy month of Ramadan
(D) belief in Jesus as the Son of God

Answer: A
Topic: Universal Elements in Islam
Theme: Development and Interaction of Cultures
Learning Objective: CUL–1
Historical Thinking Skill: Comparison
Key Concept: 3.1.III

58
Copyright © 2017, 2015, 2011, 2007 Pearson Education, Inc. All rights reserved.
Questions 3.9–3.13 refer to the image below.

Illustration of the Great Aztec City-State of Tenochtitlan

3.9. Given that their capital city was located on a heavily populated island, what innovation did the
Aztecs originate to provide adequate farmland to support their capital’s inhabitants and increase
agricultural yield?
(A) The Aztecs made use of floating platforms of fertile soil to compensate for the lack of space.
(B) The Aztecs relied on their extensive merchant class to negotiate with allies for additional
land.
(C) The Aztecs utilized their military to procure adjacent lands for the use of its residents.
(D) The Aztecs depended on their nobility to redistribute their lands for the good of the
common people.

Answer: A
Topic: Feeding the People: The Economy of the Empire
Theme: Interaction Between Humans and the Environment
Learning Objective: ENV–8
Historical Thinking Skill: Causation
Key Concept: 3.3.I.A

3.10. Which of the following was not an important continuity between the Toltec and Aztec
civilizations?
(A) the strong militaristic ethic of both civilizations
(B) the emphasis on human sacrifice in religious practices
(C) the use of the same capital city for administrative purposes
(D) the use of similar gods in in religious practices

Answer: C
Topic: The Toltec Heritage and Aztec Rise to Power

59
Copyright © 2017, 2015, 2011, 2007 Pearson Education, Inc. All rights reserved.
Theme: Development and Interaction of Cultures
Learning Objective: CUL–3; CUL– 4l CUL–5
Historical Thinking Skill: Patterns of Continuity and Change over Time
Key Concept: 3.2.I.A

3.11. Which religious aspects of Aztec and Hindu belief systems are similar?
(A) Both belief systems incorporated male and female forms of deities.
(B) Both belief systems utilized a strict caste-like system to organize believers.
(C) Both belief systems emphasized the importance of sacred rivers.
(D) Both belief systems included human sacrifice in their rituals.

Answer: A
Topic: Religion and the Ideology of Conquest
Theme: Development and Interaction of Cultures
Learning Objective: CUL–1
Historical Thinking Skill: Comparison; Contextualization
Key Concept: 3.1.III

3.12. What did the Incan concept of split inheritance do?


(A) The Incan concept of split inheritance ensured equal distribution of wealth among all
nobility having blood relation to the leader.
(B) The Incan concept of split inheritance served as a form of death tax that required all
inhabitants of the empire to bequeath half of their wealth to the state.
(C) The Incan concept of split inheritance mitigated family quarrels because wealth accrued by
Incan families had to be distributed equally among all adult descendants.
(D) The Incan concept of split inheritance contributed to the ongoing and rapid expansion of the
empire because it led the dead Inca’s male relatives to support the cult of the Inca’s mummy
for eternity.

Answer: D
Topic: Conquest and Religion
Theme: Development and Interaction of Cultures
Learning Objective: CUL–4
Historical Thinking Skill: Argumentation
Key Concept: 3.2.I

3.13. How were the Incan and Aztec civilizations alike?


(A) Both civilizations successfully wiped out all traditions of their preceding cultures.
(B) Both civilizations were less successful in their application of military and imperial
organization than their preceding cultures.
(C) Both civilizations had a well-developed merchant class that was responsible for trade.
(D) Both civilizations successfully adapted to their unique environments to suit their agricultural
needs.

Answer: D
Topic: Comparing Incas and Aztecs
Theme: Development and Interaction of Cultures
Learning Objective: CUL–1

60
Copyright © 2017, 2015, 2011, 2007 Pearson Education, Inc. All rights reserved.
Historical Thinking Skill: Comparison; Contextualization
Key Concept: 3.1.III

61
Copyright © 2017, 2015, 2011, 2007 Pearson Education, Inc. All rights reserved.
Questions 3.14–3.17 refer to the passage below.

SOURCE: Richard A. Newhall, ed., Jean Birdsall, trans., The Chronicle of Jean de Venette (New York: Columbia University Press,
1953).

This plague, it is said, began among the unbelievers, came to Italy, and then crossing the Alps reached
Avignon, where it attacked several cardinals and took from them their whole household. Then it spread,
unforeseen, to France, through Gascony and Spain, little by little, from town to town, from village to
village, from house to house, and finally from person to person. . . . During the epidemic, God of His
accustomed goodness deigned to grant this grace, that however suddenly men died, almost all awaited
death joyfully. Nor was there anyone who died without confessing his sins and receiving the holy
viaticum. . . .

Some said that this pestilence was caused by infection of the air and waters, since there was at this time
no famine nor lack of food supplies, but on the contrary great abundance. As a result of this theory of
infected water and air as the source of the plague the Jews were suddenly and violently charged with
infecting wells and water and corrupting the air. The whole world rose up against them cruelly on this
account. In Germany and other parts of the world where Jews lived, they were massacred and
slaughtered by Christians, and many thousands were burned everywhere, indiscriminately.

Jean de Venette (Member of the Clergy), On the Progress of the Black Death in France

3.14. Why was the Jewish population often violently targeted in postclassical Europe?
(A) Jews had a known proficiency of formulating and working with poisons throughout their
history.
(B) Jews, after migrating to Europe from the Middle East, were a religious minority and viewed
with suspicion.
(C) Jews worked predominately in agrarian occupations and had access to water sources.
(D) Jews infuriated Christians because of their aggressive missionary efforts.

Answer: B
Topic: The Decline of the Medieval Synthesis
Theme: Interaction Between Humans and the Environment; Creation, Expansion, and Interaction of
Economic Systems
Learning Objective: ENV–7; ECON–10
Historical Thinking Skill: Patterns of Continuity and Change over Time
Key Concept: 3.1.IV

3.15. Where did the Black Death, or bubonic plague, originate?


(A) in western Europe, where it was passed to other regions by Christian missionaries
(B) in Africa, where it entered western Europe with groups of Islamic missionaries
(C) in Asia, where it entered western Europe through Mediterranean trade routes
(D) in eastern Europe, where it was passed to other regions by military campaigns

Answer: C
Topic: The Decline of the Medieval Synthesis
Theme: Interaction Between Humans and the Environment; Creation, Expansion, and Interaction of
Economic Systems

62
Copyright © 2017, 2015, 2011, 2007 Pearson Education, Inc. All rights reserved.
Learning Objective: ENV–7; ECON–10
Historical Thinking Skill: Causation
Key Concept: 3.1.IV

3.16. Which of the following statements best explains why commoners from postclassical Europe might
be motivated to leave their families and engage in the Crusades?
(A) The Pope had promised that death on the battlefield would ensure entry into heaven.
(B) The prospect of acquiring vast riches in the Middle East drew many commoners to become
Crusaders.
(C) Overpopulation in western Europe pushed commoners eastward to seek new lands to
conquer.
(D) Invaders from Asia were in the process of forcing commoners to resettle in areas unfit for
farming.

Answer: A
Topic: The West’s Expansionist Impulse
Theme: Development and Interaction of Cultures
Learning Objective: CUL–5
Historical Thinking Skill: Causation
Key Concept: 3.3.III

3.17. Gothic architecture, which benefited from Muslim influence and spread throughout postclassical
Europe, was characterized by __________.
(A) lintels, columns, and pediments
(B) thick walls, small windows, and domes
(C) pointed arches, large windows, and flying buttresses
(D) perpendicular construction, oriel windows, and corbels

Answer: C
Topic: Religious Themes in Art and Literature
Theme: Development and Interaction of Cultures
Learning Objective: CUL–8
Historical Thinking Skill: Contextualization
Key Concept: 3.1.III

63
Copyright © 2017, 2015, 2011, 2007 Pearson Education, Inc. All rights reserved.
Questions 3.18–3.22 refer to the passage below.

SOURCE: Boston University African Studies Center, Kingdom of Mali, Primary Source Documents,
http://www.bu.edu/africa/outreach/k_o_mali/.

From the beginning of my coming to stay in Egypt I heard talk of the arrival of this sultan Musa . . . and
found the Cairenes eager to recount what they had seen of the Africans’ prodigal spending. I asked the
emir Abu . . . and he told me of the opulence, manly virtues, and piety of his sultan. “When I went out to
meet him [he said] that is, on behalf of the mighty sultan al-Malik al-Nasir, he did me extreme honour
and treated me with the greatest courtesy. He addressed me, however, only through an interpreter
despite his perfect ability to speak in the Arabic tongue. Then he forwarded to the royal treasury many
loads of unworked native gold and other valuables.”

Description of the Visit to Cairo, in 1324, by the King of Mali, Mansa Musa, Written by Al-Umari, Who Visited Cairo Several
Years After the Mansa Musa’s Visit

3.18. Why would the Egyptians have come into contact with Mansa Musa?
(A) Mansa Musa was a prolific traveler and writer.
(B) Egypt was an important center of learning.
(C) Mansa Musa made a pilgrimage to Mecca.
(D) Egypt was a popular vacation destination.

Answer: C
Topic: The Empire of Mali and Sundiata, the “Lion Prince”
Theme: Development and Interaction of Cultures
Learning Objective: CUL–2; CUL–3; CUL– 7
Historical Thinking Skill: Causation
Key Concept: 3.1.III

3.19. Why did the kingdom of Songhay, the successor state to Mali, begin to thrive economically?
(A) because caravans from west Africa moved gold and salt through Songhay territories
(B) because Songhay maritime expeditions came into contact with the Swahili Coast
(C) because Christian kingdoms supported Songhay authority over Islamic states
(D) because Songhay was the first territory to outlaw any form of slavery

Answer: A
Topic: The Songhay Kingdom
Theme: State Building, Expansion, and Conflict; Creation, Expansion, and Interaction of Economic
Systems
Learning Objective: SB–2; SB– 4; SB–9; ECON–2
Historical Thinking Skill: Causation
Key Concept: 3.1.I; 3.1.II

3.20. Stateless societies in Africa are most often characterized by __________.


(A) professional militaries
(B) little concentration of authority
(C) universally accepted religions
(D) impressive capital cities

64
Copyright © 2017, 2015, 2011, 2007 Pearson Education, Inc. All rights reserved.
Answer: B
Topic: Societies With and Without States
Theme: State Building, Expansion, and Conflict
Learning Objective: SB–4
Historical Thinking Skill: Comparison
Key Concept: 3.3.II

3.21. What was the first universalistic religion to penetrate Africa and take root in the east African
kingdoms of Nubia and Ethiopia?
(A) Buddhism
(B) Christianity
(C) Hinduism
(D) Islam

Answer: B
Topic: The Christian Kingdoms: Nubia and Ethiopia
Theme: Development and Interaction of Cultures
Learning Objective: CUL–2; CUL–3
Historical Thinking Skill: Contextualization
Key Concept: 3.1.III

3.22. Architects of West African mosques usually used which of the following abundant building
materials?
(A) beaten clay, which allowed for diversity in appearance
(B) concrete, which allowed for strength
(C) gold, which allowed for displays of opulence
(D) molded sand, which allowed for rapid construction

Answer: A
Topic: The Architecture of Faith
Theme: Development and Interaction of Cultures
Learning Objective: CUL–8
Historical Thinking Skill: Comparison
Key Concept: 3.1.III

65
Copyright © 2017, 2015, 2011, 2007 Pearson Education, Inc. All rights reserved.
Questions 3.23–3.25 refer to the passage below.

When one uses a bronze mirror, one can adjust the clothes and cap; when one uses the ancients as a
mirror, one comprehends the rise and fall [of a state], and when one uses a person as a mirror, one sees
one’s success and missteps. I always keep these three mirrors to prevent making mistakes.

Emperor Tang Taizong, Eighth Century C.E.

3.23. The passage best supports which of the following conclusions?


(A) The emperor understood the complex nature of China’s dynastic cycle.
(B) The emperor was interested in moving China towards democratic rule.
(C) The emperor was determined to reconstitute Chinese architecture.
(D) The emperor was concerned with the well-being of Chinese artisans.

Answer: A
Topic: The Emergence of the Tang and the Restoration of the Empire
Theme: State Building, Expansion, and Conflict
Learning Objective: SB–2
Historical Thinking Skill: Interpretation
Key Concept: 3.2.1

3.24. In the Confucian-styled reconstituted Tang dynasty, which of the following groups gained the most
prestige?
(A) landed aristocrats
(B) artisans
(C) merchants
(D) scholar-gentry elites

Answer: D
Topic: Rebuilding the World’s Largest and Most Pervasive Bureaucracy
Theme: State Building, Expansion, and Conflict
Learning Objective: SB–2
Historical Thinking Skill: Patterns of Continuity and Change over Time
Key Concept: 3.2.I

3.25. Why could it be argued that Empress Wu brought social upheaval to traditional dynastic rule in
China?
(A) The empress relied on foreign advisors rather than on the scholar-gentry.
(B) The empress instituted equal rights between men and women with regard to divorce.
(C) The empress favored Buddhism and attempted to make it the state religion.
(D) The empress disposed of the long-standing Chinese tradition of conscription for state
projects.

Answer: C
Topic: State and Religion in the Tang and Song Eras
Theme: Development and Transformation of Social Structures; State Building, Expansion, and Conflict
Learning Objective: SOC–5; S–B2

66
Copyright © 2017, 2015, 2011, 2007 Pearson Education, Inc. All rights reserved.
Historical Thinking Skill: Patterns of Continuity and Change over Time
Key Concept: 3.3.III

67
Copyright © 2017, 2015, 2011, 2007 Pearson Education, Inc. All rights reserved.
Questions 3.26–3.28 refer to the passage and image below.

SOURCE: Lynda Shaffer, “Southernization” Journal of World History 5, no. 1 (Honolulu, HI: University of Hawaii Press, 1994).

In the early part of the Song dynasty . . . a new variety of early-ripening rice was introduced into China
from Champa, a kingdom then located near the Mekong River Delta in what is now Vietnam, and by
1012 it had been introduced in the lower Yangzi and Huai river regions. . . . Because the variety of rice
was relatively more drought-resistant, it could be grown in places where older varieties had failed,
especially on higher land and on terraces that climb hilly slopes, and it ripened even faster than the
other early-ripening varieties already grown in China. This made double-cropping possible in some
areas, and in some places, even triple-cropping became possible . . . the hardiness and productivity of
various varieties of rice were and are in large part responsible for the density of population in South,
Southeast, and East Asia.

According to the Buddhist monk Shu Wenying, the Song Emperor Zhengzhong (998–1022), when he
learned that Champa rice was drought-resistant, sent special envoys to bring samples back to China.

Lynda Shaffer (Tufts University Historian), “Southernization,” 1994

New Farming Techniques Developed as the Importance of Rice Increased in


China During the Song Dynasty

68
Copyright © 2017, 2015, 2011, 2007 Pearson Education, Inc. All rights reserved.
3.26. Why was the discovery mentioned in the passage important to China?
(A) China was interested in discovering a new commodity to trade with other regions.
(B) China needed to support one of the largest populations in the world.
(C) China discovered that rice could be used as a fertilizer for millet and other crops.
(D) China could increase its tribute to other regions if it produced more rice.

Answer: B
Topic: Expanding Agrarian Production and Life in the Country
Theme: Creation, Expansion, and Interaction of Economic Systems
Learning Objective: ECON–1; ECON–3
Historical Thinking Skill: Interpretation
Key Concept: 3.3.I; 3.3.II; 3.3.III

3.27. Which of the following is not a reason why Emperor Yangdi constructed the Grand Canal?
(A) Overland routes were slow and difficult.
(B) Taxes could be more efficiently collected.
(C) Tolls could be used to pay off foreign investors.
(D) Military forces could be more easily moved to suppress rebellions.

Answer: C
Topic: Tang and Song Prosperity: The Basis of a Golden Age
Theme: Interaction Between Humans and the Environment; State Building, Expansion, and Conflict;
Creation, Expansion, and Interaction of Economic Systems
Learning Objective: ENV–6; SB–1; ECON–3
Historical Thinking Skill: Causation
Key Concept: 3.1.I; 3.1.II; 3.3.I; 3.3.II; 3.3.III

3.28. As the Tang reopened and protected trade routes to Persia, which luxury commodities were
exchanged in high volumes between these two regions?
(A) horses to Persia and tin to China
(B) silk to Persia and fine tapestries to China
(C) salt to Persia and tea to China
(D) gold to Persia and silver to China

Answer: B
Topic: A New Phase of Intercontinental Commercial Expansion by Land and Sea
Theme: Interaction Between Humans and the Environment
Learning Objective: ENV–3
Historical Thinking Skill: Patterns of Continuity and Change over Time
Key Concept: 3.1.1; 3.1.2

69
Copyright © 2017, 2015, 2011, 2007 Pearson Education, Inc. All rights reserved.
Questions 3.29–3.31 refer to the map below.

Transcontinental Empire of Chinggis Khan

3.29. Which military technologies did the Mongols most often use to create the largest and most
diverse preindustrial empire in world history?
(A) blowguns and spears
(B) chariots and lances
(C) slings and war hammers
(D) short bows and stirrups

Answer: D
Topic: Building the Mongol War Machine
Theme: State Building, Expansion, and Conflict
Learning Objective: SB–6
Historical Thinking Skill: Contextualization
Key Concept: 3.1.I

3.30. Which of the following statements best describes the Mongol’s attitude towards other societies
and religions?
(A) Only the indigenous Mongol religion was allowed to be practiced throughout the empire.
(B) Technology from other pastoralist groups was welcomed but not from sedentary people.
(C) The Mongols were generous patrons of other cultures and attracted many foreigners to
their territories.
(D) The Mongols resisted interregional trade because foreign cultures would dilute their
civilization.

Answer: C

70
Copyright © 2017, 2015, 2011, 2007 Pearson Education, Inc. All rights reserved.
Topic: Mongol Tolerance and Foreign Cultural Influence
Theme: Development and Interaction of Cultures; State Building, Expansion, and Conflict
Learning Objective: CUL–4; SB–6
Historical Thinking Skill: Contextualization
Key Concept: 3.1.III; 3.2.I; 3.2.II

3.31. Which statement best describes how Kubilai Khan instituted Mongol rule in China during the Yuan
dynasty?
(A) The Mongol nobility were intermarried with the Chinese aristocracy to legitimize Mongol
rule.
(B) The Chinese were integrated into the military, thus creating an even more powerful Mongol
army.
(C) the Mongols retained centralized power, and the Chinese were used to govern regional and
local areas.
(D) The existing Chinese bureaucracy was taught the Mongol language in order to exploit the
usefulness of the scholar-gentry class.

Answer: C
Topic: The Mongol Interlude in Chinese History
Theme: State Building, Expansion, and Conflict
Learning Objective: SB–4
Historical Thinking Skill: Patterns of Continuity and Change over Time
Key Concept: 3.2.I; 3.2.II

71
Copyright © 2017, 2015, 2011, 2007 Pearson Education, Inc. All rights reserved.
Questions 3.32–3.34 refer to the passage below.

One woman will drive twenty or thirty wagons, since the terrain is level. . . . It is the women’s task to
drive the wagons, to load the dwellings on them and to unload again, to milk the cows, to make butter
and grut [curds or cheese], and to dress the skins and stitch them together, which they do with a thread
made from sinew.

The men make bows and arrows, manufacture stirrups and bits, fashion saddles, construct the dwellings
and the wagons, tend the horses and mares, churn the comas [that is, the mare’s milk], produce the
skins in which it is stored, and tend and load the camels.

William of Rubruck (A Franciscan Friar Who Visited the Mongols in the 1250s
on Behalf of the King of France), A Report on Gender Relations,

3.32. How did women of the Mongol empire differ from their contemporaries in the Song dynasty?
(A) Mongol women often experienced arranged marriages. Song women often chose their own
spouses.
(B) Mongol women could not fight in combat operations. Song women were often combatants.
(C) Mongol women had rights to property and education. Song women did not have these
opportunities.
(D) Mongol women were often confined to duties within the household. Song women were
expected to be more active in society.

Answer: C
Topic: Gender Roles and the Convergence of Mongol and Chinese Culture
Theme: Development and Transformation of Social Structures
Learning Objective: SOC–1
Historical Thinking Skill: Comparison
Key Concept: 3.2.I; 3.3.III

3.33. What happened to peasants after the Golden Horde conquered Russia?
(A) Peasants were forced to emigrate to western Europe to accommodate Mongol settlers.
(B) Peasants had to pay tribute not only to the Russian nobility but also to their Mongol
overlords.
(C) Peasants were liberated from the hold of Russian nobles and were assimilated into the
Mongol civilization.
(D) Peasants had to assume the responsibilities of landowners, because the Mongols
exterminated the Russian nobility.

Answer: B
Topic: Russia in Bondage
Theme: State Building, Expansion, and Conflict
Learning Objective: SB–6; SB–8
Historical Thinking Skill: Causation
Key Concept: 3.1.I; 3.2.II

3.34. Mongol invasions in the thirteenth and fourteenth centuries destroyed which of the following?

72
Copyright © 2017, 2015, 2011, 2007 Pearson Education, Inc. All rights reserved.
(A) the Abbasid capital of Baghdad
(B) the Mamluks of Egypt
(C) Tammerlane’s Turkic forces
(D) the Abbasid capital of Baghdad and the Mamluks of Egypt

Answer: A
Topic: New Waves of Nomadic Invasions and the End of the Caliphate
Theme: Interaction Between Humans and the Environment; State Building, Expansion, and Conflict
Learning Objective: ENV–3, SB–6
Historical Thinking Skill: Contextualization
Key Concept: 3.3.II; 3.1.I

73
Copyright © 2017, 2015, 2011, 2007 Pearson Education, Inc. All rights reserved.
Questions 3.35–3.37 refer to the image below.

Arab Scientists Testing and Working with a Wide Variety of


Instruments

3.35. Which of the following became the official language of the Abbasid court for religion, law, and
science?
(A) Persian
(B) Turkish
(C) Latin
(D) Arabic

Answer: A
Topic: An Age of Learning and Artistic Refinement
Theme: Development and Interaction of Cultures
Learning Objective: CUL–7
Historical Thinking Skill: Patterns of Continuity and Change over Time
Key Concept: 3.1.III

3.36. How can medicine in the Abbasid empire best be described?


(A) as traditional remedies passed across generations through matrilineal lines
(B) as reliant on shamans, who became adept at curing many diseases and ailments

74
Copyright © 2017, 2015, 2011, 2007 Pearson Education, Inc. All rights reserved.
(C) as generally underdeveloped, which caused many unnecessary deaths among the growing
population
(D) as renowned for an effective and regimented training system that created skilled physicians

Answer: D
Topic: Achievements in Science
Theme: Development and Interaction of Cultures
Learning Objective: CUL–7
Historical Thinking Skill: Patterns of Continuity and Change over Time
Key Concept: 3.1.III

3.37. After the Muslim incursion into South Asia during the postclassical period, which of the following
best describes the compatibility of Hindu and Islamic cultures?
(A) compatible, owing to similar polytheistic belief systems
(B) compatible, owing to similar monotheistic belief systems
(C) incompatible, owing to differences in belief systems based on strict oral traditions versus
sacred texts
(D) incompatible, owing to different beliefs in egalitarianism versus a strict hierarchy

Answer: D
Topic: Patterns of Accommodation
Theme: Development and Interaction of Cultures; State Building, Expansion, and Conflict
Learning Objective: CUL–4; CUL–5; SB–4
Historical Thinking Skill: Contextualization
Key Concept: 3.3.III

75
Copyright © 2017, 2015, 2011, 2007 Pearson Education, Inc. All rights reserved.
Questions 3.38–3.40 refer to the map below.

The Voyages of Zheng He

3.38. Why did transregional maritime trade and exploration expand during the fifteenth century?
(A) Russia began exporting large volumes of timber, which aided shipbuilding.
(B) Overland travel became dangerous without Mongol protection.
(C) China found that other civilization had better manufacturing capabilities than its own.
(D) African traders became leaders in ocean-going trade because of their central location.

Answer: B
Topic: The Structure of Transregional Trade
Theme: Creation, Expansion, and Interaction of Economic Systems
Learning Objective: ECON–12
Historical Thinking Skill: Contextualization
Key Concept: 3.3.I; 3.3.II

3.39. Which of the following statements best describes why the Ming dynasty abruptly discontinued the
transregional trade referenced in the map?
(A) China could not compete with the existing trade network in the Indian Ocean.
(B) China found that its goods were not valued by merchants in other lands.
(C) China decided that overland trade routes were more efficient and cost effective than
oceanic routes.
(D) China decided to return to traditional expenditures rather than distant foreign trade.

76
Copyright © 2017, 2015, 2011, 2007 Pearson Education, Inc. All rights reserved.
Answer: D
Topic: Chinese Outreach and Reconsideration
Theme: Creation, Expansion, and Interaction of Economic Systems
Learning Objective: ECON–12
Historical Thinking Skill: Contextualization
Key Concept: 3.2.I

3.40. Which of the following was not a reason why western Europe began a fifteenth-century expansion
of oceanic transregional trade and exploration?
(A) The Mongol empire provided new access to Asian knowledge and technology.
(B) Western Europeans benefited from superior shipbuilding techniques.
(C) Western Europeans needed alternate trade routes because of a growing Muslim stronghold
on overland routes.
(D) Western European elites desired Asian luxury products and wanted to create a more reliable
supply line.

Answer: B
Topic: Imitation and Commercial Problems
Theme: Creation, Expansion, and Interaction of Economic Systems
Learning Objective: ECON–12
Historical Thinking Skill: Contextualization
Key Concept: 3.3.I; 3.3.II

77
Copyright © 2017, 2015, 2011, 2007 Pearson Education, Inc. All rights reserved.
Questions 3.41–3.43 refer to the image below.

Painting of Dante, by Domenico di Michelina, Which Hangs in the Cathedral of Florence, 1465

3.41. How can the Renaissance movement best be described?


(A) as a revival of styles and themes from classical Greece and Rome
(B) as a religious movement that emphasized a return to the purity of Catholic teaching
(C) as an Italian movement that strengthened the practice of feudalism and manorialism
(D) as a glorification of the militaristic nature associated with Italian principalities

Answer: A
Topic: Secular Directions in the Italian Renaissance
Theme: Development and Interaction of Cultures
Learning Objective: CUL–3
Historical Thinking Skill: Patterns of Change and Continuity over Time
Key Concept: 3.1.III

3.42. How did fifteenth-century Spanish rulers assert their authority on the Iberian Peninsula?
(A) by crushing their Portuguese counterparts and forcing them to pay tribute
(B) by rivaling the religious power and authority of Rome by creating their own papacy
(C) by promoting Christianity and forcing the conversion or expulsion of other religions
(D) by tolerating Jewish and Muslim residents in return for higher taxation that benefited the
crown

Answer: C

78
Copyright © 2017, 2015, 2011, 2007 Pearson Education, Inc. All rights reserved.
Topic: The Iberian Spirit of Religious Mission
Theme: Development and Interaction of Cultures
Learning Objective: CUL–4
Historical Thinking Skill: Patterns of Change and Continuity over Time
Key Concept: 3.2.I

3.43. The expanding network of fifteenth-century Polynesian culture __________.


(A) was known for its highly stylized writing system and its literature
(B) was welcomed into existing world trade networks because of its unique goods and wares
(C) was known for creating an expansive and peaceful society that encompassed thousands of
islands
(D) was isolated from developments in other societies elsewhere in the world

Answer: D
Topic: Expansion, Migration, and Conquest in Polynesia
Theme: Development and Interaction of Cultures
Learning Objective: CUL–6
Historical Thinking Skill: Patterns of Continuity and Change over Time; Contextualization
Key Concept: 3.1.II

79
Copyright © 2017, 2015, 2011, 2007 Pearson Education, Inc. All rights reserved.
Questions 3.44–3.46 refer to the image below.

The Golden Pavilion, One of the Great Architectural Treasures


of Japan

3.44. The Golden Pavilion best represents the influence of which religions?
(A) Confucianism and Buddhism
(B) Buddhism and Christianity
(C) Hinduism and Confucianism
(D) Shintoism and Buddhism

Answer: D
Topic: Artistic Solace for a Troubled Age
Theme: Development and Interaction of Cultures
Learning Objective: CUL–8; CUL–9
Historical Thinking Skill: Contextualization
Key Concept: 3.1.III

3.45. Which of the following statements relates a key difference between Korea and China?
(A) Ethnically, Koreans were descended from the peoples of Siberia and Manchuria.
(B) Politically, Koreans were able to resist being conquered by any Chinese dynasties.
(C) Agriculturally, Koreans were not dependent on rice as a staple of their diets.
(D) Religiously, Koreans practiced Shintoism as opposed to adhering to traditional Chinese belief
systems.

Answer: A
Topic: Korea: Between China and Japan

80
Copyright © 2017, 2015, 2011, 2007 Pearson Education, Inc. All rights reserved.
Theme: Interaction Between Humans and the Environment
Learning Objective: ENV–4
Historical Thinking Skill: Comparison
Key Concept: 3.1.II; 3.1.IV

3.46. Of the following manifestations of Vietnamese culture, which remained more distinct from
Chinese culture than the others?
(A) art and literature
(B) politics
(C) agriculture
(D) religion

Answer: A
Topic: Between China and Southeast Asia: The Making of Vietnam
Theme: Development and Interaction of Cultures
Learning Objective: CUL–7
Historical Thinking Skill: Comparison
Key Concept: 3.1.III

81
Copyright © 2017, 2015, 2011, 2007 Pearson Education, Inc. All rights reserved.
Questions 3.47–3.50, on the feudal systems that existed in both Japan and western Europe, refer to
the images below.

Night Attack on the Sanjo Palace (from the Scrolls of the Events of the Heiji Era)

82
Copyright © 2017, 2015, 2011, 2007 Pearson Education, Inc. All rights reserved.
The Knight and the Landsknecht, by Albrecht Dürer, c. 1497

3.47. Name a similarity between the feudal systems described in the images.
(A) Both systems were based on negotiated contracts.
(B) Both systems included provisions for ritualized suicide.
(C) Both systems allowed peasants to join the warrior nobility.
(D) Both systems had ethical codes of behavior.

Answer: D
Topic: Comparing Feudalisms
Theme: Development and Transformation of Social Structures
Learning Objective: SOC–2
Historical Thinking Skill: Comparison
Key Concept: 3.3.III

3.48. Which of the following statements regarding gender roles in medieval Japan is true?
(A) Women were given in marriage to cement alliances between warrior households.
(B) Women fulfilled both the role of men and women in theatrical productions.
(C) Women benefitted from primogeniture, which granted them rights of inheritance.
(D) Women retained roles as primary celebrants in religious ceremonies.

83
Copyright © 2017, 2015, 2011, 2007 Pearson Education, Inc. All rights reserved.
Answer: A
Topic: Toward Barbarism? Military Division and Social Change
Theme: Development and Interaction of Cultures
Learning Objective: CUL–4
Historical Thinking Skill: Patterns of Continuity and Change over Time
Key Concept: 3.3.III

3.49. The early parliaments of postclassical western Europe attempted to institutionalize which of the
following principles?
(A) the principle that the church, nobles, and peasants should have rights
(B) the principle that monarchs should consult with their vassals
(C) the principle that noble families should pay lower taxes than the peasantry
(D) the principle that power should be vested in centralized monarchs

Answer: B
Topic: Feudal Monarchies and Political Advances
Theme: State Building, Expansion, and Conflict
Learning Objective: SB–2
Historical Thinking Skill: Patterns of Continuity and Change over Time
Key Concept: 3.2.I

3.50. What was the rationale behind the three-field system?


(A) to increase the fertility of the land
(B) to increase variety in crops
(C) to decrease insect infestations
(D) to decrease irrigation shortages

Answer: A
Topic: The Manorial System: Obligations and Allegiances
Theme: Interaction Between Humans and the Environment
Learning Objective: ENV–8
Historical Thinking Skill: Patterns of Continuity and Change over Time
Key Concept: 3.3.1

84
Copyright © 2017, 2015, 2011, 2007 Pearson Education, Inc. All rights reserved.
3.51. Why was the practice of footbinding largely restricted to the Chinese elite?

X-Ray of Bound Feet, China, Between 1890 and 1923

(A) Upper-class women were not required to perform labor-intensive duties.


(B) Upper-class women were more likely to be practitioners of religions that encouraged
footbinding.
(C) Upper-class women were more likely to escape from their husbands and were, as a
consequence, restrained.
(D) Upper-class women were interested in fashion trends that would distinguish them from
peasants.

Answer: A
Topic: Footbinding as a Marker of Male Dominance
Theme: Development and Interaction of Cultures
Learning Objective: CUL–4; CUL–5
Historical Thinking Skill: Patterns of Continuity and Change over Time
Key Concept: 3.2.I; 3.3.III

3.52. What was the occupation of Khadijah, Muhammad’s first wife?


(A) She was a priestess of Mecca’s pagan religion.
(B) She was a produce merchant at an oasis outside of Medina.
(C) She was the head of a thriving trading business and Muhammad’s boss.
(D) She was an attendant in the service of Muhammad’s father.

Answer: C
Topic: Civilization and Gender Relationships
Theme: Development and Interaction of Cultures

85
Copyright © 2017, 2015, 2011, 2007 Pearson Education, Inc. All rights reserved.
Learning Objective: CUL–4; CUL–5
Historical Thinking Skill: Patterns of Continuity and Change over Time
Key Concept: 3.2.I, 3.3.III

3.53. Which of the following reasons led to older Aztec women being responsible for the raising of
young Aztec females, as opposed to their own mothers?
(A) Women were employed as spies to gather reconnaissance on other tribes and were gone for
extended periods of time.
(B) Women were involved in military operations equally with men, and many perished in battle.
(C) Women served many polytheistic gods as priestesses and had little time for their own
families.
(D) Women were required to grind corn most of the day because Aztecs lacked water- and
animal-powered mills.

Answer: D
Topic: Overcoming Technological Constraints
Theme: Creation, Expansion, and Interaction of Economic Systems
Learning Objective: ECON–5
Historical Thinking Skill: Contextualization
Key Concept: 3.3.III

86
Copyright © 2017, 2015, 2011, 2007 Pearson Education, Inc. All rights reserved.
Section II
Short-Answer Questions
3.54. Use the passage and maps below and your knowledge of world history to answer all parts of the
question that follows.

The city has numerous shops of artisans and merchants. Weavers of cotton cloth are particularly
well-represented. Slave girls sell all the foodstuffs and fresh bread. The men wear fine black or
blue cotton fabrics, some of which are imported from Europe by merchants from Berber
territory. All the women here, except for the slaves, veil their faces.

The inhabitants of the city are quite wealthy, especially the foreigners among them. This explains
why the present king has even given two of his daughters to two particularly wealthy brothers
who are merchants. There are many wells with fresh, pure water. When the Niger floods, canals
deliver water to the city. There is a great abundance of grain and livestock. The inhabitants
consume much milk and butter, and there is much fruit. On the other hand, salt is in very short
supply. It has to be brought in from . . .

The people of Timbuktu use unrefined gold nuggets and gold coins as their standard currency,
although for small purchases, they use cowries from Iran.

Excerpt of a Description of Timbuktu by Leo Africanus

87
Copyright © 2017, 2015, 2011, 2007 Pearson Education, Inc. All rights reserved.
Leading Trade Routes of Europe

88
Copyright © 2017, 2015, 2011, 2007 Pearson Education, Inc. All rights reserved.
The Voyages of Zheng He

(A) Briefly describe a beneficial technological response that facilitated trade by one postclassical
urban area depicted in any of the stimuli above.
(B) Briefly describe the commodities exchanged between two relevant postclassical urban
centers.
(C) Briefly describe two societal changes that occurred in a region due to the rise of
urbanization in the postclassical period.

Sample Answers

(A) Students might include the following technological responses that facilitated trade:
The Arab zone invented dhows, lateen sails, the sextant, and the astrolabe.
Africa invented the camel saddle.
China created the Grand Canal, cotton sails, the magnetic compass, junks, treasure ships,
sternpost rudders, and flying money.
Western Europe invented the caravel.
Central Asia created passports, a postal service, and inns and stables for use on the Silk
Road.

(B) Students might include the following areas and commodities:


The interior of Africa supplied ivory, slaves, exotic animals, salt, and gold.
The Swahili Coast supplied bananas, fish, and cattle.
China supplied paper, gunpowder, silk, porcelain, and jade.

89
Copyright © 2017, 2015, 2011, 2007 Pearson Education, Inc. All rights reserved.
Western Europe supplied timber, copper, honey, wool, and tin.
The Arab zone supplied pottery, textiles, and glass.
Southeast Asia supplied spices and forest products.
India supplied cotton, spices, and textiles.

(C) Students might include the following societal changes:


State involvement in economic activity (e.g., Ming China, Italian principalities, Islamic
dynasties, Mali)
New patterns of labor (e.g., corvée and coerced labor, free peasant agriculture, guild
organizations, military obligations)
Trade between urban areas that led to extensive cultural diffusion (e.g., religions, art and
architecture, scientific ideas)
Change in gender roles

Topic: Civilization and Gender Relationships; Town and Country: Commercial Boom and Agrarian
Expansion; The First Flowering of Islamic Learning; Early Islam and the World; An Age of Learning and
Artistic Refinements; The Spread of Islam to Southeast Asia; Kingdoms of the Grasslands; The Swahili
Coast of East Africa; Civilization in Eastern Europe; The Byzantine Empire; The Emergence of Kievan Rus’
New Economic and Urban Vigor; Changing Economic and Social Forms in the Postclassical Centuries;
Tang and Song Prosperity: The Basis of a Golden Age; Long Distance Trade and Cross‐Cultural Exchange:
Life under the Mongol Imperium; The Mongol Empire as a Bridge Between Civilizations; Key Changes in
the Middle East; The Structure of Transregional Trade; The Rise of the West
Theme: State Building, Expansion, and Conflict; Creation, Expansion, and Interaction of Economic
Systems; Development and Transformation of Social Structures
Learning Objective: SB–1; SB–2; SB–4; SB–5; SB–6; ECON–1; ECON–2; ECON–5; ECON–12; SOC–1; SOC–
2; SOC–3
Historical Thinking Skill: Causation; Periodization; Contextualization; Interpretation; Patterns of Change
and Continuity over Time; Analyzing Evidence: Content and Sourcing
Key Concept: 3.1.I; 3.1.IV; 3.2.II; 3.3.I; 3.3.II; 3.3.III

90
Copyright © 2017, 2015, 2011, 2007 Pearson Education, Inc. All rights reserved.
3.55. Use the map and image below and your knowledge of world history to answer all parts of the
question that follows.

The Four Khanates of the Divided Mongol Empire

Persian Miniature of a Prince Submitting to


Chinggis Khan

(A) Identify one military strategy or technology that the Mongols employed to conquer foreign
lands.
(B) Identify and explain the system of administration that the Mongols utilized to rule one
civilization they subjugated.

91
Copyright © 2017, 2015, 2011, 2007 Pearson Education, Inc. All rights reserved.
(C) Identify a situation where the Mongol war machine did not achieve its military objectives.

Sample Answers

(A) Students might include the following technologies or strategies:


Siege weapons, cavalry, stirrups, short bows, tumens, encirclement, feigning retreat, terror,
reconnaissance, incendiaries, cannons, messengers

(B) Students might include the following Mongol administrative techniques:


In Russia (e.g., tribute system allowing Russian nobility to rule as long as they dressed like
Mongols and collected tribute/taxes)
In China (similar answers for China may be given)
In the Muslim heartland (similar answers for the Muslim heartland may be given)

(C) Students might include these examples of Mongol failures to achieve military objectives:
Mongol defeat at the hands of Mameluke and Crusader armies in route to Egypt
Mongol military failures due to the heat and jungles of Vietnam
Mongol failure to achieve military objectives in western Europe, India, or Japan

Topic: The Transcontinental Empire of Chinggis Khan; The Mongol Drive to the West; The Mongol
Interlude in Chinese History
Theme: State Building, Expansion, and Conflict
Learning Objective: SB–2; SB–4, SB–6; SB–10
Historical Thinking Skill: Patterns of Continuity and Change over Time; Contextualization; Analyzing
Evidence: Content and Sourcing
Key Concept: 3.2.I; 3.2.II; 3.3.III

92
Copyright © 2017, 2015, 2011, 2007 Pearson Education, Inc. All rights reserved.
Section III
Long Essay Questions
Question 3.56: Compare China’s political and cultural influence on two of the following regions between
c. 600 C.E. and c. 1450 C.E.

Japan—————Korea—————Vietnam

Sample Answer

This question is asking students to compare China’s political and cultural influence on Japan and Korea,
on Japan and Vietnam, or on Korea and Vietnam and to support their thesis. Students should
understand that compare also means contrast and that they are required to provide evidence and
analysis for both, but not necessarily evenly. To gain the thesis point the student must address all parts
of the question by qualifying comparisons and contrasts within the given timeframe. Qualifying
categories for comparison and contrast must associate political and cultural evidence related to China’s
influence on two of the regions. A strong essay will not only provide evidence related to politics and
culture but also analyze the impact of this evidence on these areas.

Points gained through the use of evidence shall be determined by the amount of evidence used. To gain
the maximum two points, the students will need to use at least five pieces of evidence. Examples of
evidence and analysis that students might utilize to gain points are described in the paragraphs that
follow.

Korea was conquered by China and was the most regularly influenced of the three areas largely due to
its proximity to China. When Korea was conquered by the Tang dynasty, the Silla kingdom became a
tribute state of China. This tributary relationship opened up a flow of culture and politics from China to
Korea. Korean scholars consulted with Confucian scholars, resulting in a transfer of knowledge though
Chinese writings. Chinese script was also adapted to the Korean language, and civil-service exams were
used to some extent to fill administrative positions, but birth was still the largest determining factor
when it came to political positions. Vietnam was also conquered by China’s Tang dynasty, but the
Vietnamese remained largely distinct from China, both culturally and politically. Sinification in Vietnam
was the least successful of the three areas largely because of topographical barriers and the fact that,
unlike Japan and Korea, Vietnam did not cooperate with its Chinese overlords. The Vietnamese people
revolted frequently. Chinese script was also incorporated into Vietnamese politics and culture. Vietnam
did utilize civil-service exams sparingly, but use didn’t translate to a strong Chinese-style bureaucracy
and the scholar-gentry never gained an important role. Japan, although never conquered by China, went
the furthest of the three areas in intentionally implementing Chinese politics and culture. Sinification in
Japan was most prominent among the imperial elite, who readily accepted China’s political influence.
Japan’s emperors incorporated both Confucian political and cultural aspects of China through the Taika
reforms, which included Chinese-style civil-service exams and Chinese script.

Buddhism was a key cultural influence that passed from China to all three areas. Japan blended
Buddhism with its state religion of Shintoism. Japanese Buddhists and aristocratic landlords joined
forces to repel Chinese influence. The Taika reforms eventually failed, leading to a decreased role in
Japan for a Confucian bureaucracy and to an increase in regional power, which led to feudalism. Like

93
Copyright © 2017, 2015, 2011, 2007 Pearson Education, Inc. All rights reserved.
Japan, the land-owning aristocracy in Korea remained too strong to truly centralize political power and
create a Chinese-style bureaucracy there. Buddhism was also welcomed in Vietnam due to the
prominent role of women, but Buddhism in Vietnam was also heavily influenced by Indian beliefs, which
weren’t as prominent in either Japan or Korea. Vietnamese women refused the patriarchal Confucian
system. All classes in Vietnam resisted Chinese fashion and preferred traditional Vietnamese styles of
clothing, which were deemed beneath the Chinese. The Vietnamese also continued to blacken teeth,
which was not a cultural element of China. Court life and culture in Japan mimicked the court etiquette,
architecture (pagodas/temples), art (nature/landscapes), and fashion of the Tang. Tang influence on
court life was acknowledged in Lady Murasaki’s Tale of Genji, one of the world’s first novels. Japanese
education also reflected strong elements of Chinese history and Confucian teachings. Korea also
adopted several cultural elements from China, such as a universal law code, fashion, court etiquette,
universities, art, architecture, and pottery. Korean pottery became better in quality and style than
Chinese pottery. Elements of Chinese cultural influence that did take hold in Vietnam were military
organization and irrigation, leading to increased population. Lastly, the Vietnamese rejected the Chinese
concept of extended family and continued to favor the nuclear family structure.

In this question, one point can be gained through the students’ use of the targeted historical thinking
skill of comparison. This can be accomplished by describing similarities and differences between two of
the regions specified in the question. An additional one point can also be secured if students use specific
examples of similarities and differences or analyze reasons for similarities and/or differences.

There are many ways that students can earn one point for synthesis. One way is for students to make
connections to earlier forms of Sinification in the Han dynasty during the classical period. Students may
also choose to discuss additional explicit qualifiers that figured into Sinification, such as economics,
geography, or the environment.

Topic: Spread of Chinese Civilization: Japan, Korea, and Vietnam


Theme: State Building, Expansion, and Conflict; Development and Interaction of Cultures
Learning Objective: SB–1; SB–2; SB–4; SB–7; SB–10; CUL–2; CUL–3; CUL–4; CUL–5; CUL–6; CUL–8
Historical Thinking Skill: Comparison
Key Concept: 3.1.III; 3.2.I; 3.3.II; 3.3.III

94
Copyright © 2017, 2015, 2011, 2007 Pearson Education, Inc. All rights reserved.
Question 3.57: Compare two of the following postclassical empires (c. 600 C.E to c. 1450 C.E.) in terms
of their economic practices and the participation of women in society.

Abbasid Empire—————Aztec Empire—————Byzantine Empire

Sample Answer

The question is asking students to compare women’s participation in society and the distinct economic
systems in the Abbasid and Aztec empires, the Abbasid and Byzantine empires, or the Aztec and
Byzantine empire and to support their thesis. Students should understand that compare also means
contrast and that they are required to provide evidence and analysis for both, but not necessarily
evenly. To gain the thesis point the student must address all parts of the question by qualifying
comparisons and contrasts within the given timeframe. Qualifying categories for comparison and
contrast must associate relevant evidence related to economic practices and women’s participation in
society in two of the provided empires. A strong essay will not only provide evidence related to
economies and gender roles but also analyze the impact of this evidence on these areas.

Points gained through the use of evidence shall be determined by the amount of evidence used. To gain
the maximum two points, students need to cite at least five pieces of evidence. Examples of evidence
and analysis that students might utilize to gain points are described in the paragraphs that follow.

The Byzantine and Abbasid empires were highly bureaucratic states whose capitals of Constantinople
and Baghdad were well-positioned major hubs of trade in the Afro-Eurasian network. The Aztecs did not
benefit from the well-established trade routes of this network, but certainly conducted profitable long-
distance trade through their merchant class, the pochteca. This segment of society specialized in long-
distance trade of luxury items like cocoa and plumes of exotic birds. The Abbasid also had a strong
merchant class, which was vital to the movement of luxury goods over long distances throughout the
Afro-Eurasian trading network for consumption by the elite. The movement of goods also proceeded
overseas from the Mediterranean to the South China Sea in Arab dhows. Muslim merchants often joined
forces with Christian and Jewish merchants so that trade would not be interrupted by a particular
groups’ holy day of the week. Because of the strong economy of the Abbasid, capital was continually
reinvested in new commercial ventures that grew the Abbasid caliphate. Wealth was spent on the
building and operation of mosques, religious schools, baths, and rest stops for weary traders and
travelers. Barter was the most common form of trade, but coins were commonplace and used
throughout the Afro-Eurasian trade network. The wazir was the chief administrator of the Abbasid who
oversaw the empire’s economic practices, including the collection and redistribution of tribute from all
provinces. The Abbasids were fairly effective at collecting tribute from subject peoples, but the farther a
province was from Baghdad, the more difficult collection became. The Byzantines were also active
traders in the Mediterranean and the Red Sea as well as on the Indian Ocean. Like the other two
empires, barter was the most common means of trade, but coins were regularly minted and used by the
Byzantine. Surviving weights and balances lent themselves to trade negotiations and to the
standardization of postclassical trade in the empire. The Byzantine also profited from a solid tax base
associated with highly productive peasants in the eastern Mediterranean and Asiatic regions of the
empire. When these areas in the east and west were cut off by invaders on both fronts, the economy of
the empire dwindled. Agriculture and luxury items were key economic elements of all three areas. Aztec
economic practices centered on agriculture, with the main crop being maize, or corn. To increase
production yield to feed a growing population, the Aztecs used chinampas, which allowed peasants to
harvest up to four crops of corn each year, on a grand scale. Other notable agricultural commodities

95
Copyright © 2017, 2015, 2011, 2007 Pearson Education, Inc. All rights reserved.
were sweet potatoes, fruits, avocados, beans, squashes, chia, chilies, and herbs. Nobles’ lands were
worked by servants or slaves procured in regional wars. The Aztec also relied on tribute from conquered
populations to provide items like cotton cloth and other food sources. The Aztec government was
responsible for the distribution of food and goods among the population. Nobles received a much larger
portion of tribute than the peasantry. The state also directed the redistribution of conquered territories.
Each community ran seasonal markets, but all paled in comparison to the great market at Tlatelolco.
Inspectors and judges regulated goods and prices at these markets. Cacao beans and gold dust were
used as currency, although barter was the major means of trade. Abbasid artisans, who were highly
valued for their skills, contributed greatly to the economy and were incredibly adept at producing
furniture, glassware, jewelry, and tapestries. The most accomplished artisans joined guild-like
organizations. In most regions of the empire, peasants did not own the land they worked but occupied it
as tenants, sharecroppers, or migrant laborers required to give the majority of their crops to the
landowner. From this economic practice, a landed elite class—the ayan—emerged in the countryside.
The Byzantine traded spices, silk, incense, and other highly valued items treasured by the elite. The
empire became a key producer of silk, which reduced its dependence on China for that luxury item, after
it acquired knowledge of silkworms from Asia. Additionally, a new pattern of trade came to prominence
between the empire and Scandinavia in the north. Luxury goods flowed north, and furs and other crude
materials returned south. High levels of commerce were a constant source of wealth for the empire.

Slavery was a regular economic feature in each of these postclassical empires. Slaves were received as
tribute in the Aztec empire and were used to further agricultural production. The amount of tribute
demanded by the Aztecs depended on whether or not the conquered land acquiesced or resisted Aztec
authority and supremacy. Slavery had its economic place during Abbasid rule as well. Unskilled and
brutish labor often fell to slaves in the countryside and in mining areas, but slaves also served the caliphs
and other nobility as domestic servants. The growing wealth of the Abbasid led to great demand for
female slaves. Most of these female slaves performed domestic services and were valued for their
intelligence and beauty. Slave women often came from areas outside the empire and were often more
highly educated than freeborn wives. Slave women most often functioned as concubines and servants.
They could go to the market and did not have to wear the veils and robes that were required attire for
free women in public places. Similar to the Abbasid, slavery in the Byzantine Empire was most regularly
seen in the domestic area. The commonest means of acquiring slaves were as prisoners of war.

Aristocratic women in the Byzantine Empire are regarded as having had greater influence than women
in many other civilized postclassical societies. The most notable example of their status is that women
held the imperial throne in Byzantium. The empress Theodora and her sister Zoe actually ruled jointly
for a time as empresses. Theodora, in particular, was known for dealing harshly with unruly nobles.
Matrilineal lines were sometimes used to establish social standing, and children took their mother’s
family name. For the majority of women, however, patriarchal norms persisted. For example, women
and men who were not family ate or worshipped separately. Although Byzantine women possessed
favorable status in the home, after bearing children their status often climbed higher. Women were
married usually in their mid-teens, and arranged marriages were common among higher-ranking
members of society. Women of middle- and upper-class backgrounds could be expected to have some
education; a few had occupational avenues open to them in the empire becoming doctors, business
owners, artisans, and members of the clergy. Low-born women often labored in the fields or in family
workshops. The status of freeborn Abbasid women varied widely. Like Byzantine women, some high-
born women and highly prized concubines had influence on the caliph and on other advisors early in the
empire, but the freedom and influence of women were greatly curtailed as the empire progressed. The
veil became an emblem of increasing subjugation to men, and confinement to the home was

96
Copyright © 2017, 2015, 2011, 2007 Pearson Education, Inc. All rights reserved.
increasingly commonplace in Abbasid society. Freeborn women from the lower Abbasid classes farmed,
wove textiles, or raised animals to support their families; women from higher classes were allowed
almost no career outlets outside of the home. Women of the Abbasid were often married at puberty,
and the legal age for marriage was nine years old. While Aztec women didn’t achieve the social status of
their Byzantine counterparts, Aztec women were a key to the productivity of the empire. The Aztecs
were a highly militaristic society, and women, barred from soldiering, lacked access to that determiner
of wealth and prestige. Their exclusion was a large factor in women becoming less equal. Due to the
lack of milling technology in the Americas, most women of childbearing age spent a great deal of time
grinding maize and corn into a usable form. Grinding corn and maize was incredibly labor intensive and
required the assignment of rearing younger children to the older women in society. Aztec women were
renowned for expertise in the areas of weaving, crafting, and dying textiles. Women were also
responsible for raising domesticated animals (turkeys) that served as additional food sources. Most
women in Aztec society married in their late teens or early twenties and were expected to be virgins.
Women of noble birth were usually the subjects of arranged marriages for political and alliance
purposes. Polygamy did exist in the higher ranks of Aztec society, but peasants were monogamous. The
Aztecs were keenly aware of the dangers of childbirth to women. Because of this, midwifery was an
important occupation for Aztec women. Women that didn’t survive the birth of a child were given the
same ritual burial rites as soldiers killed in war. Cihuacoatl, a deity believed to support women who died
giving birth, was an important goddess to Aztec women.

In this question, one point can be gained through the students’ use of the targeted historical thinking
skill of comparison. This can be accomplished by describing similarities and differences between two of
the regions specified in the question. An additional one point can also be secured if students use specific
examples of similarities and differences or analyze reasons for similarities and/or differences.

There are many ways that students can earn one point for synthesis. One way is for students to make
connections to gender issues and to economic practice of other postclassical empires, such as the Incas
or any of the Chinese dynasties. Students may also choose to discuss additional explicit qualifiers that
figure into economics and gender roles, such as politics, society, or the environment.

Topic: From Arab to Islamic Empire: The Early Abbasid Era; The Islamic Heartlands in the Middle and
Late Abbasid Eras; Civilization in Eastern Europe; The Byzantine Empire; Feeding the People: The
Economy of the Empire; Aztec Society in Transition
Theme: Creation, Expansion, and Interaction of Economic Systems; Development and Transformation of
Social Structures
Learning Objective: ECON–1; ECON–2; ECON–3; ECON–4; ECON–10; ECON–11; ECON–12; SOC–1; SOC–
4; SOC–5
Historical Thinking Skill: Comparison
Key Concept: 3.1.I; 3.1.III; 3.3IV; 3.2.I; 3.2II; 3.3.I; 3.3.II; 3.3.III

97
Copyright © 2017, 2015, 2011, 2007 Pearson Education, Inc. All rights reserved.
Section IV
Document-Based Questions
Directions: The following question is based on the accompanying documents. The documents have been
edited for the purpose of this exercise.

In your response, you should do the following:

• State a relevant thesis that directly addresses all parts of the question.

• Support the thesis or a relevant argument with evidence from all, or all but one, of the
documents.

• Source and analyze the significance of at least four of the documents on the following basis:
intended audience, author’s purpose, historical context, format or medium, and/or point of view.

• Develop context by relating your argument to broader historical events or processes.

• Synthesize the elements above into a persuasive essay that extends your argument by connecting
it to:

• a different historical period or geographical region


• another course theme or historical approach that is not the focus of the essay (such as
political, social, economic, or intellectual history)
• another perspective from a different academic discipline (such as economics, art history,
anthropology, or government)

98
Copyright © 2017, 2015, 2011, 2007 Pearson Education, Inc. All rights reserved.
Question 3.58: The period known as the Renaissance witnessed a change in the perception of the nature
of man. Using the documents and your knowledge of world history, compare and contrast views held by the
Renaissance thinkers on this topic.

Historical Background: The Renaissance, a cultural and political movement grounded in urban
vitality and expanding commerce, began in Italy during the fourteenth century. The earlier phases of
the Renaissance involved literary and artistic themes more friendly to the secular world than the
previous religiously oriented outlook. Merchants sought out new markets. City‐state governments
evaluated techniques for efficient administration.

Document 1
Excerpt from Machiavelli, The Prince, 1531
Here the question arises: whether it is better to be loved than feared or feared than loved. The answer
is that it would be desirable to be both but, since that is difficult, it is much safer to be feared than to be
loved, if one must choose. For on men in general this observation may be made; they are ungrateful,
fickle, and deceitful, eager to avoid dangers and avid for gain and while you are useful to them they are
all with you, offering you their blood, their property, their lives, and their sons so long as danger is
remote, as we noted above, but when it approaches they turn on you. Any prince, trusting only in their
words and having no other preparations made, will fall to his ruin.

Document 2
Excerpt from Giovanni Pico (Count of Mirandola), The Oration on the Dignity of Man, 1486
As man is born, the Father has planted in him seeds of every sort, shoots of every life; those which each
man cultivates will grow, and bear their fruits in him. If these are vegetables, he will become a plant; if
sensual, a brute; if rational, a heavenly being; if intellectual, an angel and son of God. But if Man, not
contented with any creature’s lot, betakes himself into the center of his oneness, then, made one with
God, in the solitary darkness of the Father he who was created above all things will excel all things. Who
would not admire this chameleon of ours?

Document 3
Excerpt from Giorgio Varsari, Lives of the Painter, 1568t
The richest gifts are occasionally seen to be showered, as by celestial influence, upon certain human
beings; nay they sometimes supernaturally and marvelously gather in a single person—beauty, grace,
and talent united in such a manner that to whatever the man thus favored may turn himself, his every
action is so divine as to leave all other men far behind. . . . This was . . . the case of Leonardo de Vinci . . .
who had . . . so rare a gift of talent and ability that to whatever subject he turned his attention . . . he
presently made himself absolute master of it. . . .

He would without a doubt have made great progress in the learning and knowledge of the sciences had
he not been so versatile and changeful . . . the instability of his character led him to undertake many
things, which, having commenced, he afterwards abandoned.

99
Copyright © 2017, 2015, 2011, 2007 Pearson Education, Inc. All rights reserved.
Document 4
Excerpt from Peitro Aetino, Letter to Michelangelo, 1537
Just as it is disgraceful and sinful to be unmindful of God so it is reprehensible and dishonourable for any
man of discerning judgment not to honour you as a brilliant and venerable artist whom the very stars
use as a target at which to shoot the rival arrows of their favour. You are so accomplished, therefore,
that hidden in your hands lives the idea of a new king of creation, whereby the most challenging and
subtle problem of all in the art of painting, namely that of outlines, has been mastered by you that in
the contours of the human body you express and contain the purpose of art. . . . And it is surely my duty
to honour you with this salutation since the world has many kings but only one Michelangelo.

Document 5
Excerpt from Francesco Petrarch, Letter to Posterity, 1372
I have always possessed extreme contempt for wealth . . . I have on the contrary led a happier existence
with plain living and ordinary fare . . . the pleasure of dining with one’s friends is so great that nothing
has ever given me more delight than their unexpected arrival.

I possess a well-balanced rather than a keen intellect—one prone to all kinds of good and wholesome
study, but especially to moral philosophy and the art of poetry. The latter I neglected as time went on,
and took delight in sacred literature. . . . Among the many subjects that interested me, I dwelt especially
on antiquity, for our own age always repelled me, so that, had it not been for the love of those dear to
me, I should have preferred to have been born in any other period than our own. In order to forget my
own time, I have constantly striven to place myself in spirit in other ages, and consequently I delighted
in history.

Document 6
Excerpt from Nicholas Copernicus, Dedication to The Revolutions of the Heavenly Bodies, 1543
I am not so much in love with my conclusions as not to weigh what others will think about them, and
although I know that the meditations of a philosopher are far removed from the judgment of the laity,
because his endeavor is to seek out the truth in all things, so far as this is permitted by God to the
human reason, I still believe that one must avoid theories altogether foreign to orthodoxy.

100
Copyright © 2017, 2015, 2011, 2007 Pearson Education, Inc. All rights reserved.
Document 7
Painting of Dante, by Domenico di Michelina, Which Hangs in the Cathedral of Florence, 1465

Dante, Italian writer of the fourteenth century, holds a copy of his great work, The Divine Comedy, which is filled with both
religious and Renaissance symbolism.

Sample Answer

An acceptable response would utilize six or seven documents (all or all but one of the documents
provided) to deliver a comparative analysis of the change in perception of the nature of man during the
Renaissance. The thesis, which should consist of one or more congruent sentences in the opening
paragraph, should present similarities and differences in the perception of the nature of man given the
documents provided. An example of a good thesis might assert that the perception of man became
more grounded in humanism and individualism during the Renaissance, although religion was still a
looming and ever‐present theme throughout this period of rebirth.

Analysis of the documents should present evidence to support the thesis. Students might organize the
documents in a variety of ways around the themes of humanism, individualism, rationalism, religion,
politics, and/or what it means to be a “Renaissance Man.” To achieve all four points in the evidence
category of the rubric, students will need to relate at least six of the documents to the thesis and
analyze them by addressing historical context, audience, purpose, and/or point of view in at least four
documents.For example, in Document 1, students might focus on a prince’s need to be feared since the

101
Copyright © 2017, 2015, 2011, 2007 Pearson Education, Inc. All rights reserved.
Machiavelli’s perception of men is that they are inherently evil and self‐serving. A strong humanistic
tone is taken by the author of Document 2, where man is described with the ability to mold himself like
a “chameleon” into anything. The tone of Document 3 continues the theme of man’s talents, by focusing
on the virtuosic Leonardo de Vinci and his ability to master whatever subject he turned his mind to.
However, Vasari notes, that with more focus on the single subject of science, de Vinci could have made
greater contributions. Aetino, in Document 4, recognizes the greatness of Michelangelo’s individual
talent and would find it disgraceful and sinful for any man not to pay him homage. An unbridled pursuit
of all studies and moderation is demonstrated by Petrarch in Document 5. He shows reverence to other
ages and the study of antiquity, which was a common theme during the Renaissance. In Document 6,
Copernicus demonstrates the tone of rationalism, noting that man should avoid theories and focus on
what can be proved. Lastly, in Document 7, the painting symbolizes the secular and religious themes
that were ever‐present in the works of Renaissance thinkers.

Strong essays will not simply list descriptions or analysis of the documents but rather make connections
between the documents in order to provide arguments that support the thesis statement. For example,
connections that are consistent with the sample thesis provided above are (1) a comparison of how the
presence of religion is presented in Documents 2, 3, 6, and 7; (2) a discussion of the greatness of
Michelangelo’s more focused achievements (Document 4) in contrast to the varied approach that
Renaissance thinkers like de Vinici (Document 3) and Petrarch (Document 5) held to.

When writing the response to this document-based question, it is necessary for students to provide
outside examples and knowledge to support their thesis. An example of outside knowledge that isn’t
discussed in the documents provided is that artists and writers believed that men deserved attention
and recognition for their talents rather than for being directed by a divine plan, as was common during
medieval times. Practical codes of behavior that were a focus of Renaissance perceptions might also be
addressed. The contextualization point could be earned is by commenting on how feudalism did not gain
ground during this period or how the fall of the Mongols required Renaissance merchants to seek sea‐
based trading routes to acquire goods from other regions.

Common examples that may be used to earn the synthesis point are to discuss the Catholic Church’s
reluctance to support scientific claims that did not coincide with its teachings. Mention of Galileo
Galilei’s prosecution by the Inquisition in 1630 would be appropriate. Another way that students might
gain the synthesis point is to introduce the discipline of economics and the growing power of merchants
during the Renaissance to support the humanistic and individualistic perceptions of man.

Topic: Secular Directions in the Italian Renaissance; Renaissance Culture


Theme: Development and Interaction of Cultures; State Building, Expansion, and Conflict
Learning Objective: CUL–2; CUL–3; CUL–4; CUL–7; SB–2; SB–4
Historical Thinking Skill: Patterns of Continuity and Change over Time; Comparison; Contextualization;
Argumentation; Analyzing Evidence: Content and Sourcing; Interpretation; Synthesis
Key Concept: 3.1.I; 3.1.III; 3.2.I; 3.2.II; 3.3.III

102
Copyright © 2017, 2015, 2011, 2007 Pearson Education, Inc. All rights reserved.
Question 3.59: Using the documents and your knowledge of world history, analyze the responses to the
spread of Buddhism in China.

Historical Background: Buddhism had become a major force in Chinese life during the Six Dynasties era.
Throughout the classical and postclassical periods, Chinese scholars and political figures reacted
differently to the spread of Buddhism.

Document 1
Excerpt from “The Four Noble Truths” (The First Sermon Preached by the Buddha), 563 B.C.E–483 B.C.E,
According to Buddhist Tradition
The First Noble Truth is the Noble Truth of Sorrow. Birth is sorrow, age is sorrow, disease is sorrow,
death is sorrow, contact with the unpleasant is sorrow, separation from the pleasant is sorrow, every
wish unfulfilled is sorrow.

The Second Noble Truth is the Noble Truth of the Arising of Sorrow; it arises from craving, which leads
to rebirth, which brings delight and passion, and seeks pleasure—the craving for sensual pleasure, the
craving for continued life, and the craving for power.

The Third Noble Truth is the Noble Truth of Stopping of Sorrow. This is complete stopping of that
craving, so that no passion remains, leaving it, being emancipated from it, being released from it, giving
no place to it.

The Fourth Noble truth is the Noble Truth of the Way that Leads to the Stopping of Sorrow.

Document 2
Group of Stone Sculptures, Longmen Caves, China

At sites such as Longmen, near the Tang capital, massive statues of the Buddha were carved out of rocky cliff sides, beginning in
the sixth century C.E. Empress Wu had several multistory statues of Buddha created.

103
Copyright © 2017, 2015, 2011, 2007 Pearson Education, Inc. All rights reserved.
Document 3
Excerpt from Anonymous Chinese Scholar, “The Disposition of Error,” c. 500 C.E.
Question: If Buddhism is the greatest and most venerable of ways, why did the great sages of the past
and Confucius not practice it? If Confucian Classics no one mentions it. Why, then, do you love the Way
of the Buddha and rejoice in outlandish arts? Can the writings of the Buddha exceed the Classics and
commentaries and beautify the accomplishments of the sages?

Answer: All written works need not necessarily be the words of Confucius. To compare the sages to the
Buddha would be like comparing a white deer to a unicorn, or a swallow to a phoenix. The records and
teachings of the Confucian Classics do not contain everything. Even if the Buddha is not mentioned in
them, what occasion is there for suspicion?

Question: Now of happiness there is none greater than the continuation of one’s line, of unfilial conduct
there is none worse than childlessness. The monks forsake wives and children, reject property and
wealth. Some do not marry all their lives.

Answer: Wives, children, and property are the luxuries of the world, but simple living and inaction are
the wonders of the Way. The monk practices the Way and substitutes that for worldly pleasures. He
accumulates goodness and wisdom in exchange for the joys of having a wife and children.

Document 4
Excerpt from Han Yu (A Leading Confucian Scholar and Official at the Tang Imperial Court), “Memorial on Buddhism,” 817 C.E.
Your servant begs leave to say that Buddhism is no more a cult of the barbarian peoples spread to
China. It did not exist here in ancient times.

Now, I hear that Your majesty has ordered the community of monks to go to greet the finger bone of
the Buddha [a relic brought to China from India], and Your Majesty will ascend a tower to watch the
procession as this relic is brought into the palace. If these practices are not stopped, and this relic of the
Buddha is allowed to be carried from one temple to another, there will be those in the crowd who will
cut off their arms and mutilate their flesh in offering to the Buddha.

Now the Buddha was a man of the barbarians who did not speak Chinese and who wore clothes of a
different fashion. The Buddha’s sayings contain nothing about our ancient kings and the Buddha’s
manner of dress did not conform to our laws; he understood neither the duties that bind sovereign and
subject, nor the affections of father and son. If the Buddha were still alive today and came to our court,
Your Majesty might consent to receive him, but he would then be escorted to the borders of the nation,
dismissed, and not allowed to delude the masses. How then, when he has long been dead, could the
Buddha’s rotten bones, the foul and unlucky remains of his body, be rightly admitted to the palace?
Confucius said: “Respect ghost and spirits, but keep them at a distance!” Your servant is deeply
ashamed and begs that this bone from the Buddha be given to the proper authorities to be cast into fire
and water, that this evil be rooted out, and later generations spared this delusion.

104
Copyright © 2017, 2015, 2011, 2007 Pearson Education, Inc. All rights reserved.
Document 5
Excerpt from Zong Mi (A Leading Buddhist Scholar, Favored by the Tang Imperial Household), “On the Nature of Man,” Early
Ninth Century C.E.
Confucius, Laozi, and the Buddha were perfect sages. They established their teachings according to the
demands of the age and the needs of various beings. They differ in their approaches, in that they
encourage the perfection of good deeds, punish the wicked ones, and reward good ones; all three
teachings lead to the creation of an orderly society and for this they must be observed with respect.

Document 6
Excerpt from Tang Emperor Wu, Edict on Buddhism, 845 C.E.
We have heard that the Buddha was never spoken of before the Han dynasty; from then on the religions
of idols gradually came to prominence. So in this latter age Buddhism has transmitted its strange ways
and has spread like a luxuriant vine until it has poisoned the customs of our nation. Buddhism has
spread to all the nine provinces of China; each day finds its monks and followers growing more
numerous, and its temples more lofty. Buddhism wears out the people’s strength, pilfers their wealth,
causes people to abandon their lords and parents for the company of teachers, and severs man and wife
with its monastic decrees. In destroying law and injuring humankind indeed nothing surpasses this
doctrine!

Now if even one man fails to work the fields, someone must go hungry; if one woman does not tend her
silkworms, someone will go cold. At present there are inestimable number of monks and nuns in the
empire, all of them waiting for the farmers to feed them and the silkworms to clothe them while the
Buddhist public temples and private chapels have reached boundless numbers, sufficient to outshine
the imperial palace itself.

Having thoroughly examined all earlier reports and consulted public opinion on all sides, there no longer
remains the slightest doubt in Our minds that this evil should be eradicated.

Document 7
Excerpt from Zhi Dun (Chinese Scholar, Author, and Confidant of Chinese Aristocrats and High Officials During the Period When
Northern China Was Invaded by Central Asian Steppe Nomads), c. 350 C.E.
Whosever in China, in this era of sensual pleasures, serves the Buddha and correctly observes the
commandments, who recites the Buddhist scriptures, and who furthermore makes a vow to the reborn
without ever abandoning his sincere intention will, at the end of his life, when his [spirit] should [pass]
away, be miraculously transported thither. He will behold the Buddha and be enlightened in his sprit,
and then he will enter Nirvana.*

*Nirvana: the extinction of desire and individual consciousness.

Sample Answer

An acceptable response would utilize six or seven documents (all or all but one of the documents
provided) to deliver an analysis of the responses to the spread of Buddhism in China. The thesis, which
should consist of one or more congruent sentences in the opening paragraph, should present various
responses to Buddhism in China. An example of a good thesis might assert that, although some leaders
and scholars welcomed Buddhism, others that saw the religion as a threat to Chinese society and

105
Copyright © 2017, 2015, 2011, 2007 Pearson Education, Inc. All rights reserved.
wanted the religion and its adherents banned. Yet, there are scholars in these documents that believed
Buddhism could peacefully coexist with China’s indigenous belief systems.

Analysis of the documents should present evidence to support the thesis. Students might organize the
documents in a variety of ways around the factors cited in the thesis to describe a variety of responses
to Buddhism by political leaders, scholars, religious figures, and/or when or where the documents
originated. To achieve all four points in the evidence category of the rubric, students will need to relate
at least six of the documents to the thesis and analyze them by addressing historical context, audience,
purpose, and/or point of view in at least four of the documents. For example, in Document 1, students
might focus on the fact that Buddhism was founded in India and that its doctrine may (or may not) have
found congruence with indigenous Chinese belief systems. Document 2 provides tangible evidence that
Buddhism took root in China and that the Tang empress Empress Wu attempted to elevate Buddhism to
the state religion. The claim of the anonymous scholar in Document 3 argues that there is no reason for
suspicion, even though Buddhism was not mentioned in Confucian writings. Students might also note
that Document 3 does imply that Buddhist monks forsake worldly luxuries like a family in order to gain
goodness and wisdom, which would be considered against Confucian teachings. The audience for
Document 4 is the Tang emperor himself, and the author, Han Yu, is a leading Confucian scholar and an
official of the imperial court. Han Yu begs the emperor not to allow the relic of the Buddha to be
brought into the palace and instead wishes it to be destroyed. This Confucian scholar argues that
Buddhism is a foreign and barbarian entity that should be banished in order to protect current and
future Chinese people from its delusions. The conciliatory tone taken by the Buddhist scholar in
Document 5 is conveyed in the idea that the central figures of Confucianism, Daoism, and Buddhism are
all “perfect sages” whose teachings should all be respected as they lead to an orderly society. In
Document 6, Emperor Wu of the Tang dynasty acknowledges the spread of Buddhism and has issued an
edict that admonishes Buddhist monastic practices as it destroys laws and injures humankind. He
portrays the monks and followers as freeloaders and declares that Buddhism should be eradicated.
Lastly, in Document 7, an earlier scholar, author, and confidant of Chinese aristocrats and high officials
openly discusses the basic beliefs of Buddhism and that those that forsake worldly luxuries and follow
Buddhist teachings can find enlightenment.

Strong essays will not simply list descriptions or analysis of the documents, but rather make connections
between the documents in order to provide arguments that support the thesis statement. Some
examples of these connections that are consistent with the sample thesis provided above are (1) a
comparison of how Buddhism was welcomed in China, which is supported by Documents 2 and 7; (2) a
strong rebuke of the presence of Buddhism in China, found in Documents 1, 4, and 6; (3) a discussion of
how some scholars believed Buddhism and the indigenous belief systems of China could coexist
(Documents 3 and 5).

When writing the response to this document-based question, it is necessary for students to provide
outside examples and knowledge to support their thesis. An example of outside knowledge that isn’t
discussed in the documents provided is the different forms of Buddhism that found preference among
different segments of Chinese society. The pure land strain of Mahayana Buddhism won many
adherents among the peasantry because of its salvationist message; in contrast, the Chan, or Zen, strain
of Buddhism, with its focus on meditation and beauty, had great appeal among the educated and elite
classes.

The contextualization point could be earned by commenting on how Buddhism was received by Korean,
Japanese, and Vietnamese cultures during the Sinification of Asia.

106
Copyright © 2017, 2015, 2011, 2007 Pearson Education, Inc. All rights reserved.
Common examples that may be used to earn the synthesis point are to discuss the Daoist response to
Buddhist gains in China, as there are no documents that directly represent this point of view. Another
example students might use to gain the synthesis point is to discuss how Buddhism was accepted in its
birthplace of India, where there was also a dominant and established belief system—Hinduism.

Topic: State and Religion in the Tang and Song Eras; The Anti-Buddhist Backlash
Theme: Development and Interaction of Cultures; State Building, Expansion, and Conflict
Learning Objective: CUL–2; CUL–3; CUL–4; CUL–5; CUL–6; SB–4; SB–8
Historical Thinking Skill: Comparison; Contextualization; Analyzing Evidence: Content and Sourcing;
Interpretation; Synthesis
Key Concept: 3.1.III; 3.2.I; 3.3.III

107
Copyright © 2017, 2015, 2011, 2007 Pearson Education, Inc. All rights reserved.
Period 4
Global Interactions
(c. 1450 to c. 1750)
Section I
Multiple-Choice Questions
Questions 4.1–4.3 refer to the image below.

The Port of Lisbon, Portugal

4.1. Identify a direct result of the demand for increased transoceanic travel and trade.
(A) Europeans built on the technological knowledge of past empires and their understanding of
wind patterns to create new ship designs.

108
Copyright © 2017, 2015, 2011, 2007 Pearson Education, Inc. All rights reserved.
(B) Europeans traded sugar and slaves with empires from China to the Americas.
(C) Europeans acquired more slaves after the “great dying” as the demand for sugar increased.
(D) Europeans depended on the sextant and their understanding of windwheels to sail around
the tip of Africa.

Answer: A
Topic: Maritime technology; Technological innovations
Theme: Creation, Expansion, and Interaction of Economic Systems
Learning Objective: ECON–10; ECON–12
Historical Thinking Skill: Patterns of Continuity and Change over Time
Key Concept: 4.1.II

4.2. Which of the following statements is a supportable inference based on the image?
(A) Russian sailors, after turning to new navigation techniques, began to trade with the
Americas.
(B) Changing patterns of long-distance trade intensified regional trade networks and spurred
the growth of port cities.
(C) With the growth of new shipping companies, Australia became a major destination for
settlement and for the fur trade.
(D) Demand for goods such as coffee and tobacco spurred the growth of hacienda-style
plantations in the Azores.

Answer: B
Topic: Maritime technology; Technological innovations
Theme: Creation, Expansion, and Interaction of Economic Systems
Learning Objective: ECON–10; ECON–12
Historical Thinking Skill: Synthesis
Key Concept: 4.1.II

4.3. What method did European rulers use to control their domestic and colonial economies?
(A) European rulers taxed their colonies heavily to ensure more power for the crown.
(B) European rules lowered import duties to help increase the demand for goods and promote
trade across the globe.
(C) European rulers and merchants used joint-stock companies to compete against their rivals
in global trade.
(D) European rulers used the development of new international banking practices to safeguard
shipments of gold from the Americas.

Answer: C
Topic: Mercantilism; Joint-stock companies
Theme: State Building, Expansion, and Conflict; Creation, Expansion, and Interaction of Economic
Systems
Learning Objective: SB–2; ECON–3
Historical Thinking Skill: Analyzing Evidence: Content and Sourcing
Key Concept: 4.1.IV.C

109
Copyright © 2017, 2015, 2011, 2007 Pearson Education, Inc. All rights reserved.
Questions 4.4–4.6 refer to the map below.

Major Political Units of the World, c. 1750

4.4. Identify a direct result of the intensification of the global trade network on preexisting regional
trade networks after the 1500s?
(A) The intensification resulted in more economic power for the Swahili states in East Arica.
(B) The intensification did not have a significant effect on the preexisting regional trade
networks in the Indian Ocean, in the Mediterranean, in the Sahara, or on overland Eurasian
routes.
(C) The intensification resulted in prosperity but also economic disruptions among merchants
and governments in the Indian Ocean, in the Mediterranean, in the Sahara, and on overland
Eurasian routes.
(D) The intensification resulted in great wealth for the empires of Britain and Italy.

Answer: C
Topic: Globalizing networks and their political and economic effects
Theme: State Building, Expansion, and Conflict
Learning Objective: SB–4
Historical Thinking Skill: Analyzing Evidence: Content and Sourcing
Key Concept: 4.1.I

4.5. During the period from 1450 to 1750, transoceanic maritime voyages included which of the
following?
(A) Japanese voyages to the Americas to trade for gold and to create settlements for Japan’s
expanding population
(B) Portuguese voyages to West Africa, Spanish voyages across the Atlantic, and North Atlantic
crossings for fishing and settlements

110
Copyright © 2017, 2015, 2011, 2007 Pearson Education, Inc. All rights reserved.
(C) Chinese reconnaissance voyages along the western coast of North and South America and to
the Pacific Islands and the Atlantic Ocean
(D) East African merchant voyages to the Indian Ocean for trade and exploration

Answer: B
Topic: State support for maritime voyages and trade
Theme: Creation, Expansion, and Interaction of Economic Systems; State Building, Expansion, and
Conflict
Learning Objective: ECON–3; ECON–12; ECON–13; SB–3
Historical Thinking Skill: Causation
Key Concept: 4.1.III.A: 4.1.III.B; 4.1.III.C

4.6. What characterized the role of European merchants in Asian trade?


(D) The Pacific system involved the movement of goods and the mixing of various cultures of
peoples and types of laborers.
(D) Europeans typically did not interfere with Asian trade in their efforts to promote European
products and services.
(D) Europeans monopolized the local slave trade in an attempt to recreate the Atlantic slave
trade in Asia.
(D) Europeans transported goods from one Asian country to another market in Asia or in the
Indian Ocean region.

Answer: D
Topic: Mercantilism; Joint-stock companies
Theme: State Building, Expansion, and Conflict
Learning Objective: SB–3
Historical Thinking Skill: Analyzing Evidence: Content and Sourcing, Causation
Key Concept: 4.1.IV.A

111
Copyright © 2017, 2015, 2011, 2007 Pearson Education, Inc. All rights reserved.
Questions 4.7–4.9 refer to the map below.

The Columbian Exchange

4.7. Identify a consequence of the Columbian Exchange.


(A) Amerindian populations surged.
(B) Africans were forced to migrate to Europe and the Americas.
(C) New crops were introduced into the Old World and the New World.
(D) European populations surged.

Answer: D
Topic: Columbian Exchange; Atlantic world; Atlantic System
Theme: Development and Transformation of Social Structures; Creation, Expansion, and Interaction of
Economic Systems
Learning Objective: SOC–2; ECON– 5
Historical Thinking Skill: Causation
Key Concept: 4.1.V.A; 4.1.V.B

4.8. Which American crops became staple foods and benefitted the populations of Afro-Eurasia?
(A) sugar, potatoes, maize
(B) potatoes, maize, manioc, quinoa
(C) potatoes and maize
(D) citrus fruits, potatoes, sugar beets

112
Copyright © 2017, 2015, 2011, 2007 Pearson Education, Inc. All rights reserved.
Answer: C
Topic: Columbian Exchange
Theme: Creation, Expansion, and Interaction of Economic Systems
Learning Objective: ECON–1
Historical Thinking Skill: Analyzing Evidence: Content and Sourcing
Key Concept: 4.1.V.B; 4.1.V.D

4.9. What was the result of Europeans bringing their agricultural and settlement practices to the
Americas?
(A) The physical environment was impacted through deforestation and soil depletion.
(B) European settlements were overshadowed by those of the Amerindians.
(C) The Amerindians adopted many of the agriculture and settlement practices of the
Europeans.
(D) The Europeans adopted the agricultural practices of the Amerindians to protect the
environment.

Answer: A
Topic: Colonization
Theme: Interaction Between Humans and the Environment; State Building, Expansion, and Conflict
Learning Objective: ENV–5; SB–3
Historical Thinking Skill: Causation
Key Concept: 4.1.V.E

113
Copyright © 2017, 2015, 2011, 2007 Pearson Education, Inc. All rights reserved.
Questions 4.10–4.12 refer to the images below.

Jesuits in Chinese Dress at the Chinese Emperor’s Court Clerics Devouring Their Flocks

4.10. The increase in the intensification of exchange between the hemispheres resulted in which of the
following?
(A) The concept of God, Gold, and Glory limited the expansion of Christianity into Africa.
(B) Belief systems did not blend as people migrated throughout the hemispheres.
(C) Indian Ocean traders from the coast of Swahili brought Christianity to Indonesia.
(D) The practice of Christianity continued to spread throughout Eurasia aided by the Jesuits.

Answer: D
Topic: Reform of existing religions and creation of syncretic belief systems
Theme: Development and Interaction of Cultures
Learning Objective: CUL–2
Historical Thinking Skill: Argumentation
Key Concept: 4.1.IV

4.11. In the period from 1450 to 1750, which of the following factors contributed most directly to the
spread and reform of existing religions and creation of syncretic belief systems?
(A) the increase in interactions and connections between and within the hemispheres

114
Copyright © 2017, 2015, 2011, 2007 Pearson Education, Inc. All rights reserved.
(B) the blending of Islam and Christianity and rise of trade, which brought the two faiths
together
(C) as Buddhism declined in East Asia, the spread of Christianity and Hinduism to the region by
merchants
(D) Belief systems did not undergo reforms or changes during this time period.

Answer: A
Topic: Reform of existing religions and creation of syncretic belief systems
Theme: Creation, Expansion, and Interaction of Economic Systems; Development and Interaction of
Cultures
Learning Objective: ECON–8; CUL–2; CUL–4; CUL–5
Historical Thinking Skill: Causation
Key Concept: 4.1.IV

4.12. Existing belief systems in the period from 1450 to 1750 included all of the following reforms or
changes except one. Which one?
(A) the reform of Haitian vodun as a result of interactions between Christianity and African
religions
(B) the reform of Sikhism in South Asia as a result of interactions between Hinduism and Islam
(C) the end of the split between the Sunni and Shi’a as a result of the contact between the
Ottomans and Safavids
(D) the continued spread of Islam to Afro-Eurasia as Sufi practices encouraged the adaption of
Islam to local practices

Answer: C
Topic: Syncretic belief systems and practices; The Shi’a Challenge of the Safavids
Theme: Development and Interaction of Cultures
Learning Objective: CUL–4
Historical Thinking Skill: Analyzing Evidence: Content and Sourcing
Key Concept: 4.1.VI; 4.1.V

115
Copyright © 2017, 2015, 2011, 2007 Pearson Education, Inc. All rights reserved.
Questions 4.13–4.15 refer to the image below.

Silver Mine at Potosí

4.13. Commercialization and the creation of a global economy in the period from 1450 to 1750 were
closely connected to which of the following?
(A) the rise of slavery across Africa
(B) the new global circulation of silver from the Americas
(C) the discovery of gold in California
(D) the increase in trade across the Silk Road because of the Pax Mongolica

Answer: B
Topic: Colonial objectives
Theme: State Building, Expansion, and Conflict
Learning Objective: SB–3
Historical Thinking Skill: Analyzing Evidence: Content and Sourcing
Key Concept: 4.1.IV.B

4.14. Colonial economies in the Americas depended on a range of coerced labor, including which of the
following?
(A) Spanish adaptation of the Inca mita system
(B) practices of serfdom brought to the New World by Russian immigrants
(C) taxation of locals to raise money for the home country
(D) practices of indentured servitude developed by new African elites

116
Copyright © 2017, 2015, 2011, 2007 Pearson Education, Inc. All rights reserved.
Answer: A
Topic: Coerced labor
Theme: Creation, Expansion, and Interaction of Economic Systems
Learning Objective: ECON–5
Historical Thinking Skill: Causation
Key Concept: 4.2.II.B

4.15. Where did the movement of goods, wealth, and free and unfree laborers, and the mixing of
African, American, and European cultures and peoples take place?
(A) during the Middle Passage
(B) on the Indian Ocean and in the South Seas
(C) in and around the Pacific
(D) across the Atlantic

Answer: D
Topic: African, American, European interaction
Theme: State Building, Expansion, and Conflict
Learning Objective: SB–4
Historical Thinking Skill: Interpretation; Synthesis
Key Concept: 4.1.IV.D

117
Copyright © 2017, 2015, 2011, 2007 Pearson Education, Inc. All rights reserved.
Questions 4.16–4.18 refer to the image and chart below.

Estimated Number of Africans Imported to British


Collection of Taxes from the Russian Peasantry North America, 1701–1770

SOURCE: Data from Eric Williams, Capitalism and


Slavery (Chapel Hill, NC: University of North Carolina
Press, 1944).

4.16. All of the following factors except one caused peasant labor to intensify in many areas during the
period from 1450 to 1750. Which one?
(A) the increase in cotton textile production in India
(B) the increase in silk textile production in China
(C) the increase in the flow of silver from the Americas
(D) the development of frontier settlements in Russian Siberia

Answer: C
Topic: New political and economic elites, plantations; Coerced labor
Theme: Creation, Expansion, and Interaction of Economic Systems
Learning Objective: ECON– 3; ECON–4,
Historical Thinking Skill: Argumentation
Key Concept: 4.2.II.A

4.17. The increased demand for slaves in the Americas was a direct result of which of the following?
(A) warring kingdoms in Africa that provided more captives to slave traders
(B) the growth of the plantation economy
(C) the European expansion across the plains
(D) improvements in shipping and maritime technology

Answer: B
Topic: Increased demand for labor
Theme: Development and Transformation of Social Structures

118
Copyright © 2017, 2015, 2011, 2007 Pearson Education, Inc. All rights reserved.
Learning Objective: SOC–7
Historical Thinking Skill: Causation
Key Concept: 4.2.II.C

4.18. Colonial economies in the Americas depended on a range of coerced labor systems such as
__________.
(A) chattel slavery, indentured servitude, and tributary states
(B) the mita system, serfdom, and chattel slavery
(C) serfdom, the encomienda and hacienda systems, and indentured servitude
(D) indentured servitude, encomienda and hacienda systems, and chattel slavery

Answer: D
Topic: Coerced labor
Theme: Creation, Expansion, and Interaction of Economic Systems
Learning Objective: ECON–5
Historical Thinking Skill: Analyzing Evidence: Content and Sourcing
Key Concept: 4.2.II.D

119
Copyright © 2017, 2015, 2011, 2007 Pearson Education, Inc. All rights reserved.
Questions 4.19–4.21 refer to the passage below.

When the dance was loveliest and when song was linked to song, the Spaniards were seized with an
urge to kill the celebrants. They all ran forward, armed as if for battle. They closed the entrances and
passageways . . . then [they] rushed into the Sacred Patio to slaughter the inhabitants . . . they attacked
the man who was drumming and cut off his arms. Then they cut off his head, and it rolled across the
floor. They attacked all the celebrants stabbing them, spearing them, striking them with swords . . .
others they beheaded . . . or split their heads to pieces. . . .The blood of the warriors flowed like water
and gathered into pools. . . .They invaded every room, hunting and killing.

Aztec Account of the Massacre at the Temple (from the Florentine Codex), May 1520

4.19. Which of the following did not contribute to the Spanish conquest of the Aztecs?
(A) The Aztecs were a peaceful civilization.
(B) The Aztecs were not yet immune to diseases that the Spaniards carried.
(C) The Spaniards had made several alliances with the enemies of the Aztecs.
(D) The Spaniards had superior weaponry and armor.

Answer: A
Topic: Colonial empires
Theme: State Building, Expansion, and Conflict
Learning Objective: SB–9
Historical Thinking Skill: Causation; Interpretation
Key Concept: 4.3.II.C

4.20. Fra Bartolomé de las Casas __________.


(A) argued in favor of the mita
(B) supported peaceful conversion of the native American population to Christianity
(C) worked closely with Hernán Cortés to establish the administrative structure of New Spain
(D) established Christian missions in California

Answer: B
Topic: Changing political and economic structures; Restructuring of hierarchies
Theme: Development and Transformation of Social Structures; State Building, Expansion, and Conflict;
Development and Interaction of Cultures; Creation, Expansion, and Interaction of Economic Systems
Learning Objective: SOC–2; SB–2; SB–4; CUL–2; CUL–4; ECON–5
Historical Thinking Skill: Causation
Key Concept: 4.1.VI; 4.2.II; 4.3.I

4.21. How did fifteenth- and sixteenth-century Europeans view the inhabitants of the New World?
(A) Europeans viewed them as barbarians after witnessing the Mexica and Aztec practice of
human sacrifice.
(B) Europeans viewed them as enemies because of their advanced technology.
(C) Europeans viewed them as children who needed to be educated.
(D) Europeans viewed them as brothers and sisters who would teach them the skills necessary
to survive.

120
Copyright © 2017, 2015, 2011, 2007 Pearson Education, Inc. All rights reserved.
Answer: A
Topic: Rulers legitimize power; Differential treatment of groups
Theme: State Building, Expansion, and Conflict
Learning Objective: SB–1; SB–7
Historical Thinking Skill:
Key Concept: 4.3.I.A

121
Copyright © 2017, 2015, 2011, 2007 Pearson Education, Inc. All rights reserved.
Questions 4.22–4.24 refer to the images below.

Suleymaniye Mosque and Golden Horn, Istanbul, Turkey Taj Mahal at Agra

4.22. Why did rulers continue to use monumental architecture during the period from 1450 to 1750?
(A) to celebrate their coronations
(B) to create a gathering places for worship
(C) to legitimize their rule
(D) to provide jobs for the masses.

Answer: C
Topic: Legitimization and consolidation of imperial rule
Theme: Development and Transformation of Social Structures
Learning Objective: SOC–7
Historical Thinking Skill: Patterns of Continuity and Change over Time
Key Concept: 4.3.I.A

4.23. What concept supported the ruler’s power?


(A) the mandate of kings
(B) strict succession policies
(C) divine right
(D) the right of sultans

Answer: C
Topic: Legitimization and consolidation of imperial rule
Theme: Development and Transformation of Social Structures
Learning Objective: SOC–7
Historical Thinking Skill: Contextualization
Key Concept: 4.3.I A

4.24. What was a direct result of the use of gunpowder, canons, and armed trade by the Ottoman and
Mughal empires?
(A) The Ottomans became land locked while the Mughals enjoyed new status.
(B) Their land empires expanded dramatically in size.

122
Copyright © 2017, 2015, 2011, 2007 Pearson Education, Inc. All rights reserved.
(C) They had to create new economic systems .
(D) Their burgeoning economies resulted in religious disharmony.

Answer: B
Topic: Land-based and maritime expansion
Theme: State Building, Expansion, and Conflict
Learning Objective: SB–3
Historical Thinking Skill: Causation
Key Concept: 4.3.II.B

123
Copyright © 2017, 2015, 2011, 2007 Pearson Education, Inc. All rights reserved.
Questions 4.25–4.27 refer to the map below.

Routes and Major Products Exchanged in the Asian Trading Network, c. 1500

4.25. Which of the following provided significant challenges to state consolidation and expansion during
the period from 1450 to 1750?
(A) competition over trade routes, state rivalries, and local resistance
(B) lack of refueling stations for large ocean-going vessels
(C) lack of understanding of the monsoon winds
(D) religious differences among various trading networks and peoples.

Answer: A
Topic: Economic and political rivalries
Theme: State Building, Expansion, and Conflict
Learning Objective: SB–3
Historical Thinking Skill: Comparison
Key Concept: 4.3.III

4.26. What role did Europeans play in Indian Ocean trade that caused increased competition between
the Europeans and the Ottomans?
(A) The Europeans were unsuccessful traders and used the Ottomans as middlemen.

124
Copyright © 2017, 2015, 2011, 2007 Pearson Education, Inc. All rights reserved.
(B) The Europeans signed agreements with the Mughals that cut the Ottomans out of their
trade networks.
(C) The Europeans attempted to use force to regulate and dominate the spice trade.
(D) The Europeans traded furs for gold and spices.

Answer: C
Topic: Inter-state rivalry
Theme: Creation, Expansion, and Interaction of Economic Systems
Learning Objective: ECON–3
Historical Thinking Skill: Causation
Key Concept: 4.3.III

4.27. The new trading-post empires established in the period from 1450 to 1750 in Africa and Asia were
profitable for the Europeans but had an adverse effect on the power of which states?
(A) on India, Indonesia, and Sri Lanka
(B) on interior states in West and Central Africa
(C) on India, Cambodia, and Melaka
(D) on East African states, India, and Japan

Answer: B
Topic: New political and economic elites
Theme: Creation, Expansion, and Interaction of Economic Systems
Learning Objective: ECON–3
Historical Thinking Skill: Synthesis
Key Concept: 4.3.II.A

125
Copyright © 2017, 2015, 2011, 2007 Pearson Education, Inc. All rights reserved.
Questions 4.28–4.30 refer to the image and passage below.

Caricature of John Tetzel, Famous Indulgence Preacher

“As soon as gold in the basin rings, right then the soul to
Heaven springs.”

5. The pope has neither the will nor the power to remit any penalties beyond those imposed either
at his own discretion or by canon law.
6. The pope himself cannot remit guilt, buy only declare and confirm that it has been remitted by
God.
21. Hence those who preach indulgences are in error when they say that a man is absolved and saved
from every penalty by the pope’s indulgences.
27. There is no divine authority for preaching that the soul flies out of purgatory immediately the
money chinds in the bottom ofthe chest.
36. Any Christian whatsoever, who is truly repentant,enjoys plenary remission from penalty and guilt,
and this is given to him by God without letters of indulgence.
62. The true treasure of the Church is the Holy Gospel of the glory and the grace of God.

Six Theses, or Propositions, from Martin Luther’s 95 Theses, 1517

4.28. What is the central subject of the image and passage?


(A) the Catholic Reformation
(B) the conflict between the Eastern and Western branches of the Catholic Church
(C) the remission of sins for joining the Crusades
(D) the sale of indulgences

Answer: D

126
Copyright © 2017, 2015, 2011, 2007 Pearson Education, Inc. All rights reserved.
Topic: Reform of existing religions and creation of syncretic belief systems
Theme: Development and Interaction of Cultures
Learning Objective: CUL–2
Historical Thinking Skill: Analyzing Evidence: Content and Sourcing
Key Concept: 4.1.IV

4.29. What was the outcome to the controversy depicted in the image and passage?
(A) the divorce and remarriage of Henry VIII
(B) the Protestant Reformation and the creation of syncretic belief systems and practices
(C) the spread of ideas across Europe via the printing press
(D) the end of interpretation of the bible by the clergy

Answer: B
Topic: Spread of religions and religious syncretism in regional and newly global trade networks
Theme: Creation, Expansion, and Interaction of Economic Systems
Learning Objective: ECON–8
Historical Thinking Skill: Causation
Key Concept: 4.1.IV

4.30. Which of the following is similar to the split and diffusion of Christianity during the period from
1450 to 1750?
(A) the growth of Buddhism in South Asia and on the islands in Southeast Asia
(B) the spread of Sufi practices to the Americas as believers adapted Islamic practices
(C) the development of Buddhism in relation to Hinduism but with the rejection of the caste
system
(D) the development Sikhism as a result of interactions between Hinduism and Islam

Answer: D
Topic: Syncretic belief systems and practices; Reform of existing religions and creation of syncretic belief
systems
Theme: Development and Interaction of Cultures
Learning Objective: CUL–4
Historical Thinking Skill: Analyzing Evidence: Content and Sourcing
Key Concept: 4.1.VI

127
Copyright © 2017, 2015, 2011, 2007 Pearson Education, Inc. All rights reserved.
Questions 4.31–4.34 refer to the image below.

The Ottoman Siege of Constantinople, 1453

4.31. Which of the following enabled the Manchu, Mughal, Ottoman, and Russian empires to expand
dramatically in size?
(A) the relatively weak West
(B) armed trade, gunpowder, and cannons
(C) the power vacuum created by religious wars in Europe
(D) the Pax Mongolica

Answer: B
Topic: Maritime and land-based empires
Theme: State Building, Expansion, and Conflict
Learning Objective: SB–9
Historical Thinking Skill: Causation
Key Concept: 4.3.II.B

4.32. Which of the following was a consequence of the Ottoman conquest of Constantinople?
(A) Europeans began to search for alternate routes to the Indies and looked to overseas
expansion.
(B) China and Japan closed their borders to all foreign trade and visitors.
(C) The slave trade and violence in Africa increased as slavers became allies with African
kingdoms.
(D) The Russians joined forces with the Europeans to prevent Ottoman expansion.

128
Copyright © 2017, 2015, 2011, 2007 Pearson Education, Inc. All rights reserved.
Answer: A
Topic: Intensification of existing trade routes; New maritime routes
Theme: Creation, Expansion, and Interaction of Economic Systems
Learning Objective: ECON–12
Historical Thinking Skill: Argumentation
Key Concept: 4.1.1

4.33. In contrast to the expansion of the Ottoman and Safavid empires, the Mughals, in the period from
1450 to 1750, __________.
(A) were a maritime empire that conquered islands in the Indian Ocean
(B) became allies with the Chinese and held off the Safavid
(C) did not launch conquests nor expand out of religious fervor
(D) were a peaceful people who converted to Buddhism under Ashoka

Answer: C
Topic: State consolidation and imperial expansion
Theme: State Building, Expansion, and Conflict
Learning Objective: SB–4
Historical Thinking Skill: Analyzing Evidence: Content and Sourcing
Key Concept: 4.3.II.B

4.34. The reign of which European monarch coincided with the rule of Akbar over the Mughal dominion
in the Indian subcontinent?
(A) Charles V of France
(B) Gustav I of Sweden
(C) James I of England
(D) Philip II of Spain

Answer: D
Topic: Legitimization and consolidation of imperial rule; Akbar’s Religious Syncretism, Hindu Allies, and a
Multicultural Empire
Theme: Development and Transformation of Social Structures
Learning Objective: SOC–7
Historical Thinking Skill: Periodization
Key Concept: 4.3.I

129
Copyright © 2017, 2015, 2011, 2007 Pearson Education, Inc. All rights reserved.
Questions 4.35–4.37 refer to the map and passage below.

The Ottoman, Safavid, and Mughal Empires

130
Copyright © 2017, 2015, 2011, 2007 Pearson Education, Inc. All rights reserved.
SOURCE: C. T. Foster and F. H. Blackburne Daniell, The Life and Letters of Ogier Ghiselin de Busbecq, 1881.

The Supreme Being who is at once the sovereign arbiter of the destinies of men and the source of all
light and knowledge, declares in the Qur’an that the true faith is that of the Muslims, and that whoever
professes another religion, far from being hearkened to and saved, will on the contrary be cast out
among the rejected on the great day of the Last Judgment; . . . Place yourself, O Prince, among the true
believers, those who walk in the path of salvation, and who turn aside with care from vice and infidelity.
May the purest and holiest blessings be upon Muhammad, the master of the two worlds, the prince of
prophets, as well as upon his descendants and all who follow his Law! . . .

Animated by this [religious decree], conforming to the Qur’an, the code of divine laws, and wishing on
one side to strengthen Islam, on the other to liberate the lands and peoples who writhe under your
yoke, we have resolved to lay aside our imperial robes in order to put on the shell and coat of mail, to
raise our ever victorious banner, to assemble our invincible armies, to take up the gauntlet of the
avenger, to march with our soldiers, whose sword strikes mortal blows. . . . In pursuit of this noble
resolution, we have entered upon the campaign, . . . we come, before commencing war, to set out
before you the words of the Qur’an, in place of the sword, and to exhort you to embrace the true faith;
this is why we address this letter to you . . .

Letter from Ottoman Ruler Selim I to Safavid Leader Ismail I

4.35. The interactions described in the passage are best understood in which of the following contexts?
(A) The Ottoman ruler Selim I welcomes Safavid ruler Ismail I and asks him to join the campaign.
(B) The Ottoman ruler Selim I accuses the Shi’ite Safavids of heresy and is preparing for battle.
(C) The Ottoman ruler Selim I attemps to expand his territory because of trade violations by the
Safavids.
(D) The Ottoman ruler Selim I requests an alliance with Ismail I as a fellow Muslim to battle the
Mughals.

Answer: B
Topic: Rulers legitimize power
Theme: State Building, Expansion, and Conflict
Learning Objective: SB–1
Historical Thinking Skill: Causation; Contextualization
Key Concept: 4.3.I.A

4.36. What role did the variants of Islam play in the interactions between the Mughal, Ottoman, and
Safavid empires?
(A) The Ottomans welcomed the Safavids and Mughals to join their empire and to promote
Islam throughout the Eastern Hemisphere.
(B) The Mughals and the Ottomans were allies and joined in campaigns against the Safavids.
(C) The Ottomans and Safavids engaged in frequent warfare because of the divisive competition
between Muslim sects.
(D) The Mughals, Ottomans, and Safavids recognized the sovereignty of each empire and did
not attempt to encroach on each other’s territory.

Answer: C

131
Copyright © 2017, 2015, 2011, 2007 Pearson Education, Inc. All rights reserved.
Topic: Competition over trade routes; State rivalries
Theme: State Building, Expansion, and Conflict
Learning Objective: SB–10
Historical Thinking Skill: Argumentation
Key Concept: 4.3.III

4.37. Which of the following was a similar factor in the rise of Mughal, Ottoman, and Safavid empires?
(A) Each empire took advantage of the power vacuum that resulted from the devastation of
Timur and the breakup of the Mongol empire.
(B) Each empire took advantage of the strength of their Janissaries to defeat others in warfare
and expand their territories.
(C) Silver from the Americas stabilized the economies of each empire and allowed for their
rapid rise and expansion.
(D) Each empire benefitted from the expansion of Islam and the wealth gained from Indian
Ocean trade networks.

Answer: A
Topic: Land-based and maritime expansion
Theme: State Building, Expansion, and Conflict
Learning Objective: SB–3
Historical Thinking Skill: Analyzing Evidence: Content and Sourcing; Comparison
Key Concept: 4.3.II.B

132
Copyright © 2017, 2015, 2011, 2007 Pearson Education, Inc. All rights reserved.
Questions 4.38–4.40 refer to the images and passage below.

Nineteenth Engraving of the Cubicles for Imperial Civil Service Exams in Beijing

Wives and Concubines of Ming Emperors Depicted in a Scene of Court Life

133
Copyright © 2017, 2015, 2011, 2007 Pearson Education, Inc. All rights reserved.
SOURCE: Matthew Ricci, China in the Sixteenth Century, trans. Louis J. Gallagher, S.J. (New York: Random House, Inc., 1942).

Another remarkable fact and quite worthy of note as marking a difference from the West, is that the
entire kingdom is administered by the Order of the Learned, commonly known as the Philosophers. The
responsibility for orderly management of the entire realm is wholly and completely committed to their
charge and care. The army, both officers and soldiers, hold them in high respect and show them the
promptest obedience and deference, and not infrequently the military are disciplined by them as a
schoolboy might be punished by his master. Policies of war are formulated and military questions are
decided by the Philosophers only, and their advice and counsel has more weight with the King than that
of the military leaders. In fact very few of these, and only on rare occasions, are admitted to war
consultations. Hence it follows that those who aspire to be cultured frown upon war and would prefer
the lowest rank in the philosophical order to the highest in the military, realizing that the Philosophers
far excel military leaders in the good will and the respect of the people and in opportunities of acquiring
wealth.

Matthew Ricci, China in the Sixteenth Century, 1942

4.38. Identify an important feature of Chinese culture during the Ming dynasty,
(A) Emperors conducted elaborate public performances of Confucian rituals to legitimize their
rule.
(B) Chinese males were forced to wear the queue to demonstrate loyalty to the emperor.
(C) The employment of eunuchs in the Forbidden City ended, giving the emperor more power.
(D) The popularity of Buddhism increased, leading to a decline in Confucian values and
education.

Answer: A
Topic: Confucian rituals
Theme: Development and Transformation of Social Structures
Learning Objective: SOC–5
Historical Thinking Skill: Analyzing Evidence: Content and Sourcing
Key Concept: 4.3.1.A

4.39. Which of the following statements regarding women’s status during the Ming and Qing dynasties
is true?
(A) Emperors granted women more rights and discouraged the practice of footbinding.
(B) Women were encouraged to study the Confucian classics to prepare for civil service exams.
(C) Women continued to be subjugated, and the practice of footbinding was prevalent among
the elite.
(D) With the arrival of Europeans, new ideas regarding emancipation for Chinese women were
enacted.

Answer: C
Topic: Changing political and economic structures affected gender hierarchies
Theme: Development and Transformation of Social Structures
Learning Objective: SOC–1
Historical Thinking Skill: Synthesis
Key Concept: 4.2.III.A

134
Copyright © 2017, 2015, 2011, 2007 Pearson Education, Inc. All rights reserved.
4.40. What enabled the Ming emperors to maintain centralized control over the population and
resources?
(A) Ming emperors practiced tight control over eunuchs to maintain harmony within the
empire.
(B) Ming emperors recruited elites to prepare for the Chinese examination system.
(C) Extensive trade with the Ottomans brought more wealth to Ming emperors.
(D) The Great Wall isolated Ming emperors and led to a nation without resources.

Answer: B
Topic: Techniques of imperial administration
Theme: Creation, Expansion, and Interaction of Economic Systems
Learning Objective: ECON–3
Historical Thinking Skill: Analyzing Evidence: Content and Sourcing
Key Concept: 4.3.I.C

135
Copyright © 2017, 2015, 2011, 2007 Pearson Education, Inc. All rights reserved.
Questions 4.41–4.43 refer to the images, map, and table below.

Cartoon Lampooning Peter the Great’s Order to the Nobility to


Cathedral of St. Basil the Blessed, Red Square, Moscow Cut Off Their Beards

136
Copyright © 2017, 2015, 2011, 2007 Pearson Education, Inc. All rights reserved.
Russia Under Peter the Great

1450 C.E. 1600 C.E. 1750 C.E.


1462 Much of Russia freed from 1604–1613 Time of Troubles 1762–1796 Reign of Catherine the
Tatars by Ivan III (Ivan the Great
Great)
1480 Moscow region free; 1613–1917 Romanov dynasty 1773–1775 Pugachev revolt
Russian expansion presses
south
1533–1584 Life of Ivan IV (Ivan the 1637 Russian pioneers to Pacific 1772 1793 Partition of Poland
Terrible), first to 1795
emphasize the title of
tsar; boyar power
reduced
1552–1556 Russian expansion in 1649 Law enacted making 1785 Law enacted tightening
central Asia, western serfdom hereditary landlord power over serfs
Siberia
1689–1725 Reign of Peter the Great
1700–1721 Wars with Sweden
1703 Founding of St.
Petersburg

137
Copyright © 2017, 2015, 2011, 2007 Pearson Education, Inc. All rights reserved.
4.41. Which characterized Russian patterns of territorial expansion and settlement during this period?

(A) Boyars expanded north into Finland and to the south, where they clashed with the
Ottomans.
(B) Russia was landlocked at this time and did not expand its empire.
(C) Russia expanded into central Asia and into western Siberia, where peasants were recruited
to settle the new lands.
(D) Russia expanded into central Asia and western Europe, where Russian agricultural
techniques were in demand.

Answer: C
Topic: Land-based and maritime expansion; Coerced labor
Theme: State Building, Expansion, and Conflict; Creation, Expansion, and Interaction of Economic
Systems
Learning Objective: SB–3; ECON–5
Historical Thinking Skill: Analyzing Evidence: Content and Sourcing
Key Concept: 4.3.II.B; 4.2.II.A

4.42. As an absolute ruler, what change did Peter the Great introduce to westernize Russia?
(A) Peter the Great introduced political and military change through the recruitment of foreign
officers to run the navy.
(B) Peter the Great introduced economic change through strengthening mining industries and
using serfs in manufacturing.
(C) Peter the Great introduced cultural changes by renaming Russian landmarks after Western
models.
(D) Peter the Great introduced religious change by promoting Islam throughout Russia.

Answer: B
Topic: Rulers legitimize power; Rulers consolidate power over groups and populations
Theme: State Building, Expansion, and Conflict
Learning Objective: SB–1; SB–10
Historical Thinking Skill: Analyzing Evidence: Content and Sourcing
Key Concept: 4.3.I.A

4.43. Of what is St. Basil’s Cathedral in Moscow an example?


(A) the syncretization of belief systems across Russia.
(B) the use of religious ideas, art, and monumental architecture to legitimize the position of
rulers
(C) the use of paid labor to create state monuments as symbols of modernization
(D) the diffusion of European culture into Russia and East Asia

Answer: B
Topic: Monumental architecture and urban design
Theme: State Building, Expansion, and Conflict
Learning Objective: SB–5
Historical Thinking Skill: Synthesis
Key Concept: 4.3.I.A

138
Copyright © 2017, 2015, 2011, 2007 Pearson Education, Inc. All rights reserved.
Questions 4.44–4.45 refer to the map and images below.

Major Political Units of the World, c. 1750

Door Panels, Florence Baptistery, Italy

139
Copyright © 2017, 2015, 2011, 2007 Pearson Education, Inc. All rights reserved.
The Palace of Versailles

4.44. Name a result of increased oceanic trade and rising profits for merchants and monarchs.
(A) Monarchs gained great wealth through taxation and mercantilism and produced
monumental art and architecture.
(B) Serfdom ended in Europe, as peasants earned more money and gained equal status with
nobles.
(C) The practice of mercantilism ended in Europe but continued in South Asian and East Asian
countries.
(D) Monarchs continued to share power with the nobility and taxation ended in Europe.

Answer: A
Topic: Globalizing culture and political and economic effects
Theme: State Building, Expansion, and Conflict
Learning Objective: SB–9
Historical Thinking Skill: Analyzing Evidence: Content and Sourcing
Key Concept: 4.1.VII

4.45. Which of the following political and economic elites confronted new challenges to their ability to
affect polices of increasingly powerful monarchs and leaders?
(A) China and Japan welcomed European traders and encouraged the settlement of new
colonies to challenge the power of Confucian scholars.
(B) Zamindars in the Mughal empire and the daimyo in Japan faced challenges as their leaders
gained power and wealth.
(C) Boyars in Russia and the Amerindian nobles in the Americas faced few challenges as they
continued to accumulate more power and wealth.
(D) As a result of the rise of global trade, China and Indian elites gained superiority and power
over European trade networks.

140
Copyright © 2017, 2015, 2011, 2007 Pearson Education, Inc. All rights reserved.
Answer: B
Topic: Changing political and economic structures affected gender hierarchies
Theme: Development and Transformation of Social Structures
Learning Objective: SOC–1
Historical Thinking Skill: Causation
Key Concept: 4.2.III.B

4.46. Which statement best characterizes the pace of change that had occurred in Europe by the
eighteenth century?
(A) Europe changed slowly as it emerged from the Dark Ages into the eighteenth century.
(B) By the eighteenth century, European states had all become constitutional monarchies and
had agreed to a balance of power between nation-states.
(C) Continued contact with Russia had resulted in the diffusion of Russian culture and ideas into
Europe by the eighteenth century.
(D) Literacy had expanded in Europe by the eighteenth century, and governments had begun
funding programs for the visual and performing arts.

Answer: D
Topic: Spread of Literacy; Funding and expansion of the arts
Theme: Development and Interaction of Cultures
Learning Objective: CUL–8; CUL–9
Historical Thinking Skill: Causation, Synthesis
Key Concept: 4.1.VII

4.47. Which statement about gender, family, or demographic restructuring that occurred during this
period is false?
(A) Demographic changes in Africa resulted from the slave trade.
(B) Amerindian populations moved east to escape the European settlement.
(C) European families tended to become smaller.
(D) European men depended on women in Southeast Asia when conducting trade in that
region.

Answer: B
Topic: Changing political and economic structures affected gender hierarchies
Theme: Development and Transformation of Social Structures
Learning Objective: SOC–1
Historical Thinking Skill: Periodization
Key Concept: 4.2.III.C

141
Copyright © 2017, 2015, 2011, 2007 Pearson Education, Inc. All rights reserved.
Section II
Short-Answer Questions
4.48. Use the image and map below and your knowledge of world history to answer all parts of the
question that follows.

French Engraving, Haitian Sugar Plantation, Eighteenth Century

142
Copyright © 2017, 2015, 2011, 2007 Pearson Education, Inc. All rights reserved.
The Atlantic Slave Trade in Numbers

(A) How did agriculture change between 1450 and 1750?


(B) What caused the dramatic expansion of the Atlantic slave trade?
(C) What role did the production of sugar play in triangular trade?

Sample Answers

(A) Student responses might include the following observations:


Demographic growth was restored with the introduction of American food crops (i.e.,
potatoes, tomatoes, maize) throughout the Eastern Hemisphere (Columbian
Exchange), which led to increased agriculture in areas.
Farmers adapted to the Little Ice Age by introducing new crops and farming methods.
The Columbian Exchange led to new ways of interaction with the environment.
Increased demand for cash crops led to new forms of coerced labor.

(B) The rise of the plantation economy (sugar, cotton, tobacco) resulted in the increased
demand for slaves. Students might also discuss that the demand for slaves increased

143
Copyright © 2017, 2015, 2011, 2007 Pearson Education, Inc. All rights reserved.
because Amerindians, who died as a result of their lack of immunity to European diseases or
who ran away, were in limited supply to do the plantation work.

(C) Overall, as the production of sugar increased, the demand for African slaves increased.
Sugar production was very labor intensive. Sugar production was the third leg of the
triangular trade network. Sugar was shipped as molasses and was distilled into rum. In
Europe or New England, profits from trade in molasses and rum were used to purchase
goods. American and Europeans traders went to Africa with trade goods on the first leg of
the triangular network to obtain slaves. Slaves were sent across the Atlantic, in the second
leg, or Middle Passage. In the Americas, slaves were used to produce more sugar, and the
cycle continued.

Topic: Plantations; Atlantic world; African, American, European interactions


Theme: Creation, Expansion, and Interaction of Economic Systems; Development and Transformation of
Social Structures
Learning Objective: ECON–1; SOC–4
Historical Thinking Skill: Analyzing Evidence: Content and Sourcing; Causation; Synthesis
Key Concept: 4.2.II.B; 4.1.IV.D; 4.1.V.A; 4.1.V.B

144
Copyright © 2017, 2015, 2011, 2007 Pearson Education, Inc. All rights reserved.
4.49. Use the image and passages below and your knowledge of world history to answer all parts of the
question that follows.

Hanging Scroll Depicting “A Meeting of Japan, China and


the West”

145
Copyright © 2017, 2015, 2011, 2007 Pearson Education, Inc. All rights reserved.
SOURCE: William Hunter, The “Fan kwae” at Canton Before Treaty Days, 1825–1844 (London: Kegan, Paul, Trench & Co:1882),
28–30.

Regulation 1 – All vessels of war are prohibited from entering the Bogue. Vessels of war acting as convoy
to merchantmen must anchor outside at sea till their merchant-ships are ready to depart, and then sail
away with them.

Regulation 3 – All river-pilots and ships’ Compradores must be registered at the office of the assistant
magistrate [Tongzhi] at Macao. . . . All other boatmen and people must not have communication with
foreigners, unless under the immediate control of the ships’ Compradores; and should smuggling take
place, the Compradore of the ship engaged in it will be punished.

Regulation 5 – Foreigners are prohibited from rowing about the river in their own boats.

Regulation 8 – Foreign ships arriving with merchandise must not loiter about outside the river; they
must come directly to Huangpu. They must not rove about the bays at pleasure and sell to rascally
natives goods subject to duty, that these may smuggle them, and thereby defraud His Celestial
Majesty’s revenue.

Rules Regulating Foreign Trading in Guangzhou

SOURCE: N. Manucci, “An Excellent Treatise of the Kingdome of China,” in Richard Hakluyt, ed., The Principal Navigations
(London: Bishop, Newberie, and Barker, 1598–1600), 2: 88–97.

NOTE: The manuscript was written in Macao and captured by the English on its way to Lisbon.

There are such a number of artificers ingeniously framing sundry devices out of gold, silver, and other
metals . . . and other matters convenient for man’s use, that the streets of cities being replenished with
their shops and fine workmanship are very wonderful to behold. . . . Their industry does not less appear
in founding of guns. . . . To these may be added the art of printing . . . and with marvelous facility they
daily publish huge multitudes of books. . . . You may add two more, that is to say navigation and
discipline of war, both of which have been in ancient times most diligently practiced. . . . The people of
China do above all things profess the art of literature, and learning it most diligently, they employ
themselves a long time and the better part of their age therein. . . . Graduates of the second degree are
elected in each province, and a certain number . . . ascend to the highest pitch of dignity. . . . Out of this
order the chief magistrates are chosen. . . .

“An Excellent Treatise of the Kingdome of China”

(A) Why were European merchants and missionaries interested in relations with China and
establishing bases there?
(B) Analyze China’s attitude towards the West.
(C) What do these images and passages convey regarding the Chinese and Japanese attitude
towards the West and the global exchange of ideas?

146
Copyright © 2017, 2015, 2011, 2007 Pearson Education, Inc. All rights reserved.
Sample Answers

(A) Students might include the following observations in their answers:


Merchants were trying to establish new trade route due to the fact that trade along the Silk
Roads was disrupted after the Mongol Empire collapsed and the Ottomans captured
Constantinople.
Merchants wanted to trade items such as Chinese silk, tea, porcelain, and spices.
Spices such as ginger and Indian pepper were in great demand by the European elite.
Missionaries were attempting to convert Chinese people to their faith and to gather
information about them.

(B) Students might include the following observations in their answers:


Chinese looked at Westerners as barbarians and set rules regarding their access to China.
Western nations had taken over lands along the coast of the Indian Ocean, and the Chinese
felt that Europeans could not be trusted.
After the Philippines were colonized, the Chinese were even more distrustful of Westerners.
Europeans were allowed to trade for one season in Macao, near Canton. After the season
ended, they were required to sail away.
The Chinese were interested in some of the technology, such as clocks, that the Jesuits
brought as gifts to China, but for the most part the Chinese regarded European
products as inferior.
The Chinese distrusted Jesuit missionaries as well as merchants. (Although it should be
noted that Francis Xavier and the Jesuits made substantial conversions in China,
particularly in Shanghai.)

(C) Students might include the following observations in their answers:


At first the Japanese welcomed European traders and the goods that they brought from
across the Indian Ocean.
As more European traders and missionaries arrived, European technologies, such as the
printing press and firearms, were quickly improved upon by the Japanese. Within a
short time, however, the success of European traders and missionaries was
considered a threat by the elite.
The new religion (Christianity) was considered to be a threat to Japanese customs and
beliefs, and Christians began to be persecuted and exiled.
The Japanese also felt threatened by the takeover of the Philippines by the Spanish.
The Japanese were interested in “Dutch learning,” such as astronomy. In the accompanying
scroll image, the position of the Japanese man suggests his interest in Western
science, an area in which the Japanese acknowledged Western superiority.
The Japanese virtually closed their “doors” and focused on their own culture after this time.
As Japan modernized, the samurai began to lead revolts, as they lost their place in society.

Topic: Trading post empires; Competition over trade routes; Effects on regional markets
Theme: State Building, Expansion, and Conflict; Creation, Expansion, and Interaction of Economic
Systems
Learning Objective: SB–1; SB–9; ECON–13
Historical Thinking Skill: Contextualization; Argumentation; Interpretation
Key Concept: 4.1.III; 4.1.IV A; 4.2.II.A; 4.2.III.B; 4.3.III

147
Copyright © 2017, 2015, 2011, 2007 Pearson Education, Inc. All rights reserved.
Section III
Long Essay Question
Question 4.50: Using specific examples, compare the political and cultural structure of these two
empires in the period from c. 1450 to c. 1750.

Ottoman Empire ————— Safavid Empire

Sample Answer

This question is asking students to compare the political and cultural structures of the Ottoman and the
Safavid empires and to provide evidence in support of their thesis. Students should understand that
compare also means contrast and that they are required to provide evidence and analysis for both
similarities and differences, but not necessarily evenly.

To gain the thesis point, students must address all parts of the question, qualifying comparisons and
contrasts within the given timeframe. Qualifying categories for comparison and contrast are bother
political and cultural. A strong essay will not only provide evidence of comparison of both political and
cultural structures but also include analysis of impact relative to the similarities and differences.

In this question, students can demonstrate the targeted historical thinking skill of comparison by
providing specific points of comparison and contrast between the two empires. Students must describe
similarities and differences related to political and cultural structures while analyzing the reasons for the
similarities and/or differences cited. Students must evaluate the significance of historical developments
to gain two points.

Two points can be gained through extensive analysis of evidence (content and sourcing), citing a
minimum of five pieces of evidence in support of their thesis and establishing clear linkages between
the evidence cited and their thesis. Students must draw direct comparisons and direct contrasts.

The Ottoman and Safavid were both gunpowder empires with Turkish ruling dynasties that withstood
powerful nomadic invasions and enlarged their territories. Politically, both empires followed steppe
traditions, which conferred autocratic power on the ruler. Because no clear successor was named, intra-
family conflicts and competition for power followed the deaths of rulers. Princes fought to become heir,
and political instability caused by family controversies always threatened the empires. To prevent coups
sons were kept in the palace, which led to sheltered and less competent rulers. As a consequence,
instability threatened the empires. Both had large centralized bureaucracies, with a vizier in the
Ottoman Empire carrying out administration. In order to rule their diverse populations, the Ottomans
instituted the millet system, under which autonomous religious communities retained their own laws
and identifies. Both empires instituted a jizya, or head tax.

In the Safavid Empire, Persians were recruited for positions at court, and struggles for power between
the Turkish and Persian notables ensued. Over time both empires weakened, as the empires did not
continue to expand nor gain new sources of wealth and resources from expansion. Neither empire
embraced Western technology, which was also a factor in their decline. The Wahhabi movement led to

148
Copyright © 2017, 2015, 2011, 2007 Pearson Education, Inc. All rights reserved.
the demolition of an astronomical observatory under construction and the closure of the Ottoman
printing press.

The Ottomans were Sunni Muslims, which led to political rivalries with the Shi’ite Safavids; but both had
connections to the Sufi religious order. Devotion to Islam motivated both the Ottomans and the Safavids
to conquer new lands to spread their faith. Safavid followers were known as the qizilbash, meaning red
hats or heads, who wore red caps to signify their support for the Safavid.

The Ottomans instituted the practice of devshirme, the converting of sons of Christians to Islam and
training them to serve the government as civilian administrators or in the military as Janissaries. At its
height, the empire of the Ottomans had a powerful navy, in contrast to the Safavid where sea vessels
were privately owned. However, after the Ottoman defeat by the Spanish and Venetians (at the Battle
of Lepanto), the Ottomans lost their sea supremacy, signaling the end of the “Ottoman Lake” (the
Mediterranean). The Sunni Ottomans and the Shi’ite Safavids clashed at the Battle of Chaldiran, where
the Ottomans emerged victorious. This defeat limited the ability of the Safavid to expand further, also a
factor in their decline.

Culturally, both empires were patriarchal societies. The wives of elite had influence and followed the
custom of the purdah and veiling the women. Both empires encouraged the production of miniature
paintings and rugs. Both encouraged the arts and unique artistic styles developed. Persian calligraphy
was emphasized in the empires. The Ottomans spoke Turkish whereas the Safavids spoke Persian. While
both excelled at architecture, the Safavids were superior in art and architecture.

There are many ways the synthesis point can be earned. Students may earn the synthesis point, one
point, by extending the argument to a different historical period, a different discipline of inquiry, or
another course theme (such as economic, social, cultural, or intellectual history). Students may also
discuss connections to geography or to anthropology to support their thesis in order to earn the
synthesis point. This question offers the student many options.

Topic: Rulers legitimize power; Gunpowder empires; State rivalries; State consolidation and imperial
expansion; Rulers consolidate power over groups and populations; Imperial expansion;
Theme: State Building, Expansion, and Conflict; Creation, Expansion, and Interaction of Economic
Systems; Development and Transformation of Social Structures.
Learning Objective: SB–1; SB–2; SB–4; SB–10; ECON–3;, ECON–12; SOC–3
Historical Thinking Skill: Analyzing Evidence: Content and Sourcing; Causation; Comparison; Synthesis
Key Concept: 4.1.VI; 4.3.I.A; 4.3.I.B; 4.3.I.C; 4.3.II.B

149
Copyright © 2017, 2015, 2011, 2007 Pearson Education, Inc. All rights reserved.
Question 4.51: Compare demographic and environmental effects of the Columbian Exchange on the
Americas with the demographic and environmental effects of the Columbian Exchange on one of the
following regions between 1492 and 1750.

Asia—————Europe

Sample Answer

This question is asking students to compare the demographic and environmental effects of the
Columbian Exchange on the Americas with Asia or Europe and to provide evidence to support their
thesis. Students should understand that compare also means contrast and that they are required to
provide evidence and analysis for both, but not necessarily evenly.

To gain the thesis point the student must address all parts of the question by qualifying comparisons and
contrasts within the given timeframe. Qualifying categories for comparison and contrast are
demographic and environmental effects as evidence. A strong essay will not only provide evidence of
comparison of demographic and environmental effects but also include analysis of the impacts relative
to the similarities and differences.

In this question, students demonstrate the targeted historical thinking skill of comparison by providing
specific evidence as they compare and contrast the two regions. Students must describe similarities and
differences among historical developments relating to the demographic and environmental effects while
analyzing the reasons for these similarities and/or differences. Students must evaluate the significance
of the effects to gain two points.

Two points can be gained through extensive analysis of evidence (content and sourcing), citing a
minimum of eight pieces of evidence in support of their thesis and establishing clear linkages between
the evidence cited and their thesis. Students must draw direct comparisons and direct contrasts.

When the Europeans arrived in the Americas, the indigenous peoples did not have inherited or acquired
immunity to their diseases, such as small pox, measles, diphtheria, whooping cough, and influenza. As a
result, the demographic effects of the Columbian Exchange in the Americas were staggering, as millions
died (commonly referred to as the “great dying”). Estimates encompass the loss of 90 percent of the
Aztec population alone. The Columbian Exchange is regarded as the worst demographic calamity in
world history. In Asia, diseases were endemic; while some children and infants died, survivors gained
immunity. In Europe, diseases predominantly affected children; while 10 to 15 percent died, diseases
also did not pose an existential threat to society because adults, who were responsible for the
economic, political, and social organization of European nation-states, were not affected to the degree
that adult populations in the Americas were affected.

On the other hand, over time, with the exchange of crops and animals, world population rose. New
foods, such as wheat, rice, sugar, cherries, apples, and citrus fruits, were introduced. Land was cleared
for crops and wheat was successfully grown across the plains in North and South America. In the Eastern
Hemisphere, crops such as maize and potatoes contributed to diets, grew well in places not suitable for
the cultivation of rice or millet, and supplemented traditional rice and wheat dishes to sustain China’s
population explosion. Tomatoes, peppers, peanuts, and manioc also arrived in Eurasia from the
Americas. Quinine, made from American medicinal plants, proved effective in treating malaria. Beans
also added protein to the diets of Eurasians. Potatoes were not as popular in China as in Europe. In

150
Copyright © 2017, 2015, 2011, 2007 Pearson Education, Inc. All rights reserved.
China, many associated potatoes with peasants. Later the Chinese recognized that the crop, being
concealed in the ground, was an important source of food that plundering troops could not seize during
war times. Meat-yielding and dairy animals, such as horses, cattle, pigs, sheep, chickens, and goats,
brought to the Americas provided food and sources of energy for labor. However, because of a lack of
natural predators, the animals multiplied rapidly and impacted the ecological balance. Entire landscapes
were destroyed with their hooves and foraging. Land was cleared for cattle ranches and pastures by
Spanish settlers in the Americas.

As a result of the Columbian Exchange, world population rose due to a steady and varied diet of new
foods. In Europe, the mortality rate declined and, in combination with the increase in births, the
population rose rapidly. The Columbian Exchange also included forced and voluntary transoceanic
migrations, thus spreading the human population. The demographics of the Americas were affected by
the importation of millions of slaves from Africa. Most Spanish migrants and slaves were men. Because
of the gender imbalance that ensued, Spanish men consorted with local women and with African slaves.
New social classes emerged from these unions. Migration to the Americas from across Eurasia occurred
as migrants claimed the lands depopulated by diseases. As cities grew in the Americas, in Europe, and in
Asia, the environment was changed by the clearing of forests for lumber as well as by pollution, which
affected the habitat of large-sized mammals and birds. Over time, the American flora and fauna began
to take on more of a European appearance.

In the Americas, beavers were hunted to near extinction to satisfy the demand for furs by consumers in
China and Europe. Similarly, deer, codfish, whales, and seals were hunted for their skins, food, oil and
ivory. Sable pelts from Siberia were in great demand also. As population increased, the demand for food
increased, and more land was cleared for wheat and sugar, leading to environmental problems. In
addition, the lure of wealth led to increased silver mining in Mexico and in Peru and to the degradation
of the environment.

There are many ways the synthesis point can be earned. Students may earn the synthesis point, one
point, by extending the argument to a different historical period, a different discipline of inquiry, or
another course theme (such as economic, social, cultural, or intellectual history). Students may also
discuss connections to geography or anthropology to support their thesis in order to earn the synthesis
point. This question offers the student many options.

Topic: Intensification of existing trade routes; New maritime routes; Atlantic world; Colonization;
Columbian Exchange; Changing political and economic structures affected gender hierarchies;
Restructuring of hierarchies
Theme: Creation, Expansion, and Interaction of Economic Systems; Development and Transformation of
Social Structures; Interaction Between Humans and the Environment
Learning Objective: ECON–12; SOC–1; SOC–2; SOC–4; ENV–5; ENV–7; ENV–8
Historical Thinking Skill: Comparison; Contextualization; Synthesis; Analyzing Evidence: Content and
Sourcing
Key Concept: 4.1.III.C; 4.1.IV.D; 4.1.V.B; 4.1.V.C; 4.1.V.D; 4.1.V.E; 4.2.III.A; 4.2.III.C

151
Copyright © 2017, 2015, 2011, 2007 Pearson Education, Inc. All rights reserved.
Section IV
Document-Based Questions
Directions: The following question is based on the accompanying documents. The documents have been
edited for the purpose of this exercise.

In your response, you should do the following:

• State a relevant thesis that directly addresses all parts of the question.

• Support the thesis or a relevant argument with evidence from all, or all but one, of the
documents.

• Source and analyze the significance of at least four of the documents on the following basis:
intended audience, author’s purpose, historical context, format or medium, and/or point of view.

• Develop context by relating your argument to broader historical events or processes.

• Synthesize the elements above into a persuasive essay that extends your argument by connecting
it to:

• a different historical period or geographical region


• another course theme or historical approach that is not the focus of the essay (such as
political, social, economic, or intellectual history)
• another perspective from a different academic discipline (such as economics, art history,
anthropology, or government)

152
Copyright © 2017, 2015, 2011, 2007 Pearson Education, Inc. All rights reserved.
Question 4.52: Using the documents and your knowledge of world history, analyze the social and
economic effects of the global flow of silver from the mid-sixteenth century to the early eighteenth
century.

Historical Background: Spanish colonial America and Tokugawa Japan led the world in silver production
from 1500 to 1750. In the early 1570s, the Ming Chinese government required that all domestic taxes
and trade fees be paid in silver.

Document 1
Global Flow of Silver

Document 2
Ye Chunji (County Official During the Ming Dynasty), Order Issued to Limit Wedding Expenses, 1570s
The frugal man with only one bar of silver currency can have something left over, whereas the
extravagant man with a thousand can still not have enough.

Document 3
Tomás de Mercado (Spanish Scholar), Manual of Deals and Contracts, Seville, 1571
High prices ruined Spain as the prices attracted Asian commodities and the silver currency flowed out to
pay for them. The streets of Manila in the Spanish territory of the Philippines could be paved with
granite cobblestones brought from China as ballast* in Chinese ships coming to get silver for China.

*A heavy substance used to improve the stability of a ship

153
Copyright © 2017, 2015, 2011, 2007 Pearson Education, Inc. All rights reserved.
Document 4
Wang Xijue (Ming Dynasty Court Official), Report to the Emperor, 1593
The venerable elders of my home district explain that the reason grain is cheap despite poor harvests in
recent years is due entirely to the scarcity of silver coin. The national government requires silver for
taxes but disburses little silver in its expenditures. As the price of grain falls, tillers of the soil receive
lower returns on their labors, and thus less land is put into cultivation.

Document 5
Ralph Fitch (British Merchant), An Account of His Travels to the East Indies, Published in 1599
When the Portuguese go from Macao, the most southern port city in China, to Japan, they carry much
white silk, gold, perfume, and porcelain and they bring from Japan nothing but silver. They have a great
ship that goes to Japan every year, and brings back more than 600,000 coins’ worth of Japanese silver.
The Portuguese use this Japanese silver to their great advantage in China. The Portuguese bring from
China gold, perfume, silk, copper, porcelain, and many other luxury goods.

Document 6
Xu Dunqiu Ming (Writer) The Changing Times(An Essay About the Commercial City of Hangzhou), 1610
In the past, the dye shops would allow customers to have several dozen pieces of cloth dyed before
settling accounts and charging the customers. Moreover, customers could pay for dying the cloth with
rice, wheat, soybeans, chickens, or other fowl. Now, when you have your cloth dyed you receive a bill,
which must be paid with silver obtained from a moneylender.

Document 7
He Qiaoyuan (Ming Dynasty Court Official), Report to the Emperor (On the Possibility of
Repealing the 1626 Ban on Foreign Trade), 1630
The Spanish have silver mountains, which they mint into silver coins. When Chinese merchants trade in
Southeast Asia and the Indian Ocean, they trade the goods we produce for the goods of others. But
when they go to Luzon (Philippines) they only return with silver coins. Chinese silk yarn worth 100 bars
of silver can be sold in the Philippines at a price of 200 to 300 bars of silver there. Moreover, porcelain
from the official pottery works as well as sugar and fruit from my native province, are currently desired
by the foreigners.

154
Copyright © 2017, 2015, 2011, 2007 Pearson Education, Inc. All rights reserved.
Document 8
Charles D’Avenant (English Scholar), “An Essay on the East-India Trade” (Regarding the Debate on a Bill in Parliament
to Restrict Indian Textiles), 1697
Since we were supplanted in the spice-trade by the Dutch, our chief investments or importations from
the East Indies have been in dyed cotton cloth, silks, drugs, cotton-yarn, and wool; part of which
commodities are for our own use, but a much greater part, in times of peace, were brought to London
for sale to France, Germany, the Netherlands, Spain, Italy, and our colonies.

For Europe draws from Asia nothing of solid use; only materials to supply luxury, and only perishable
commodities, but sends to Asia gold and silver, which is there buried and never returns.

But since Europe has tasted of this luxury, since the custom of a hundred years has made Asian spices
seem necessary to all degrees of people, since Asian silks are pleasing everywhere to the better sort,
and since their dyed cotton cloth is useful wear at home, and in our own colonies, and for the Spaniards
in America, it can never be advisable for England to quit this trade, and leave it to any other nation.

Sample Answer

An acceptable thesis would consist of one or more congruent sentences in the opening paragraph that
would present a plausible analysis of the social and economic effects of the global flow of silver from the
mid-sixteenth century to the early eighteenth century. For example, students might discuss that, in
China, the increased flow of silver led to negative social and economic effects whereas, in Europe,
European economies and societies benefited from the increase in luxury goods. An excellent thesis
would include information concerning (1) the integration of Europeans in the globalization of trade and
the economic opportunities that arose therefrom and (2) the creation of growing social divisions within
China. Students might also mention that the flow of silver eventually weakened states.

To achieve all four points in the evidence category of the rubric, students need to relate at least six of
the documents to their thesis and to analyze each document by addressing either historical context,
audience, purpose, or the point of view of each document. It is always wise to incorporate all
documents and to relate evidence back to the thesis (1) by addressing the intended audience or the
document author’s point of view or purpose or (2) by placing documents in the context of what was
happening. For instance, the author of Document 2 seems to feel that people should be more frugal and
possibly represents a Confucian point of view; the audience for Document 2 would be the people in
China. Document 3 has a Spanish point of view, and the author’s tone is resentful owing to silver flowing
from Spain to China. Document 4 may be used to discuss the decline of traditional Chinese society and
the economy. Students may characterize Document 4, similar to Documents 2 and 6, as a representation
of a Confucian point of view. Document 5, an account from a British merchant, has mercantilist tone and
mentions Portugal’s trade advantages. Students might pair Document 5 with Document 8. Document 6
may be used as evidence to explain the decline of traditional Chinese society and of the economy
overall. Document 7 has an interesting point of view. The speaker’s motives are questionable, because
goods from his native province are in demand and, if the ban is lifted, his region stands to benefit
economically. Students might also use this document to demonstrate a sophisticated analysis of change
over time, as evidence that China is moving away from traditional ideas about economy and trade.
Document 8, written by and Englishman, might lead students to comment that he is a scholar or that, as
an Englishman, he would favor an English trade advantage in the region to support England’s own
economic gain.

155
Copyright © 2017, 2015, 2011, 2007 Pearson Education, Inc. All rights reserved.
There are plenty of outside examples students could use to support their thesis. Students might mention
the advancements that Europeans enjoyed in technology and the speed at which global networks
moved. Students might discuss silver mining in the Americas in relation to the exploitation of indigenous
peoples through coerced labor. A couple of ways to gain the contextualization point: (1)to examine the
role of the Columbian Exchange in relation to global trade networks or (2) to examine the increase in
coerced labor necessary to produce silver and luxury goods in China, which were in great demand.

The synthesis point could be gained by discussing the impact of global networks of trade on the
Americas or on another region into the next century. Students might also reference a discipline beyond
history, such as economics, and relate supply and demand to their analysis of the flow of silver.

Topic: Mercantilism; Joint-stock companies; Transoceanic shipping; Atlantic world; Merchants;


Globalizing networks and their political and economic effects
Theme: Creation, Expansion, and Interaction of Economic Systems; State Building, Expansion, and
Conflict; Development and Transformation of Social Structures
Learning Objective: ECON–3; ECON–9; ECON–11; ECON–12; ECON–13; SB–3; SB–9; SOC–2; SOC–7
Historical Thinking Skill: Analyzing Evidence: Content and Sourcing; Contextualization
Key Concept: 4.1.IV.A; 4.1.IV.B; 4.1.IV.C; 4.1.IV.D

156
Copyright © 2017, 2015, 2011, 2007 Pearson Education, Inc. All rights reserved.
Question 4.53: Using the documents and your knowledge of world history, analyze the European view of
the inhabitants of the New World and how European knowledge of New World inhabitants shaped the
way in which Europeans viewed themselves.

Historical Background: The arrival of Columbus in Hispaniola in 1492 would change the Americas and
patterns of global interaction. Spanish conquistadores, missionaries, and merchants were quick to follow
Columbus across the Atlantic and establish contact with the indigenous peoples.

Document 1
Henricus Martellus (German Cartographer), World Map, c. 1489
What Columbus knew of the world in 1492 is represented in this map of Henricus Martellus (c. 1489),
copies of which are believed to have reached Christopher Columbus in Spain. After he left Europe,
sailing west across the Atlantic, Columbus expected Asia to be his first major landfall.

157
Copyright © 2017, 2015, 2011, 2007 Pearson Education, Inc. All rights reserved.
Document 2
Diego Duran (Spanish Artist), Hernando Cortés Besieged by Aztec Warriors, Sixteenth Century
Armored Spanish soldiers, under the command of Pedro de Alvarado (d. 1541) and bearing crossbows,
engage unprotected and crudely armed Aztecs, who are nonetheless portrayed as larger than life by
Spanish artist Diego Duran (sixteenth century).

Document 3
Michel de Montaigne (Author and Philosopher), Reflections on the Nature of “Barbarism,” 1575
SOURCE: Montaigne, author and philosopher, reflects on the nature of “barbarism,” The Complete Essays of
Montaigne.

Now, to return to my subject, I think there is nothing barbarous and savage in that nation [Brazil], from
what I have been told. . . . Each man calls barbarism whatever is not in his own practice; for indeed it
seems we have no other test of truth and reason than the example and pattern of the opinions and
customs of the country we live in. There [we] always [find] the perfect religion, the perfect government,
the perfect and accomplished manners in all things. Those [foreign] people are wild, just as we call the
fruits that Nature has produced by herself and in her normal course; where really it is those that we
have changed artificially and led astray from the common order that we should call wild. The former
retain alive and vigorous their genuine virtues and proprieties, which we debased the latter by adapting
them to gratify our corrupted taste. . . . It is not reasonable that [our human] art should win the place of
honor over our great and powerful mother nature.

158
Copyright © 2017, 2015, 2011, 2007 Pearson Education, Inc. All rights reserved.
Document 4
Christopher Columbus, Description of His First Voyage to the New World
The people of this island and of all the other islands which I have found and of which I have information,
all go naked, men and women, as their mothers bore them. . . . They have no iron or steel or weapons,
nor are they fitted to use them. This is not because they are not well built and of handsome stature, but
because they are very marvelously timorous. . . . They refuse nothing that they possess, if it be asked of
them; on the contrary, they invite any one to share it and display as much love as if they would give
their hearts. . . . They do not hold any creed nor are they idolaters; but they all believe that power and
good are in the heavens and were very firmly convinced that I, with these ships and men, came from the
heavens.

Document 5
Bartolomé de las Casas (Dominican Friar), Description of the Inhabitants of the New World
God has created all these numberless people to be quite the simplest, without malice or duplicity, most
obedient, most faithful to their natural lord, and to the Christians, whom they serve; the most humble,
most patient, most peaceful, and calm, without strife nor tumults; not wrangling, nor querulous, as free
from uproar, hate and desire of revenge, as any in the world. . . .

They are likewise of a clean, unspoiled, and vivacious intellect, very capable, and receptive to every
good doctrine; most prompt to accept our Holy Catholic Faith, to be endowed with virtuous customs;
and they have as little difficulty with such things as any people created by God in the world.

Document 6
Jacques Cartier, First Contact with the Indians, 1534
NOTE: Jacques Cartier, landing at Cape Gaspé while on a mission for King Frances I in 1534, was responsible for France’s early
possession of Canada.

The fourth of July we went along the coast of the said land on the Northerly side to find some
harborough, where wee entred into a creek altogether open toward the South, where there is no
succour against the wind: we thought good to name it S. Martines Creeke. . . . [W]ee sawe two
companies of boates of wilde men going from one land to the other: their boates were in number about
fourtie or fiftie. One part of the which came to the said point, and a great number of men went on shore
making a great noise, beckening unto us that wee should come on land, shewing us certaine skinnes
upon pieces of wood, but because we had but one onely boat, wee would not goe to them, but went to
the other side lying in the See: they seeing us flee, prepared two of their boats to follow us, with which
came also five more of them that were comming from the Sea side, all which approched neere unto our
boate, dancing, and making many signes of joy and mirth, as it were desiring our friendship, saying in
their tongue . . . with many other words that we understood not. . . . [We] made signes unto them that
they should turne back, which they would not do, but with great furie came toward us: and suddenly
with their boates compassed us about: and because they would not away from us by any signes that we
could make, we shot off two pieces among them, which did so terrifie them, that they put themselves to
flight toward the sayde point, making a great noise: and having staid a while, they began anew, even as
at the first to come to us againe, and being come neere our boat wee strucke at them with two lances,
which thing was so great a terrour unto them, that with great haste they beganne to flee, and would no
more follow us.

159
Copyright © 2017, 2015, 2011, 2007 Pearson Education, Inc. All rights reserved.
Sample Answer

An acceptable thesis would consist of one or more congruent sentences in the opening paragraph that
would present a plausible analysis of the way that Europeans viewed the inhabitants of the New World
and how their knowledge shaped the way that Europeans viewed themselves. Students should organize
the documents around the European view of the indigenous peoples and of the European view of
themselves. For instance, students might discuss European ideas of the indigenous peoples as
barbarians and savage as opposed to European benevolence, sophistication, and superiority. An
excellent thesis would include sophisticated analysis relating to the tendency of European authors to see
the inhabitants of the New World through the lens of preconceived ideas. Students, for example, may
discuss the European concept of manifest destiny as a justification for claiming lands in the New World.

To achieve all four points in the evidence category of the rubric, students need to relate at least five of
the documents to the thesis and to analyze each document by addressing historical context, audience,
purpose, or point of view. It is always wise to use all documents and to relate evidence back to the
thesis by addressing the intended audience or the author’s point of view or purpose or by placing the
document in the context of what was happening. For instance, the map displayed in Document 1 would
have been available to many sea captains, but Columbus determined that it mapped a faster route to
Asia. However, Columbus didn’t calculate distance correctly and thought he had made landfall in East
Asia. The Martellus map represents a European point of view. As seen in Document 2, Europeans
represented the Aztec through a lens that portrayed the Aztec as larger than life and aggressive. The
audience for the illustration would be Europeans and would set the tone for the manner in which the
peoples in the Americas would be perceived and treated.

On the other hand, the Europeans viewed themselves as superior, as evidenced in Document 3.
Students might also note that, in Documents 3, 4, and 5, there is tendency of Europeans to identify the
New World and its peoples with “nature.” There are also numerous responses to the peoples in the New
World that students can analyze in Documents 2, 3, 4, 5, and 6. In Document 4, an account from
Columbus, he seems to view the inhabitants as friendly and generous, but unfit and unattractive by
European standards. Students may wonder if Columbus portrayed the inhabitants as generous in order
to justify the wealth he took. He also portrays the inhabitants as godly, perhaps again to justify coerced
conversions to Christianity. Students may mention the fact that the pope supported the conversion of
New World inhabitants and the claiming their land for God. Document 5, written by a Dominican friar,
has the same point of view as Document 4, again seeing the indigenous people through a European lens.
Being a man of God and motivated to convert, Bartolomé de las Casas would, naturally, discuss faith and
godliness of the indigenous peoples. In Document 6, written by a Frenchman, the people are wild and
aggressive. In all of the documents, the Europeans view themselves as superior, civilized, and god-
fearing men. In addition, the Europeans see themselves as bringing God and civilizing ideas to the
Americas.

The synthesis point can be gained by discussing the impact of the Columbian Exchange on the
indigenous peoples or by analyzing European views in connection with the encomienda system, the mita
system, or another form of coerced labor. For example, students could inject economics into their
responses by discussing factors of production. Students could also mention the continuity of Western
exceptionalism, European feelings of superiority, and Church-approved exploration of lands worldwide.

Topic: Trading-post empires; Land-based and maritime expansion; Columbian Exchange


Theme: State Building, Expansion, and Conflict; Development and Transformation of Social Structures

160
Copyright © 2017, 2015, 2011, 2007 Pearson Education, Inc. All rights reserved.
Learning Objective: SB–1; SB–3; SB–9; SOC–7
Historical Thinking Skill: Analyzing Evidence: Content and Sourcing; Contextualization; Synthesis
Key Concept: 4.3.II.C; 4.I.V.A

161
Copyright © 2017, 2015, 2011, 2007 Pearson Education, Inc. All rights reserved.
Period 5
Industrialization and Global Integration
(c. 1750 to 1900)
Section I
Multiple-Choice Questions
Questions 5.1–5.3 refer to the passage below.

There is a destiny now possible to us, the highest ever set before a nation to be accepted or refused.
Will you youths of England make your country again a royal throne of kings, a sceptred isle, for all the
world a source of light, a centre of peace; mistress of learning and of the Arts, faithful guardian of time-
honoured principles? This is what England must do or perish: she must found colonies as fast and as far
as she is able, formed of her most energetic and worthiest men; seizing every piece of fruitful waste
ground she can set her feet on, and there teaching these her colonists that their chief virtue is to be
fidelity to their country, and their first aim is to advance the power of England by land and sea.

John Ruskin (British Critic), Inaugural Lecture, Oxford University, 1870

5.1. What action does Ruskin justify in the passage?


(A) imperialist action by European nations
(B) the pursuit of industrialization by western European states
(C) the overthrow of the British monarchy in favor of a constitutional government
(D) the establishment of artistic academies in England to rival those of the French

Answer: A
Topic: Imperialism
Theme: State Building, Expansion, and Conflict
Learning Objective: SB–2; SB–3
Historical Thinking Skill: Argumentation; Periodization
Key Concept: 5.2.I.B

5.2. All of the following are plausible motives for the imperialist ideas conveyed in Ruskin’s lecture
excerpt one. Which one?
(A) locating additional raw materials for British factories
(B) increasing the availability of international markets for European goods
(C) growing support for mercantilist policies among European governments
(D) the emerging racial ideologies of the nineteenth century

Answer: C
Topic: Imperialism and industrial revolution; Need for raw materials

162
Copyright © 2017, 2015, 2011, 2007 Pearson Education, Inc. All rights reserved.
Theme: Interaction Between Humans and Environment; State Building, Expansion, and Conflict;
Creation, Expansion, and Interaction of Economic Structures; Development and Transformation of Social
Structures
Learning Objective: ENV–9; SB–9; ECON–9; SOC–3
Historical Thinking Skill: Causation; Argumentation
Key Concept: 5.1.I; 5.1.II.A; 5.1.II.C; 5.2.III

5.3. Although Ruskin called for increased action, which of the following European states saw its
influence and involvement in imperialism decline?
(A) England
(B) Norway
(C) Japan
(D) Portugal

Answer: D
Topic: Formation of new nation states; Imperialism; Expansion of the global economy
Theme: State Building, Expansion, and Conflict; Creation, Expansion, and Interaction of Economic
Structures
Learning Objective: SB–1; SB–2; SB–3; ECON–3
Historical Thinking Skill: Causation; Periodization
Key Concept: 5.2.I.B

163
Copyright © 2017, 2015, 2011, 2007 Pearson Education, Inc. All rights reserved.
Questions 5.4–5.6 refer to the image below.

American Anti-Imperialist Cartoon

5.4. Which of the following was an indigenous response to the imperialist actions of England
lampooned in the cartoon?
(A) the Meiji Restoration
(B) the American Civil War
(C) the Sepoy revolt
(D) the Greek war for independence

Answer: C
Topic: Imperialism; Resistance to imperialism
Theme: State Building, Expansion, and Conflict; Development and Transformation of Social Structures,
Learning Objective: SB–1; SOC–7
Historical Thinking Skill: Analyzing Evidence: Content and Sourcing; Interpretation
Key Concept: 5.3.III.D

5.5. In what ways did British control of one region identified in the image impact other regions?
(A) Rebellions in one colonial region typically led to copycat rebellions in other colonies.
(B) England seized resources from colonies in Asia to promote industrialization in its African
colonies.

164
Copyright © 2017, 2015, 2011, 2007 Pearson Education, Inc. All rights reserved.
(C) As England gained control of new regions, it was forced to let older colonies go to avoid
overextension.
(D) England often used indigenous peoples from one colony as forces to conquer and control
other indigenous peoples in that colony or in nearby colonies.

Answer: D
Topic: Imperialism; Labor systems; Global migration
Theme: Creation, Expansion, and Interaction of Economic Structures; Development and Transformation
of Social Structures
Learning Objective: ECON–5; ECON–6; SOC–2; SOC–8
Historical Thinking Skill: Causation; Contextualization
Key Concept: 5.4.II.B

5.6. Why is Egypt included in the image?


(A) England hoped to develop a canal that would facilitate access to the Indian Ocean.
(B) England wanted to develop a self-sufficient textile industry in Egypt.
(C) Egypt threatened English control in the Middle East.
(D) Egypt petitioned the English for inclusion in its empire.

Answer: A
Topic: Industrialization; Imperialism; Trade and economic interactions
Theme: Interaction Between Humans and Environment; State Building, Expansion, and Conflict
Learning Objective: ENV–5; SB–4; SB–9
Historical Thinking Skill: Causation; Contextualization
Key Concept: 5.1.IV

165
Copyright © 2017, 2015, 2011, 2007 Pearson Education, Inc. All rights reserved.
Questions 5.7–5.9 refer to the passage below.

Take up the White Man’s Burden—


Send forth the best ye breed—
Go bind your sons to exile
To serve your captives’ need;
To wait in heavy harness
On fluttered folk and wild—
Your new-caught, sullen peoples,
Half devil and half child.

Rudyard Kipling, “The White Man’s Burden,” 1899

5.7. What ideology is illustrated in Kipling’s poem?


(A) socialism
(B) conservatism
(C) Social Darwinism
(D) communism

Answer: C
Topic: Imperialism; Ideologies of the nineteenth century
Theme: Development and Interaction of Cultures; State Building, Expansion, and Conflict; Creation,
Expansion, and Interaction of Economic Structures; Development and Transformation of Social
Structures
Learning Objective: CUL–3; SB–4; ECON–8; SOC–6
Historical Thinking Skill: Analyzing Evidence: Content and Sourcing; Causation; Contextualization
Key Concept: 5.2.III

5.8. The poem communicates justification for what type of political action?
(A) imperialism
(B) colonial rebellion
(C) urban reform
(D) enfranchisement of women

Answer: A
Topic: Imperialism
Theme: Development and Interaction of Cultures; State Building, Expansion, and Conflict; Creation,
Expansion, and Interaction of Economic Structures; Development and Transformation of Social
Structures
Learning Objective: CUL–3; SB–4; ECON–8; SOC–6
Historical Thinking Skill: Causation; Periodization
Key Concept: 5.2.III

5.9. The ideas in Kipling’s poem appear to contradict what other idea or ideas that emerged between
the years 1750 and 1900?
(A) socialism’s advocacy for improved conditions for workers
(B) Herbert Spencer’s ideas concerning Social Darwinism

166
Copyright © 2017, 2015, 2011, 2007 Pearson Education, Inc. All rights reserved.
(C) Charles Darwin’s ideas concerning evolutionary biology
(D) Enlightenment ideas about natural rights

Answer: D
Topic: Ideologies – Social Darwinism
Theme: Development and Interaction of Cultures; State Building, Expansion, and Conflict; Development
and Transformation of Social Structures
Learning Objective: CUL–3; SB–4; SOC–6; SOC–7
Historical Thinking Skill: Patterns of Continuity and Change over Time; Periodization
Key Concept: 5.3.I.A

167
Copyright © 2017, 2015, 2011, 2007 Pearson Education, Inc. All rights reserved.
Questions 5.10–5.11 refer to the passage below.

Manifesto of the Society for German Colonization, April 1884

In the partition of the earth, as it has proceeded from the beginning of the fifteenth century up to our
times, the German nation received nothing. All the remaining European culture-bearing peoples possess
areas outside our continent where their languages and customs can take firm root and flourish. The
moment that the German emigrant leaves the borders of the Reich behind him, he is a stranger
sojourning on foreign soil. The German Reich, great in size and strength through its bloodily achieved
unity, stands in the leading position among the continental European powers: her sons abroad must
adapt themselves to nations which look upon us with either indifference or even hostility. For centuries
the great stream of German emigration has been plunging down into foreign races where it is lost sight
of. Germandom outside Europe has been undergoing a perpetual national decline. . . .

The Society for German Colonization aims to undertake on its own, in a resolute and sweeping manner,
carefully chosen colonization projects and thereby supplement the ranks of organizations with similar
tendencies.

Its particular tasks will be:

1. to provide necessary sums of capital for colonization;


2. to seek out and lay claim to suitable districts for colonization;
3. to direct German emigrants to these regions.

Carl Peters (German Author), Die Gründung von Deutsch-Ostafrika (The Foundation of German East Africa), 1906

5.10. The passages is an extension of an idea that had previously resulted in __________.
(A) the abolition of serfdom in Russia.
(B) the elimination of the international slave trade across the Atlantic Ocean
(C) the unification of Germany
(D) the implementation of the Great Reform Bill of 1832 in England

Answer: C
Topic: Nation-state formation; Imperialism
Theme: Development and Interaction of Cultures; State Building, Expansion, and Conflict; Development
and Transformation of Social Structures
Learning Objective: CUL–4; SB–4; SOC–6; SOC–7
Historical Thinking Skill: Patterns of Continuity and Change over Time; Periodization
Key Concept: 5.3.II

5.11. In what region did Germany fail to fulfill the objective of the manifesto?
(A) East Africa
(B) West Africa
(C) Caribbean Islands
(D) Pacific Islands

Answer: C

168
Copyright © 2017, 2015, 2011, 2007 Pearson Education, Inc. All rights reserved.
Topic: Nation-state formation; Imperialism; Rise of industrializing powers
Theme: State Building, Expansion, and Conflict; Creation, Expansion, and Interaction of Economic
Structures; Development and Transformation of Social Structures
Learning Objective: SB–1; SB–2; SB–9; SB–10; ECON–3; SOC–7
Historical Thinking Skill: Causation; Contextualization
Key Concept: 5.2.I.B

169
Copyright © 2017, 2015, 2011, 2007 Pearson Education, Inc. All rights reserved.
Questions 5.12–5.14 refer to the image below.

Cartoon from Puck, August 15, 1900

5.12. What was a Chinese reaction to the events depicted in the cartoon?
(A) the Boxer Rebellion
(B) Chinese industrialization
(C) the Sino-Japanese War
(D) the Meiji Restoration

Answer: A
Topic: Resistance to industrialization; Responses to imperialism
Theme: State Building, Expansion, and Conflict; Creation, Expansion, and Interaction of Economic
Structures; Development and Transformation of Social Structures
Learning Objective: SB–1; SB–2; SB–7; SB–9; ECON–3; SOC–7
Historical Thinking Skill: Causation
Key Concept: 5.3.III.D

5.13. Which nineteenth-century event had forced a submission similar to the one depicted in the
cartoon?

170
Copyright © 2017, 2015, 2011, 2007 Pearson Education, Inc. All rights reserved.
(A) the Taiping Rebellion
(B) the Sino-Japanese War
(C) the Opium War
(D) the Boxer Rebellion

Answer: C
Topic: Resistance to industrialization; Responses to imperialism
Theme: State Building, Expansion, and Conflict; Creation, Expansion, and Interaction of Economic
Structures; Development and Transformation of Social Structures
Learning Objective: SB–1; ECON–3; SOC–7
Historical Thinking Skill: Periodization; Contextualization
Key Concept: 5.2.I.E

5.14. Most of the countries depicted in the cartoon had an advantage over China? What was that
advantage?
(A) advanced systems of monetization, including printed currencies
(B) industrialized economies
(C) stable, democratic forms of government
(D) stable and content labor forces

Answer: B
Topic: Industrialization; Formation of new nation-states
Theme: Creation, Expansion, and Interaction of Economic Structures; Development and Transformation
of Social Structures; Development and Interaction of Cultures
Learning Objective: ECON–4; SOC–3; CUL– 3
Historical Thinking Skill: Causation; Comparison
Key Concept: 5.1.I.D; 5.1.V.B

171
Copyright © 2017, 2015, 2011, 2007 Pearson Education, Inc. All rights reserved.
Questions 5.15–5.19 refer to the map and the passage below.

Qing Empire, c. 1890

Anti-foreign sentiment resulted in the rapid growth of a Chinese secret society (which had existed for
centuries) known as the I Ho Ch’uan (Righteous Harmonious Fists), but referred to by the Westerners as
“Boxers.” The Boxers called for the expulsion of the “foreign devils” and their Chinese Christian
converts. The society stressed the ritualistic use of the martial arts and traditional Chinese weapons.
Anti-foreign incidents, including the burning of homes and businesses, increased dramatically in 1898
and 1899, and was primarily directed at Chinese Christians. The number of killings by the Boxers
continued to grow, and on 30 December 1899 included a British missionary. Western governments
lodged strong protests with the Chinese Dowager Empress, Tzu Hsi. She responded on 11 January 1900,
with a declaration that the Boxers represented a segment of Chinese society, and should not be labeled
a criminal organization. Her unenthusiastic support for the Chinese Army’s attempts at quelling the
violence and the influence of Boxer sympathizers at the Imperial court, led Western governments to
deploy military forces on the Chinese coast to protect their citizens and interests.

The Boxer Rebellion and the U.S. Navy, 1900–1901

5.15. Which event began the process of creating the Chinese treaty ports shown on the map?
(A) the Taiping Rebellion
(B) the Boxer Rebellion
(C) the Opium War
(D) the Sino-Japanese War

172
Copyright © 2017, 2015, 2011, 2007 Pearson Education, Inc. All rights reserved.
Answer: C
Topic: Responses to imperialism
Theme: State Building, Expansion, and Conflict; Creation, Expansion, and Interaction of Economic
Structures; Development and Transformation of Social Structures
Learning Objective: SB–9; SB–10; ECON–3; SOC–7
Historical Thinking Skill: Causation; Contextualization; Periodization
Key Concept: 5.2.I.E

5.16. What European ideology motivated the creation of treaty ports?


(A) economic imperialism
(B) Social Darwinism
(C) utilitarianism
(D) socialism

Answer: A
Topic: Ideologies of the nineteenth century; Responses to imperialism
Theme: State Building, Expansion, and Conflict; Creation, Expansion, and Interaction of Economic
Structures; Development and Transformation of Social Structures
Learning Objective: SB–9; 10; ECON–3; SOC–7
Historical Thinking Skill: Causation; Contextualization
Key Concept: 5.2.I.E

5.17. What did the Europeans hope to gain through treaty ports?
(A) a better location from which to launch an invasion of China
(B) refueling stations among their colonial holdings in Asia and the Pacific
(C) an Asian colony to block U.S. advances in the region
(D) better access to raw materials and to markets for finished goods

Answer: D
Topic: Economic Imperialism; Industrial Revolution
Theme: State Building, Expansion, and Conflict; Creation, Expansion, and Interaction of Economic
Structures
Learning Objective: SB–9; ECON–3; ECON–12
Historical Thinking Skill: Causation
Key Concept: 5.1.II.C

5.18. What ideology fueled the rebellion discussed in the passage?


(A) scientific socialism
(B) utilitarianism
(C) nationalism
(D) Social Darwinism

Answer: C
Topic: Nineteenth century Ideologies; Responses to imperialism
Theme: Development and Interaction of Cultures; State Building, Expansion, and Conflict; Creation,
Development and Transformation of Social Structures
Learning Objective: CUL–2; CUL–4; SB–9; SOC–3; SOC–7
Historical Thinking Skill: Causation; Contextualization

173
Copyright © 2017, 2015, 2011, 2007 Pearson Education, Inc. All rights reserved.
Key Concept: 5.3.III.D

5.19. What other global event of the nineteenth century parallels the event identified in the map and
the passage?
(A) the Ghost Dance
(B) the Sepoy revolt
(C) the Boer War
(D) the Opium War

Answer: B
Topic: Responses to imperialism; Nationalism
Theme: Development and Interaction of Cultures; State Building, Expansion, and Conflict; Development
and Transformation of Social Structures
Learning Objective: CUL–2; CUL–4; SB–9; SOC–3; SOC–7
Historical Thinking Skill: Synthesis; Periodization; Causation
Key Concept: 5.3.III.D

174
Copyright © 2017, 2015, 2011, 2007 Pearson Education, Inc. All rights reserved.
Questions 5.20–5.23 refer to the passage below.

SOURCE: Marius B. Jansen, The Making of Modern Japan (Cambridge, MA: Harvard University Press, 2000), 440.

Japan had become the inspiration for all those who made up what the newspapers called “Young
China,” the reformers and intellectuals who saw the country’s salvation in a modernization movement
on the lines of Japan’s espousal of foreign ideas after the Meiji Restoration. By its defeat of China in
1895, Japan had forced them to face the reality of their country’s weakness. By its defeat of Russia ten
years later, Japan had shown that an Asian army could defeat a European one.

Marius B. Jansen, The Making of Modern Japan, 2000

5.20. How was the nineteenth-century self-strengthening movement in China different than the
Japanese action described in the passage?
(A) The self-strengthening movement had supported a preindustrial way of life, rather than
industrialization.
(B) The Japanese had refused to adopt industrial practices during the restoration of the Meiji.
(C) The Chinese government chose to embrace a more democratic form of government in the
self-strengthening movement.
(D) The Japanese decided to model their industrialization after the Chinese, while the Chinese
modeled themselves after the British.

Answer: A
Topic: Formation of new nation-states; Industrial Revolution; Responses to imperialism
Theme: Development and Interaction of Cultures; State Building, Expansion, and Conflict; Creation,
Expansion, and Interaction of Economic Structures; Development and Transformation of Social
Structures
Learning Objective: CUL–3; SB–1; SB–2; SB–4; SB–9; ECON–3; ECON–7; SOC–3
Historical Thinking Skill: Comparison
Key Concept: 5.1.V.B

5.21. What was the major cause for the Japanese success discussed in passage?
(A) Wealthy entrepreneurs financed the majority of Japan’s industrialization.
(B) The United States provided both financing and technical expertise to Japan.
(C) The Meiji government organized and sponsored much of the critical industrialization.
(D) Japanese industrialization developed without the interference of the government.

Answer: C
Topic: Formation of new nation-states; Industrial Revolution; Responses to imperialism; New patterns of
global trade
Theme: Development and Interaction of Cultures; State Building, Expansion, and Conflict; Creation,
Expansion, and Interaction of Economic Structures; Development and Transformation of Social
Structures
Learning Objective: CUL–3; SB–1; SB–2; SB–4; SB–9; ECON–3; ECON–7; SOC–3
Historical Thinking Skill: Causation
Key Concept: 5.1.V.C

175
Copyright © 2017, 2015, 2011, 2007 Pearson Education, Inc. All rights reserved.
5.22. What other state adopted an approach to industrialization similar to the one referenced in the
passage?
(A) the United States
(B) England
(C) China
(D) Russia

Answer: D
Topic: Industrial Revolution; Formation of new nation-states; Industrial production
Theme: Development and Interaction of Cultures; State Building, Expansion, and Conflict; Creation,
Expansion, and Interaction of Economic Structures; Development and Transformation of Social
Structures
Learning Objective: CUL–3; SB–1; SB–2; SB–4; SB–9; ECON–3; ECON–7; SOC–3
Historical Thinking Skill: Comparison
Key Concept: 5.1.V.C

5.23. This rise of Japan, as discussed in the passage, demonstrated what?


(A) that new industrialized empires were developing, while older agricultural-based empires
were declining
(B) that states could successfully become expansionistic without industrializing
(C) that Asian states would have to confine their growth to the Pacific region because they were
unable to defeat any European states
(D) that the presence of democratic states was increasing throughout Asia

Answer: A
Topic: Formation of new nation-states
Theme: State Building, Expansion, and Conflict; Creation, Expansion, and Interaction of Economic
Structures;
Learning Objective: SB–1; SB–2; SB–10; ECON-4
Historical Thinking Skill: Causation; Periodization
Key Concept: 5.2

176
Copyright © 2017, 2015, 2011, 2007 Pearson Education, Inc. All rights reserved.
Questions 5.24–5.26 refer to the image below.

A Political Print

5.24. What event was influenced by the social reality depicted in the image?
(A) the Sepoy Rebellion
(B) the French Revolution
(C) the revolutions of 1848 in Europe
(D) the American Revolution

Answer: B
Topic: Revolution – French Revolution
Theme: Development and Transformation of Social Structures
Learning Objective: SOC–3; SOC–7
Historical Thinking Skill: Causation; Periodization
Key Concept: 5.3.III.B

5.25. What intellectual movement offered a critique of the social situation shown in the image?
(A) Social Darwinism
(B) utopian socialism
(C) the Enlightenment
(D) utilitarianism

Answer: C
Topic: Revolution – French Revolution; Enlightenment

177
Copyright © 2017, 2015, 2011, 2007 Pearson Education, Inc. All rights reserved.
Theme: Development and Interaction of Cultures; State Building, Expansion, and Conflict; Development
and Transformation of Social Structures
Learning Objective: CUL–3; CUL–4; SOC–3; SOC–6
Historical Thinking Skill: Contextualization; Argumentation
Key Concept: 5.3.I.A

5.26. Which of the following statements represents a correct comparison between the image and the
arguments of Mary Wollstonecraft?
(A) Although the image calls for greater political representation in Europe, Wollstonecraft
argued for greater political representation by colonial citizens.
(B) Although the image argues that there should be no limits on royal authority, Wollstonecraft
suggested a constitutional approach to government.
(C) Although the image calls for increased power for religious authorities, Wollstonecraft called
for the elimination of the political power of religious institutions.
(D) Although the image criticized the inequality that existed between social classes,
Wollstonecraft criticized the gender inequality that also existed at the time.

Answer: D
Topic: Revolution – French Revolution; Enlightenment; Arguments for social transformation
Theme: Development and Interaction of Cultures; Development and Transformation of Social Structures
Learning Objective: CUL–3; CUL–5; SOC–3; SOC–6
Historical Thinking Skill: Comparison
Key Concept: 5.3.IV.B

178
Copyright © 2017, 2015, 2011, 2007 Pearson Education, Inc. All rights reserved.
Questions 5.27–5.30 refer to the image below.

Painting of the Tennis Court Oath

5.27. The action depicted in the image quickly led to which of the following in revolutionary France?
(A) the end of Robespierre’s Reign of Terror
(B) the creation of the Declaration of the Rights of Man and of the Citizen
(C) the Napoleonic Wars
(D) widespread participation in Enlightenment salons

Answer: B
Topic: Revolution – French Revolution; Enlightenment – natural rights
Theme: State Building, Expansion, and Conflict; Development and Transformation of Social Structures
Learning Objective: SB–7; SOC–3; SOC–7
Historical Thinking Skill: Causation
Key Concept: 5.3.III.B

5.28. All of the following are ways that the Enlightenment served as an inspiration for the action
depicted in the image except one. Which one?
(A) Enlightenment thinkers believed that political, but not religious, authority should be
challenged.
(B) Enlightenment thinkers argued that a state’s government is comprised of a social contract
between the government and the people.
(C) Enlightenment thinkers suggested that the government was obligated to protect the natural
rights of its citizens.

179
Copyright © 2017, 2015, 2011, 2007 Pearson Education, Inc. All rights reserved.
(D) Enlightenment thinkers urged the use of reason as a way to find practical solutions.

Answer: A
Topic: Revolution – French Revolution; Enlightenment
Theme: Development and Interaction of Cultures; State Building, Expansion, and Conflict; Development
and Transformation of Social Structures
Learning Objective: CUL–3; SB–4; SB–7; SOC–1; SOC–3
Historical Thinking Skill: Causation; Argumentation
Key Concept: 5.3.I.A; 5.3.I.B

5.29. What was the immediate cause of the event depicted in image?
(A) long-anticipated revolution in the United States
(B) skyrocketing bread prices throughout France
(C) closure of the intended meeting hall for the National Assembly
(D) adoption of Enlightenment ideas throughout Europe

Answer: C
Topic: Revolution – French Revolution; Political power struggles
Theme: State Building, Expansion, and Conflict; Development and Transformation of Social Structures
Learning Objective: SB–4; SB–7; SOC–3; SOC-7
Historical Thinking Skill: Causation; Periodization
Key Concept: 5.3.III.B

5.30. Which French colony was ultimately inspired by the event depicted in the image to seek
independence?
(A) India
(B) Haiti
(C) French Indochina
(D) Cameroon

Answer: B
Topic: Revolution – French Revolution; Revolution – Haitian independence; Enlightenment
Theme: State Building, Expansion, and Conflict; Development and Transformation of Social Structures
Learning Objective: SB–4; SB–7; SOC–3; SOC–7
Historical Thinking Skill: Causation
Key Concept: 5.3.III.B

180
Copyright © 2017, 2015, 2011, 2007 Pearson Education, Inc. All rights reserved.
Questions 5.31–5.33 refer to the image below.

Cartoon from Punch, 1906

5.31. What specific event does the image refer to?


(A) Belgian colonization of the Congo
(B) the industrialization of West Africa
(C) abolition challenges to the Atlantic slave trade
(D) competition between European states for political control in Africa

Answer: A
Topic: Imperialism; Reactions to imperialism; Ideologies of the nineteenth century
Theme: Interaction Between Humans and Environment; State Building, Expansion, and Conflict;
Creation, Expansion, and Interaction of Economic Structures; Development and Transformation of Social
Structures
Learning Objective: ENV–9; SB–1; SB–3; SB–9; SB–10; ECON–3; SOC–7
Historical Thinking Skill: Contextualization; Analyzing Evidence: Content and Sourcing

181
Copyright © 2017, 2015, 2011, 2007 Pearson Education, Inc. All rights reserved.
Key Concept: 5.2.I.C

5.32. Which of the following best describes the motive for the event shown in the image?
(A) French debt forced colonial settlers to try and extract more revenue from indigenous
people.
(B) Europeans desired to increase their access to raw materials, specifically rubber.
(C) African colonists were able to successfully rise up and overthrow the European colonizers.
(D) African colonists were needed as indentured labor in other parts of the British Empire.

Answer: B
Topic: Imperialism; Reactions to imperialism
Theme: Development and Interaction of Cultures; Development and Transformation of Social Structures
Learning Objective: CUL–4; CUL–7; SOC–3; SOC–7
Historical Thinking Skill: Causation; Contextualization
Key Concept: 5.1.II.A

5.33. Actions like those depicted in the image most likely created which of the following?
(A) apartheid policies, like those found in South Africa, to keep colonial populations separate
(B) submissive and cooperative colonial populations
(C) nationalistic sentiments among colonial populations to defend against aggression
(D) government legislation to protect the rights of colonial populations

Answer: C
Topic: Imperialism; Reactions to imperialism; Nationalism
Theme: State Building, Expansion, and Conflict; Development and Transformation of Social Structures
Learning Objective: SB–4; SOC–3; SOC–7
Historical Thinking Skill: Causation; Contextualization
Key Concept: 5.3.II

182
Copyright © 2017, 2015, 2011, 2007 Pearson Education, Inc. All rights reserved.
Questions 5.34–5.38 refer to the passage below.

The worst violations of hygienic regulations were those I saw in most of the flax-spinning mills where
linen is produced. . . . Although in western Europe all the dust-producing carding and combing machines
have long been covered and well ventilated, I saw only one Russian linen mill where such a machine was
securely covered. Elsewhere, the spools of these machines were completely open to the air, and the
scutching apparatus is inadequately ventilated. . . .

In factory No. 135 the workers are still treated as serfs. Wages are paid out only twice a year, even then
not in full but only enough to pay the workers’ taxes (other necessities are supplied by the factory
store). Furthermore this money is not given to the workers directly, but is sent by mail to their village
elders and village clerks. Thus the workers are without money the year around. Besides they are also
paying severe fines to the factory, and these sums will be subtracted from their wages at the final year-
end accounting.

Factory Inspection Report, Moscow, 1880s

5.34. Who in Russia was most responsible for starting the process discussed in the passage?
(A) the large middle class
(B) the leaders of the Russian Orthodox Church
(C) wealthy foreign investors
(D) the central Russian government

Answer: D
Topic: Industrial Revolution; Spread of the Industrial Revolution; Government involvement in the
Industrial Revolution
Theme: State Building, Expansion, and Conflict; Development and Transformation of Social Structures
Learning Objective: SB–2; SB–9; SOC–3
Historical Thinking Skill: Causation
Key Concept: 5.1.V.C

5.35. What event had to occur first in order for Russia to participate in the event discussed in the
passage?
(A) the suppression of socialist and anarchist movements in Russia
(B) the emancipation of the serfs
(C) the elimination of the monarchy as an absolutist power
(D) the establishment of a successful overseas empire

Answer: B
Topic: Industrial Revolution; Spread of the Industrial Revolution; Government involvement in the
Industrial Revolution
Theme: Interaction Between Humans and Environment; Creation, Expansion, and Interaction of
Economic Structures; Development and Transformation of Social Structures
Learning Objective: ENV–9; ECON–4; SOC–2; SOC–3
Historical Thinking Skill: Causation; Periodization
Key Concept: 5.3.I.C

183
Copyright © 2017, 2015, 2011, 2007 Pearson Education, Inc. All rights reserved.
5.36. The passages best describes which of the following elements of the Industrial Revolution?
(A) the creation of international markets for raw materials and finished goods
(B) the protectionist nature of governments
(C) the emergence of the factory system
(D) Russia’s leadership in urban industrial reforms

Answer: C
Topic: Industrial Revolution; Spread of the Industrial Revolution; Government involvement in the
Industrial Revolution; Responses to the condition of the Industrial Revolution
Theme: Interaction Between Humans and Environment; Creation, Expansion, and Interaction of
Economic Structures; Development and Transformation of Social Structures
Learning Objective: ENV–9; ECON–4; SOC–2; SOC–3
Historical Thinking Skill: Contextualization
Key Concept: 5.1.I.C

5.37. Whose economic ideas provided much of the inspiration for the events discussed in the passage?
(A) John Stuart Mill
(B) Adam Smith
(C) Charles Darwin
(D) Karl Marx

Answer: B
Topic: Industrial Revolution; Spread of the Industrial Revolution; Government involvement in the
Industrial Revolution; Ideologies of the nineteenth century; Enlightenment
Theme: Development and Interaction of Cultures; Creation, Expansion, and Interaction of Economic
Structures
Learning Objective: CUL–3; ECON–3; ECON–4
Historical Thinking Skill: Contextualization; Causation
Key Concept: 5.1.III.A

5.38. In some nation-states, similar conditions discussed in the passage led to __________.
(A) suffragette movements focused on expanding the franchise for women
(B) significant prison reform and improved police forces
(C) democracy movements that, like that of the Chartists, aimed at universal male suffrage
(D) worker organizations committed to fighting for better conditions

Answer: D
Topic: Industrial Revolution; Spread of the Industrial Revolution; Government involvement in the
Industrial Revolution
Theme: State Building, Expansion, and Conflict; Creation, Expansion, and Interaction of Economic
Structures; Development and Transformation of Social Structures
Learning Objective: SB–4; SB–9; ECON–3; ECON–9; SOC–3
Historical Thinking Skill: Comparison; Synthesis
Key Concept: 5.1.V.A

184
Copyright © 2017, 2015, 2011, 2007 Pearson Education, Inc. All rights reserved.
Questions 5.39–5.42 refer to the passage below.

Finally, China and Japan were on somewhat different paths when the Western challenge intruded in the
mid-nineteenth century. China was suffering one of its recurrent dynastic declines. Government became
less efficient, intellectual life stagnated, and popular unrest surged. A cycle of renewal might have
followed, with a new dynasty seizing more vigorous reins. But Western interference disrupted this
process, complicating reform and creating various new discontents that ultimately overturned the
imperial office. In contrast, Japan maintained political and economic vigor into the nineteenth century.
Whereas by the late nineteenth century China needed Western guidance simply to handle such
bureaucratic affairs as tariff collection and repression of peasant rebellion, Japan suffered no such
breakdown of authority, using foreign advisors far more selectively.

Peter N. Stearns et al., World Civilizations: The Global Experience, 2017

5.39. What does the passage indicate about the direction of modernization in the regions discussed?
(A) While Japan managed to modernize and respond to Western influence, China found itself
facing greater Western interference.
(B) Neither region was able to significantly modernize due to Western interference.
(C) China was able to modernize due to the self-strengthening movement, but the Meiji in
Japan blocked any efforts to do so.
(D) Both China and Japan were successful in modernization, despite efforts by Western nations
to prevent it.

Answer: A
Topic: Industrialization; Formation of new nation-states
Theme: State Building, Expansion, and Conflict; Creation, Expansion, and Interaction of Economic
Structures; Development and Transformation of Social Structures
Learning Objective: SB–1; SB–2; ECON–4; SOC–7
Historical Thinking Skill: Contextualization
Key Concept: 5.2.II.A; 5.2.II.E

5.40. To what governmental change in Japan is the passage referring?


(A) to the Taiping Rebellion
(B) to the installation of the Tokugawa Shogun
(C) to the Meiji Restoration
(D) to the governorship of Matthew Perry

Answer: C
Topic: Formation of new nation-states
Theme: State Building, Expansion, and Conflict; Creation, Expansion, and Interaction of Economic
Structures; Development and Transformation of Social Structures
Learning Objective: SB–1; SB–2; ECON–4; SOC–7
Historical Thinking Skill: Patterns of Continuity and Change over Time
Key Concept: 5.2.II.A

5.41. In what ways did foreign interaction influence the political developments discussed in the
passage?

185
Copyright © 2017, 2015, 2011, 2007 Pearson Education, Inc. All rights reserved.
(A) The devastating loss of the Sino-Japanese War forced the Tokugawa to abdicate the throne.
(B) American imperialism and interaction ultimately resulted in the collapse of Tokugawa’s
isolationist policies.
(C) The loss of the Opium War to Britain forced the Japanese to sign the unequal Treaty of
Kanagawa.
(D) The Meiji were installed in power after the Boxer Rebellion failed to remove the Tokugawa
from power.

Answer: B
Topic: Foreign Interaction; Formation of new nation-states
Theme: State Building, Expansion, and Conflict; Creation, Expansion, and Interaction of Economic
Structures; Development and Transformation of Social Structures
Learning Objective: SB–1; SB–2; ECON–4; SOC–7
Historical Thinking Skill: Causation
Key Concept: 5.2.II.A

5.42. Which of the following represents an example of the Chinese popular unrest referenced in the
passage?
(A) the self-strengthening movement by the Qing dynasty
(B) the Treaty of Nanking with the British
(C) the Sino-Japanese War
(D) the Taiping uprising against the Qing

Answer: D
Topic: Responses to imperialism
Theme: State Building, Expansion, and Conflict; Development and Transformation of Social Structures
Learning Objective: SB–4; SB–7; SOC–3; SOC–7
Historical Thinking Skill: Periodization
Key Concept: 5.3.III.A

186
Copyright © 2017, 2015, 2011, 2007 Pearson Education, Inc. All rights reserved.
Questions 5.43–5.45 refer to the image below.

Theodore Roosevelt, Panama, c. 1904

5.43. What development in industrial transportation is depicted in the image?


(A) railroad track construction
(B) steamboat construction
(C) canal construction
(D) paved road construction

Answer: C
Topic: Economic imperialism; Formation of new nation-states; Transportation revolution
Theme: Interaction Between Humans and Environment Creation, Expansion, and Interaction of
Economic Structures
Learning Objective: ENV–6; ECON–2
Historical Thinking Skill: Periodization; Causation
Key Concept: 5.1.IV

5.44. Which nation’s rise in industrial power and pursuit of economic imperialism is depicted in the
image?
(A) Great Britain

187
Copyright © 2017, 2015, 2011, 2007 Pearson Education, Inc. All rights reserved.
(B) the United States
(C) Russia
(D) France

Answer: B
Topic: Economic imperialism; Industrialization; Formation of new nation-states; New patterns of global
trade
Theme: Interaction Between Humans and Environment; State Building, Expansion, and Conflict;
Creation, Expansion, and Interaction of Economic Structures
Learning Objective: ENV–6; SB–4; SB–7; ECON–2
Historical Thinking Skill: Comparison; Contextualization
Key Concept: 5.1.IV; 5.3.I.E

5.45. The image illustrates what aspect of economic imperialism at the turn of the century?
(A) that colonial nations were growing economically independent, despite not achieving
political independence
(B) that European nations continued to be the only states with successful imperialist policies
(C) that U.S. imperialistic goals and policies were similar those of Britain and France
(D) that Asian colonies were no longer valued, as interest in western hemispheric colonies
increased

Answer: C
Topic: Economic imperialism; Industrialization; Formation of new nation-states
Theme: Interaction Between Humans and Environment; State Building, Expansion, and Conflict;
Creation, Expansion, and Interaction of Economic Structures
Learning Objective: ENV–6; SB–4; SB–7; ECON–2
Historical Thinking Skill: Causation; Comparison
Key Concept: 5.1.IV; 5.3.I.E

188
Copyright © 2017, 2015, 2011, 2007 Pearson Education, Inc. All rights reserved.
Questions 5.46–5.48 refer to the passage and image below.

[T]he education of the English boy, then as now, was considered a much more serious matter than the
education of the English boy’s sister. . . . Of course [I] went to a carefully selected girls’ school, but
beyond the facts that the head mistress was a gentlewoman and that all the pupils were girls of my own
class, nobody seemed concerned. A girl’s education at that time seemed to have for its prime object the
art of “making home attractive”—presumably to migratory male relatives. It used to puzzle me to
understand why I was under such a particular obligation to make home attractive to my brothers. We
were on excellent terms of friendship, but it was never suggested to them as a duty that they make
home attractive to me. Why not? Nobody seemed to know.

Emmeline Pankhurst, Autobiography, 1914

Emmeline Pankhurst

5.46. Beyond the argument made in her autobiography, what other cause did Emmeline Pankhurst fight
for?
(A) suffrage for women
(B) political freedom for Haiti
(C) abolition of serfdom in Russia
(D) abolition of the income tax

Answer: A
Topic: Education; Women’s rights/suffrage
Theme: Development and Interaction of Cultures; Development and Transformation of Social Structures
Learning Objective: CUL–3; CUL–5; SOC–1; SOC–2; SOC–3; SOC–4

189
Copyright © 2017, 2015, 2011, 2007 Pearson Education, Inc. All rights reserved.
Historical Thinking Skill: Contextualization
Key Concept: 5.3.IV.B

5.47. In what ways did earlier thinkers, such as Mary Wollstonecraft, impact late nineteenth-century
activists like the ones depicted in the image?
(A) Wollstonecraft argued that women should be given full political rights, just like Emmeline
Pankhurst.
(B) Wollstonecraft argued for an end to child labor, and Emmeline Pankhurst continued that
crusade.
(C) Wollstonecraft believed that women should have equal access to employment, and
Emmeline Pankhurst petitioned the British government for equal job opportunities.
(D) Wollstonecraft challenged traditional ideas about women’s rights, and Emmeline Pankhurst
extended Wollstonecraft’s arguments and called for women’s suffrage.

Answer: D
Topic: Education; Women’s rights/suffrage; Enlightenment
Theme: Development and Interaction of Cultures; Development and Transformation of Social Structures
Learning Objective: CUL–3; CUL–5; SOC–1; SOC–2; SOC–3
Historical Thinking Skill: Comparison
Key Concept: 5.3.I.B; 5.3.IV.B

5.48. Although Emmeline Pankhurst was British, other global movements pursued similar reformist
ideas, including which of the following?
(A) William Wilberforce’s abolition of the slave trade
(B) the Chinese Empress Cixi’s support of the Boxer Rebellion
(C) the Seneca Falls Convention in the United States
(D) the abolition of the mita in Mexico

Answer: C
Topic: Education; Women’s rights/suffrage; Social reform
Theme: Development and Interaction of Cultures; Development and Transformation of Social Structures
Learning Objective: CUL–3; CUL–5; SOC–6
Historical Thinking Skill: Comparison; Synthesis
Key Concept: 5.2.III

190
Copyright © 2017, 2015, 2011, 2007 Pearson Education, Inc. All rights reserved.
Questions 5.49–5.51 refer to the passage below.

We are a young people. We inhabit a world apart, separated by broad seas. We are young in the ways of
almost all the arts and sciences, although, in a certain manner, we are old in the ways of civilized
society. I look upon the present state of America as similar to that of Rome after its fall. Each part of
Rome adopted a political system conforming to its interest and situation or was led by the individual
ambitions of certain chiefs, dynasties, or associations. But this important difference exists: those
dispersed parts later reestablished their ancient nations, subject to the changes imposed by
circumstances or extent. But we scarcely retain a vestige of what once was; we are, moreover, neither
Indian nor European, but a species midway between the legitimate proprietors of this country and the
Spanish usurpers. In short, although Americans by birth we derive our rights from Europe, and we have
to assert these rights against the rights of the natives, and at the same time we must defend ourselves
against the invaders. This places us in a most extraordinary and involved situation. . . . The role of the
inhabitants of the American hemisphere has for centuries been purely passive. Politically they were
nonexistent. We are still in a position lower than slavery, and therefore it is more difficult for us to rise
to the enjoyment of freedom. . . . States are slaves because of either the nature or the misuse of their
constitutions; a people is therefore enslaved when the government, by its nature or its vices, infringes
on and usurps the rights of the citizen or subject. Applying these principles, we find that America was
denied not only its freedom but even an active and effective tyranny. . . . We have been harassed by a
conduct which has not only deprived us of our rights but has kept us in a sort of permanent infancy with
regard to public affairs.

Simón Bolivar, Jamaica Letter, 1815

5.49. What event in the period from 1450 to 1750 created the conditions that Simón Bolivar discussed
in the passage?
(A) the colonization of the Western Hemisphere by Europeans
(B) the end of feudalism and the beginning of the Renaissance
(C) the rise and expansion of the Aztecs
(D) the conquest of the Byzantines by the Ottomans

Answer: A
Topic: Revolution and independence movements – Latin America
Theme: Development and Interaction of Cultures; State Building, Expansion, and Conflict; Development
and Transformation of Social Structures
Learning Objective: CUL–2; CUL–4; SB–1; SB–4; SB–7; SB–9; SOC–3; SOC–7
Historical Thinking Skill: Argumentation; Contextualization; Periodization
Key Concept: 5.3.III.B

5.50. What other nation’s actions inspired and influenced the ideas that Simón Bolivar highlighted in the
passage?
(A) the Industrial Revolution in Britain
(B) the removal of the Tokugawa shogun during the Meiji Restoration.
(C) the successful independence movement of the United States
(D) the emancipation of the serfs in Russia

Answer: C

191
Copyright © 2017, 2015, 2011, 2007 Pearson Education, Inc. All rights reserved.
Topic: Revolution and independence movements – Latin America
Theme: Development and Interaction of Cultures; State Building, Expansion, and Conflict; Development
and Transformation of Social Structures
Learning Objective: CUL–2; CUL– 4; SB–1; SB–4; SB–7; SB–9; SOC–3; SOC–7
Historical Thinking Skill: Comparison
Key Concept: 5.3.III.B

5.51. In what ways are the ideas expressed in the passage and the ideas expressed in the French
Declaration of the Rights of Man similar?
(A) They both called for the abolition of slavery.
(B) They both called for a revival of the political system of the Roman Empire.
(C) They both called for a renewed commitment to colonization.
(D) They both called for political changes based on the ideas of the Enlightenment.

Answer: D
Topic: Revolution and independence movements – Latin America; French Revolution
Theme: Development and Interaction of Cultures; State Building, Expansion, and Conflict; Development
and Transformation of Social Structures
Learning Objective: CUL–3; CUL–4; SB–4; SB–7; SOC–3; SOC–7
Historical Thinking Skill: Comparison; Contextualization
Key Concept: 5.3.I.B

192
Copyright © 2017, 2015, 2011, 2007 Pearson Education, Inc. All rights reserved.
Questions 5.52–5.55 refer to the map below.

Industrialization in Europe, c. 1850

5.52. Where did the process depicted in the map first begin?
(A) France
(B) Prussia
(C) England
(D) Belgium

Answer: C
Topic: Industrialization; Spread of the Industrial Revolution; Railroads
Theme: Interaction Between Humans and Environment; Creation, Expansion, and Interaction of
Economic Structures
Learning Objective: ENV–9; ECON–4; ECON–5; ECON–9
Historical Thinking Skill: Causation; Periodization
Key Concept: 5.1.I.D

5.53. All of the following are factors that led to the beginning of industrial production, as shown on this
map, excluding one. Which one?
(A) availability of natural resources like coal and iron
(B) legal protections for private property
(C) extensive government involvement
(D) availability of capital

Answer: C
Topic: Industrialization; Spread of the Industrial Revolution
Theme: Development and Interaction of Cultures
Learning Objective: CUL–3
Historical Thinking Skill: Causation

193
Copyright © 2017, 2015, 2011, 2007 Pearson Education, Inc. All rights reserved.
Key Concept: 5.3.I.B

5.54. What technological invention directly facilitated the trend illustrated in the map?
(A) the water frame
(B) the telegraph
(C) the internal combustion engine
(D) the steam engine

Answer: D
Topic: Industrialization; Spread of the Industrial Revolution; Steam engine
Theme: Interaction Between Humans and Environment; Creation, Expansion, and Interaction of
Economic Structures
Learning Objective: ENV–6; ENV–9; ECON–1; ECON–2
Historical Thinking Skill: Causation
Key Concept: 5.1.I.B; 5.1.IV

5.55. If the map included Russia, what trend would it show?


(A) a failure by Russia to participate in railroad building
(B) a trans-Siberian railroad supported by government investment
(C) trans-Russian industrialization in areas like textile production, but no railroads beyond the
capital
(D) no significant industrialization of any kind in Russia because of its ongoing commitment to
serfdom

Answer: B
Topic: Industrialization; Spread of the Industrial Revolution; Government involvement; Railroads
Theme: Creation, Expansion, and Interaction of Economic Structures
Learning Objective: ECON–3; ECON–4
Historical Thinking Skill: Synthesis; Periodization; Comparison
Key Concept: 5.1.III.C

194
Copyright © 2017, 2015, 2011, 2007 Pearson Education, Inc. All rights reserved.
Questions 5.56–5.58 refer to the passage below.

SOURCE: Adam Smith, An Inquiry into the Nature and Causes of the Wealth of Nations, ed. Edwin Cannan (Chicago: The
University of Chicago Press, 1976), 8–9; 11–13 passim, 15, 218–23, 302–06.

[On Education] In the progress of the division of labour, the employment of the far greater part of those
who live by labour, that is, of the great body of the people, comes to be confined to a few very simple
operations; frequently to one or two. But the understandings of the greater part of men are necessarily
formed by their ordinary employments. The man whose whole life is spent in performing a few simple
operations, of which the effects too are, perhaps, always the same, or very nearly the same, has no
occasion to exert his understanding, or to exercise his invention in finding out expedients for removing
difficulties which never occur. He naturally loses, therefore, the habit of such exertion, and generally
becomes as stupid and ignorant as it is possible for a human creature to become . . .

The education of the common people requires, perhaps, in a civilized and commercial society, the
attention of the public more than that of people of some rank and fortune. People of some rank and
fortune are generally eighteen or nineteen years of age before they enter upon that particular business,
profession, or trade, by which they propose to distinguish themselves in the world. They have before
that full time to acquire, or at least to fit themselves for afterwards acquiring, every accomplishment
which can recommend them to the public esteem, or render them worthy of it. Their parents or
guardians are generally sufficiently anxious that they should be so accomplished, and are, in most cases,
willing enough to lay out the expence which is necessary for that purpose . . .

Adam Smith, An Inquiry into the Nature and Causes of the Wealth of Nations, 1776

5.56. In addition to the topic addressed in passage, what else did Adam Smith write about?
(A) utilitarianism
(B) romantic nationalism
(C) economic liberalism
(D) Social Darwinism

Answer: C
Topic: Enlightenment; Education; Liberalism
Theme: Creation, Expansion, and Interaction of Economic Structures
Learning Objective: ECON–3; ECON–9; ECON–11
Historical Thinking Skill: Contextualization
Key Concept: 5.1.III.A

5.57. What larger intellectual movement was Adam Smith drawing on when he wrote the words in the
passage?
(A) the Renaissance
(B) the Enlightenment
(C) conservatism
(D) romantic nationalism

Answer: B
Topic: Enlightenment; Education; Liberalism
Theme: Development and Interaction of Cultures; Development and Transformation of Social Structures

195
Copyright © 2017, 2015, 2011, 2007 Pearson Education, Inc. All rights reserved.
Learning Objective: CUL–4; CUL–7; SOC–3
Historical Thinking Skill: Periodization; Causation
Key Concept: 5.3.I.A

5.58. What other contemporary thinkers would have agreed with the argument that Adam Smith is
making in this selection?
(A) Voltaire and Montesquieu
(B) Hobbes and Condorcet
(C) Quesnay and Beccaria
(D) Locke and Rousseau

Answer: D
Topic: Enlightenment; Education; Liberalism
Theme: Development and Interaction of Cultures; Development and Transformation of Social Structures
Learning Objective: CUL–4; CUL–7; SOC–3
Historical Thinking Skill: Comparison; Contextualization
Key Concept: 5.3.I.A

196
Copyright © 2017, 2015, 2011, 2007 Pearson Education, Inc. All rights reserved.
Section II
Short-Answer Questions
5.59. Use your knowledge of world history to answer all parts of the question that follows.

(A) Identify and explain two factors that led to the rise of industrial production in Europe.
(B) Identify and explain one significant difference between the “first” and “second” Industrial
Revolutions.
(C) Identify and explain how one non-European nation approached industrialization and how it
was similar to and/or different from Western European industrialization.

Sample Answer

(A) Students could identify any of the following factors that led to industrial production in
Europe:
Access to the Atlantic
Natural resource availability—coal, iron, timber, rivers
Colonies—for supplying raw materials and markets for buying finished goods
Limited governmental interference in the economy
Openness to capital enterprises
Urbanization
Second agricultural revolution and improved agricultural productivity
Legal protection of private property

(B) Students could note the following differences between the “first” and “second” Industrial
Revolutions:
Different products and processes. The “first” Industrial Revolution involved textiles; the
“second” Industrial Revolution involved steel, chemicals, and precision machinery.
Different forms of power. The “first” Industrial Revolution was focused on steam engines.
The ”second” Industrial Revolution also used electricity.
New forms of transportation. The “first” Industrial Revolution involved steam engines
(trains, steamboats). The “second” Industrial Revolution saw the introduction of
internal combustion engines, diesel engines, and early experiments with flight.
New communication options. The “second” Industrial Revolution” saw the introduction of
the telegraph, radio waves, early experiments with the telephone, and photography.
New nations in leadership. In the “second” Industrial Revolution, Germany emerged as a
new leader in Europe.

(C) Possible examples of industrialization in non-European nations:


Japan jumped fully into industrialization but did so with government involvement (Britain
had limited governmental involvement).
The Ottomans took a more limited interest in industrialization than Western Europe, had
governmental involvement, and did pursue railroads by working with the Germans.
In Egypt, the government wanted to industrialize, but Britain would not share technology
with Egypt because Britain wanted Egyptian raw materials (cotton).

197
Copyright © 2017, 2015, 2011, 2007 Pearson Education, Inc. All rights reserved.
Russia pursued industrialization after 1861, had heavy government involvement, and
focused on steel and railroads.
In China, the Qing were slow to adopt industrialization and preferred agrarian, rather than
industrial, pursuits. The self-strengthening movement reaffirmed the Qing position
regarding industrialization.

Topic: Factors that led to the Industrial Revolution; First and second industrialization; Non-European
approaches to industrialization
Theme: Interaction Between Humans and Environment; State Building, Expansion, and Conflict;
Creation, Expansion, and Interaction of Economic Structures; Development and Transformation of Social
Structures
Learning Objective: ENV–9; SB–5; ECON–2; ECON–4; ECON–5; ECON–9; SOC–2; SOC–3; SOC–4
Historical Thinking Skill: Causation; Comparison; Periodization
Key Concept: 5.1.I.A; 5.1.I.E; 5.1.II.A; 5.1.II.C

198
Copyright © 2017, 2015, 2011, 2007 Pearson Education, Inc. All rights reserved.
5.60. Use the image and passage below and your knowledge of world history to answer all parts of the
question that follows.

Engraving of British Opium Factory, Nineteenth Century

Not only was Japan now a major world power, but its performance had excited the admiration of the
world. In England there was a “learn from Japan” movement that called for a rebirth of patriotism and
loyalty. Throughout Asia the fact the Japan had defeated a major imperialist power attracted the
admiration of nationalists of many stripes.

Marius B. Jansen, The Making of Modern Japan, 2002, p. 440

(A) Identify two nations and explain how each used economic imperialism to expand its
influence.
(B) Identify and explain how imperialism influenced state formation in one region of the world.
(C) Identify and explain how racial ideology was used to justify imperialism.

Sample Answers

(A) Students could use the following examples of two nations that used economic imperialism
to expand their influence
The United States — activity in the Pacific, the Treaty of Kanagawa with Japan, the building
of the Panama Canal, involvement in the Spanish-American War, and heavy
investment in Latin America.

199
Copyright © 2017, 2015, 2011, 2007 Pearson Education, Inc. All rights reserved.
Britain and France — Heavy investment in Latin America (Mexico, e.g.) gave Britain and
France an economic foothold in the region, even though most of their colonial
possessions had gained independence.
Britain — Britain used opium from its colony India to gain economic control and influence in
China (Opium War – students could cite the image above).
France — France and other states, like Britain, also carved out spheres of influence in China.

(B) Examples of the influence of imperialism on state formation in one region of the world:
The Meiji Restoration, which removed the Tokugawa from power, was a result of
imperialistic contact with the United States and European states. In response to
demands by Western powers, the Meiji asserted its control over Japan and created a
new constitutional, industrialized nation (students could cite the passage above).
The Qing failure to defeat European powers (and ultimately the United States) led to the
contraction of the Qing state. Failure to industrialize further weakened the Qing.
Formation of the Indian National Congress, as a petitioning body, changed the political
standing of India.
Granting of dominion status to nation-states like Canada, Australia and South Africa
changed the political status of these regions.

(C) Part C of the question is asking student’s to discuss Social Darwinism. These observations
could be made:
Social Darwinism was a derivative of Darwin’s ideas of natural selection.
Social Darwinism was used by imperial powers to rationalize their conquests of territory
controlled by “weaker” states.
Social Darwinism argued that some states (and people and groups) were destined to
conquer whereas others were destined to be conquered.
According to Social Darwinism, one was expected to accept one’s fate.

Topic: Development of imperialism


Theme: Interaction Between Humans and Environment; Development and Interaction of Cultures; State
Building, Expansion, and Conflict; Creation, Expansion, and Interaction of Economic Structures;
Development and Transformation of Social Structures
Learning Objective: ENV–9; CUL–3; CUL–4; SB–1; SB–2; SB–3; SB–4; SB–6; SB–10; ECON–3; ECON–4;
ECON–8; SOC–6; SOC–67
Historical Thinking Skills: Comparison; Causation
Key Concept: 5.2.I.C; 5.2.I.E; 5.2.II.A; 5.2.II.B; 5.2.III

200
Copyright © 2017, 2015, 2011, 2007 Pearson Education, Inc. All rights reserved.
5.61. Use the passages below and your knowledge of world history to answer all parts of the question
that follows.

Henceforth the French nation proclaims the sovereignty of the people, the suppression of all
civil and military authorities which have governed you up to the present, and of all taxes which
you sustain, in whatever form they exist; the abolition of the tithe, of feudalism, of seigneurial
[feudal] rights . . . of real and personal servitude, of aristocratic hunting and fishing privileges,
[labor service and all manorial taxes], and generally of every species of contributions with which
you have been burdened by your usurpers; it proclaims also the abolition among you of all
prerogatives and privileges that are contrary to equality. You are henceforth, brothers and
friends, all citizens, all equal in rights, and all equally summoned to govern, to serve, and to
defend your Patrie [fatherland].

Decree of the Revolutionary National Convention in France, 1792

Do they think that men who have been able to enjoy the blessing of liberty will calmly see it
snatched away? [Slaves] supported their chains only so long as they did not know any condition
of life more happy than slavery. But today when they have left, if they had a thousand lives they
would sacrifice them all rather than be forced into slavery again.

Toussaint L’Ouverture, “Protest to the Directory,” 1798

(A) Identify and explain two causes for the French Revolution of 1789.
(B) Identify and explain two causes for the revolution in the British colonies of North America.
(C) Identify and explain how ideas of the Enlightenment contributed to the independence
movement in Haiti.

Sample Answers

(A) Causes for the French Revolution:


Enlightenment principles (students could cite the passage from the Decree of the
Revolutionary National Convention in France)
Debt and government bankruptcy
Food shortages
American Revolution
Social class crisis (Estates)
Dissatisfaction with the monarchy and absolutism
Unequal taxation policies

(B) Causes for the American Revolution:


Enlightenment principles
Denial/limitation of rights of British citizens (natural rights)
Taxation without the consent of the people
No representation in the British government
Salutatory neglect
Quartering of soldiers
French and Indian War

201
Copyright © 2017, 2015, 2011, 2007 Pearson Education, Inc. All rights reserved.
(C) Enlightenment ideas that contributed to independence movement in Haiti:
Enlightenment talked about natural rights for all people.
Numerous Enlightenment thinkers (Condorcet, Montesquieu Diderot, etc.) spoke against
slavery.
The French Revolution embraced the ideas of the Enlightenment in the Declaration of the
Rights of Man. Haitians demanded those same rights and equality (students could cite
the passage from the Decree of the Revolutionary National Convention in France).
As a result of the French Revolution and its embrace of Enlightenment ideas, France
eliminated slavery in Haiti, which ultimately led to the development of an
independence movement (students could cite the passage from Toussaint
L’Ouverture)

Topic: Causes of revolution; Enlightenment; Abolition; New nation-states


Theme: Development and Interaction of Cultures; State Building, Expansion, and Conflict; Creation,
Expansion, and Interaction of Economic Structures; Development and Transformation of Social
Structures
Learning Objective: CUL–2; CUL–3; CUL–4; CUL–7; SB–1; SB–2; SB–4; SB–7; ECON–7; SOC–1; SOC–2;
SOC–3; SOC–6; SOC–7
Historical Thinking Skill: Causation, Periodization
Key Concept: 5.3.I.A; 5.3.I.B; 5.3.I.C; 5.3.III.B; 5.3.III.C

202
Copyright © 2017, 2015, 2011, 2007 Pearson Education, Inc. All rights reserved.
5.62. Use your knowledge of world history to answer all parts of the question that follows.

(A) Identify and explain two reasons for migration in the nineteenth century.
(B) Identify and explain one impact that migrants had on the communities they settled in.
(C) Identify and explain one way nation-states attempted to regulate the movement of migrants
into their countries.

Sample Answers

(A) Reasons for migration:


Changes in food production and improved medical conditions led to population increases
(which led, in turn, to population pressure and, therefore, migration).
Available transportation made migration easier.
Some migrants moved freely for job opportunities (Industrial Revolution, specialized
professions).
Some migrants moved because they were members of a coerced or semi-coerced group
(slavery at the beginning of the time period, Indian indentured servants to other parts
of the empire).
Some migrants were forced to move because of conditions (Irish Potato Famine).
Some migrants were forced to move because of transportation/convict policies.

(B) Impact of migration on communities:


Created ethnic enclaves in the places migrants settled. This transplanted migrants’cultures
to new regions.
Provided needed labor (free and coerced).
At times, created overcrowding conditions.
Examples — Chinese in Southeast Asia; Chinese in the Western Hemisphere; Indians in East
and South Africa; Indians in the Caribbean; Indians in Southeast Asia

(C) Examples of government regulation of migrant movement:


Chinese Exclusion Act in the United States
White Australia Policy

Topic: Reasons for migration; Impacts of migrants; Negative governmental responses to migration
Theme: Interaction Between Humans and Environment; Development and Interaction of Cultures; State
Building, Expansion, and Conflict; Creation, Expansion, and Interaction of Economic Structures;
Development and Transformation of Social Structures
Learning Objective: ENV–3; ENV–5; CUL–9; SB–5; ECON–2; ECON–4; ECON–12; SOC–1; SOC–8
Historical Thinking Skill: Causation; Argumentation
Key Concept: 5.4.I.A; 5.4.II.A; 5.4.II.B; 5.4.III.A; 5.4.III.C

203
Copyright © 2017, 2015, 2011, 2007 Pearson Education, Inc. All rights reserved.
Section III

Long Essay Question


Question 5.63: Evaluate the continuities and changes in empire-building for one of the following nations
during the period from 1750 to 1900.

Britain—————Japan—————United States

Sample Answer

This question asks students to evaluate the continuities and changes in empire-building during the time
period from 1750 to 1900 in Britain, Japan, or the United States. Students should understand that they
must deal with both changes and continuities in their thesis and with argument development, though
not necessarily evenly. To gain the thesis point, students must address all parts of the question by
identifying appropriate continuities and changes in empire-building for one of the designated regions
during the designated time period.

Points gained through the use of evidence shall be determined by using specific evidence and relevant
examples that fully and effectively support the thesis/argument. Evidence and analysis that
students might utilize to gain up to two points are noted in the following paragraphs:

Great Britain – Students might point to the following examples to identify changes in empire-building
during the time period: Students might note how British control in the Western Hemisphere changed
with the loss of the United States once the American Revolution began in 1775. Additionally, the British
expanded their holdings by taking control of the Indian Ocean from the Dutch in the 1700s and gained
control over Indian Ocean in 1700s. Students might note that during the first half of this time period the
British often exercised limited political control in their colonies (e.g., the practice of salutatory neglect in
North America and the authority given to the East India Company in Indi a).

However, by the end of the time period, Britain shifted from indirect to direct control in India after the
Sepoy uprising. Continuities in British empire building included ongoing economic motives (for trade
wealth opportunities, supplies of raw materials, and markets for finished goods). Additional continuities
include the promotion of private enterprise (joint-stock companies, capitalism) and ongoing paternalistic
attitudes towards indigenous people.

The United States – Students might note that the greatest change over this time period for the United
States was the transition from a colony to a nation-state that was growing in imperial power by the
beginning of the twentieth century. An additional change for the United States was the geographic focus
of empire-building. During the nineteenth century, the United States first focused on expansion across
North America (manifest destiny) and turned to Latin America (via the Monroe Doctrine and economic
investment). By the end of the nineteenth century, the United States had expanded its empire building
into the Pacific, gaining control of the Philippines after the Spanish-American War. One continuity to cite
for the United States would be the heavy reliance on trade and economic imperialism, rather than direct
political takeover of large amounts of territory, over the course of the nineteenth century. The United
States economic imperialism was critical in opening up Japan to the West, and the United States also

204
Copyright © 2017, 2015, 2011, 2007 Pearson Education, Inc. All rights reserved.
pursued economic imperialism at the end of the nineteenth century by carving out a sphere of influence
in China and in exerting economic influence in Latin America, including Panama.

Japan – Japan’s most significant change came in the middle of the nineteenth century. Prior to the
arrival of Matthew Perry, Japan’s Tokugawa Shogunate had adopted a policy of isolationism that kept
Ja[am from building an empire. After Japan ended its isolationism and installed the Meiji, the Japanese
began actively to focus on empire-building. Fueled by modernization and industrialization, Japan actively
pursued an empire and defeated the Chinese in the Sino-Japanese War and the Russians in the Russo-
Japanese War. Both wars added to the Japanese empire. As Japan transitioned from isolationism to
empire, Japan continued to value and revere Japanese culture above others and made certain that
foreign ideas did not subvert Japanese culture. Additionally, Japan’s ongoing support and reliance on
military strength and devotion fueled Japan’s ability to construct an empire.

In this question, one point can be gained through the students’ use of the targeted historical evidence
by providing examples of both continuities and changes in one nation’s pursuit of empire. An
additional one point can also be gained if a student explains the reasons for patterns of continuity and
change over time.

There are many ways that students can earn one point for synthesis. Students could talk about
decolonization in the twentieth century and relate it to the independence of the United States. Students
could discuss the expansion or isolation of other states during this time period (e.g., Russia, the Ottoman
Empire, Spain, or Portugal) and show how all of these states were part of a larger trend. Additionally,
students could talk about other historical empires—the Mongols, the Romans, the Greeks under
Alexander the Great—and discuss the overall nature of empire building.

Topic: Imperialism; Nation-state building and new nation formation


Theme: Development and Interaction of Cultures; State Building, Expansion, and Conflict; Creation,
Expansion, and Interaction of Economic Structures; Development and Transformation of Social
Structures
Learning Objective: CUL–3; SB–1; SB–2; SB–3; SB–9; SB–10; ECON–3; ECON–4; SOC–7
Historical Thinking Skill: Argumentation; Patterns of Continuity and Change over Time
Key Concept: 5.2.I.A; 5.2.I.B; 5.2.I.E; 5.2.II.A; 5.2.II.B

205
Copyright © 2017, 2015, 2011, 2007 Pearson Education, Inc. All rights reserved.
Question 5.64: Compare the way two of the following states responded to Western encroachment in
the period from 1750 to 1900.

Japan—————China—————India

Sample Answer

This question asks students to compare the way that two states responded to Western encroachment in
the period from 1750 to 1900. Students need to compare Japan and China, Japan and India, or China
and India. Students should understand that compare means discussing both similarities and differences.
They must describe both similarities and differences in their thesis and in their argument development,
though not necessarily evenly. To gain the thesis point, students must address all parts of the question
by identifying appropriate similarities and differences in responses to Western encroachment for two of
the designated regions during the designated time period.

Points gained through the use of evidence shall be determined by use of specific evidence and of
relevant examples that fully and effectively support the thesis/argument. Evidence and analysis that
students might utilize to gain up to two points are noted in the following paragraphs:

Japan and China – Similarities that students might point out include: (1) both encountered Western
influences at a time of political vulnerability (China – Opium War; Japan – incursion of Matthew Perry
and the American fleet); (2) both were forced to sign unequal treaties by Western nations (the Treaty of
Nanking and the Treaty of Kanagawa); (3) both encountered Western encroachment at a time when
they were not industrialized and were therefore unable to defend themselves effectively. Differences
that students might point out include Japan’s challenge to Western encroachment by ending
isolationism and embracing modernization and industrialization. On the other hand, China continued to
push for agrarian policies, not for industrialization. As a consequence, Japan was able to push back
against Western power. Japan’s approach to modernization turned the state into an imperial power that
began seeking colonies of its own. China, which continued to resist industrialization, continued to be
undermined by foreign powers and economic imperialism, which ultimately resulted in China being
divided into spheres of influence.

Japan and India – Similarities that students might discuss include the fact that both states attempted to
negotiate with Western powers and maintain autonomy. The Mughals were willing to negotiate with
the British East India Company in an attempt to retain political control over India. The Japanese also
negotiated with the United States to retain control over its own political system. Additionally, both
signed agreements with Western powers that gave away trade privileges to the West. Japan signed the
Treaty of Kanagawa with the United States, which assigned trade privileges to the United States and
political control to Japan. The Mughals also attempted to retain political control by giving the East India
Company trade privileges. Differences that students might note center on political differences. Although
Japan was able to retain political independence, the Mughals ultimately lost control of their nation to
the British. Another difference students might note is that Indian citizens rebelled against the presence
of westerners at both the Battle of Plassey and during the Sepoy uprising. The Japanese, however,
avoided the need to rebel by retaining and strengthening their political control.

India and China – India and China both struggled in the face of Western encroachment. Both states
found themselves the lesser partner in trade agreements with the British. Neither state was ultimately
able to control trade. In India, the East India Company came to control trade for the benefit of Britain

206
Copyright © 2017, 2015, 2011, 2007 Pearson Education, Inc. All rights reserved.
and in China the unequal Treaty of Nanking gave the British control over trade as well. An additional
similarity is that both China and India lost military campaigns against the British. The Chinese lost the
Opium Wars, and India lost the Battle of Plassey and the Sepoy uprising. A major difference that
students might note is that China was able to retain political control over the state, whereas India lost
political autonomy to the British after the Sepoy uprising.

In this question, one point can be gained through the students’ use of targeted historical evidence by
providing examples of both similarities and differences in the way two states responded to western
encroachment. An additional one point can also be gained if a student explains the reasons for a
similarity and difference.

There are many ways that students can earn one point for synthesis. For example, students could
discuss African responses to Western encroachment in this same time period, or they could evaluate the
reactions of American empires—the Aztecs, Inca, and Anasazi—to encroachment in the period from
1450 to 1750. Students could also discuss a related course theme (industrial power) or the intellectual
arguments made to justify imperialism, such as Social Darwinism or the discourse surrounding “civilizing
missions.”

Topic: Imperialism; Formation of new nation-states; Industrialization


Theme: Development and Interaction of Cultures; State Building, Expansion, and Conflict; Creation,
Expansion, and Interaction of Economic Structures; Development and Transformation of Social
Structures
Learning Objective: CUL–3; SB–1; SB–2; SB–3; SB–6; SB–9; SB–10; ECON–3; SOC–7
Historical Thinking Skill: Argumentation; Comparison
Key Concept: 5.2.V.B; 5.2.V.C; 5.2.I.A; 5.2.I.B; 5.2.I.E; 5.2.II.A; 5.2.II.B

207
Copyright © 2017, 2015, 2011, 2007 Pearson Education, Inc. All rights reserved.
Section IV
Document-Based Question
Directions: The following question is based on the accompanying documents. The documents have been
edited for the purpose of this exercise.

In your response, you should do the following:

• State a relevant thesis that directly addresses all parts of the question.

• Support the thesis or a relevant argument with evidence from all, or all but one, of the
documents.

• Source and analyze the significance of at least four of the documents on the following basis:
intended audience, author’s purpose, historical context, format or medium, and/or point of view.

• Develop context by relating your argument to broader historical events or processes.

• Synthesize the elements above into a persuasive essay that extends your argument by connecting
it to:

• a different historical period or geographical region


• another course theme or historical approach that is not the focus of the essay (such as
political, social, economic, or intellectual history)
• another perspective from a different academic discipline (such as economics, art history,
anthropology, or government)

208
Copyright © 2017, 2015, 2011, 2007 Pearson Education, Inc. All rights reserved.
Question 5.65: Using the documents and your knowledge of world history, analyze the factors that led
to European imperialism during the nineteenth century.

Historical Background: Over the course of the nineteenth century, European nation-states began to look
for places around the globe to establish new colonies. As older colonies gained their independence, new
regions of the world proved attractive to European governments as potential new colonies.

Document 1
Denis Diderot (Eighteenth-Century French Philosopher), Supplément au Voyage de Bougainville, 1772
This country belongs to you. Why? Because you have set foot on it? If a Tahitian landed one day on your
coast and he engraved on one of your rocks or on the bark of a tree: “This country belongs to the
inhabitants of Tahiti,” what would you think?

Document 2
Lord Cromer (British Government Agent), “The Government of the Subject Races,” 1882
We need not always enquire too closely what these people, who are all, nationally speaking, more or
less in statu pupillari [student status], themselves think is in their own interests . . . it is essential that
each special issue should be decided mainly with reference to what, by the light of western knowledge
and experience . . . we conscientiously think is best for the subject race.

Document 3
Clement Hill (British Foreign Office Official), Foreign Office Memorandum, December 9, 1884
The geographical position of the East Coast lays it more within the general area of our foreign policy
than that of the West Coast. . . . Our alternative route by the Cape to India may at any time make it
important that we should have possession of or at least free access to good harbors. The importance is
not less since the French move to Madagascar. Is it not worth considering whether in view of the
European race for territories on the West Coast . . . we might not confine ourselves to securing the
utmost possible freedom of trade on that coast, yielding to other Powers the territorial responsibilities .
. . and seeking compensation on the East Coast where . . . we are at present, but who can say for how
long, without a European rival; where the political future of the country is of real importance to Indian
and Imperial interests; where the climate is superior; where commerce is capable of vast extension, and
where our influence could be exercised . . . in the extension of civilization, and the consequent
extinction of the Slave Trade for which we have so long labored.

Document 4
Pinheiro Chagas (Foreign Minister), Decree Following the Proclamation of a Portuguese Protectorate over Dahomey, 1886
The monarch of Dahomey, the terror of all Europeans, shows himself affable and condescending to the
Portuguese alone. It was he who asked urgently that our protectorate should be established upon the
coast of his kingdom. . . . It is he who appears disposed to accept at our hands the benefits of European
civilization, and to this design he has already borne honorable witness by abolishing human sacrifices.
The Portuguese protectorate in Dahomey is the lighted pathway that links this kingdom of darkness with
Europe.

209
Copyright © 2017, 2015, 2011, 2007 Pearson Education, Inc. All rights reserved.
Document 5
G. W. Steevens (British Journalist), Daily Mail, June 23, 1897
Up they came, more and more, new types, new realms at every couple of yards, an anthropological
museum—a living gazetteer of the British Empire. With them came their English officers, whom they
obey and follow like children. And you begin to understand, as never before, what the Empire amounts
to. Not only that we possess all these remote outlandish places . . . but also that all these people are
working, not simply under us, but with us—that we send out a boy here and a boy there, and a boy
takes hold of the savages of the part he comes to, and teaches them to march and shoot as he tells
them, to obey him and believe in him and die for him and the Queen. . . . A plain, stupid, uninspired
people, they call us, and yet we are doing this with every kind of savage man there is. And each one of
us—you and I, and that man in his shirt-sleeves at the corner—is a working part of this world-shaping
force. How small you must feel in face of this stupendous whole, and yet how great to be a unit in it.

Document 6
Karl Pearson (English Author), National Life from the Standpoint of Science,1907
History shows me one way, and one way only, in which a state of civilization has been produced,
namely, the struggle of race with race, and the survival of the physically and mentally fitter race.
This dependence of progress on the survival of the fitter race, terribly black as it may seem to some of
you, gives the struggle for existence its redeeming features; it is the fiery crucible out of which comes
the finer metal. You may hope for a time when the sword shall be turned into the ploughshare, when
American and German and English traders shall no longer compete in the markets of the world for raw
materials, for their food supply, when the white man and the dark shall share the soil between them,
and each till it as he lists . . . . The path of progress is strewn with the wreck of nations; traces are
everywhere to be seen of the hecatombs of inferior races, and of victims who found not the narrow way
to the greater perfection. Yet these dead peoples are, in very truth, the stepping stones on which
mankind has arisen to the higher intellectual and deeper emotional life of today.

Sample Answer

An acceptable thesis would consist of a direct answer to the question concerning the factors that led to
imperialism and might include arguments about causes, including trade, a belief in the survival of the
fittest, a belief in the “civilizing” nature of imperialism, and other related factors. The thesis must also
include analysis of the historical complexity of the topic by illustrating relationships among historical
evidence, including contradiction, corroboration and/or qualification. For example, a student might
write: “Although each of these was an important factor that led to imperialism, the access to harbors
and economic benefits that came from trade were the most important factors for the Europeans.” The
historically defensible claim that directly answers the question is worth one point and the statement
that develops historical complexity is worth an additional point.

To earn both points in the document analysis category, students need to relate at least five of the
documents to the thesis and analyze at least four of the documents by addressing the historical context,
audience, purpose, or point of view of each document. Document 1 questions the European mindset
that mere arrival justified control and came from an Enlightenment mindset about the rights that people
should have. Document 2 justifies imperialism by arguing that Europeans believe that they have superior
knowledge to those that they are colonizing; the purpose of the author is to defend the actions

210
Copyright © 2017, 2015, 2011, 2007 Pearson Education, Inc. All rights reserved.
Europeans are taking. Document 3 outlines specific economic advantages that can be gained through
imperialism and is written in the context of ongoing competition for India and Indian Ocean control. In
Document 4, the author reports that the people of Dahomey actually want to become a protectorate of
the Portuguese; however, the Portuguese foreign minister issued the proclamation to justify the colonial
actions the Portuguese had already taken. Document 5 speaks to the glory of empire building and
argues that colonization is actually beneficial for those who were colonized. The author of Document 5
is writing to a British audience and justifying to them why colonization is a good course of action for
them to take. Karl Pearson, in Document 6, defends imperialism by drawing on racial ideologies, like
survival of the fittest, to try and develop a “scientific” rationale for the actions of imperializing nations.

Students have numerous opportunities to earn one point for inclusion of evidence beyond the
documents. To earn this point, students might include other motives for imperialism, including
industrialization (raw materials, markets for finished goods), competition and rivalry among European
nations, Darwinism as the origin of Social Darwinism, basic ideas of the Enlightenment that might have
informed such thinkers as Diderot, and other writers that addressed imperialism (Kipling, e.g.).

One way the contextualization point could be earned is by discussing the loss of most of the colonies in
the Western Hemisphere or by addressing how the formation of new nation-states led to increased
competition. Another way students could demonstrate contextualization would be to discuss how
imperialism was a continuation of a colonial process that had begun in the sixteenth century.

The synthesis point could be gained by demonstrating how colonization in the sixteenth and
seventeenth centuries was a parallel event to nineteenth-century imperialism. A student could also
analyze how non-European states, such as Japan and/or the United States, also participated in
imperialism.

Topic: Factors that led to Imperialism


Theme: Interaction Between Humans and Environment; Development and Interaction of Cultures; State
Building, Expansion, and Conflict; Creation, Expansion, and Interaction of Economic Structures;
Development and Transformation of Social Structures
Learning Objective: ENV–9; CUL–3; CUL–4; SB–1; SB–2; SB–3; SB–9; SB–10; ECON–4; ECON–48; SOC–7
Historical Thinking Skill: Analyzing Evidence: Content and Sourcing; Interpretation; Causation;
Periodization
Key Concept: 5.2.I.A; 5.2.I.B; 5.2.I.C; 5.2.I.D; 5.2.I.E; 5.2.III

211
Copyright © 2017, 2015, 2011, 2007 Pearson Education, Inc. All rights reserved.
Question 5.66. Using the documents and your knowledge of world history, analyze the motives for
revolutionary movements in the late eighteenth and early nineteenth centuries.

Historical Background: The 1450 C.E.to 1750 C.E. era saw the creation of European colonies in the
Western Hemisphere and the establishment of such political ideas as absolutism in Europe. By the end of
the eighteenth century, pressure was mounting against these trends, which ultimately culminated in a
wave of revolutions across Europe and the Americas.

Document 1
Thomas Jefferson, Declaration of Independence, June 1776
NOTE: The following selection was deleted from the final draft of the Declaration of Independence by the Second Continental
Congress. Emphasis is in the original.

He [King George III of England] has waged cruel war against human nature itself, violating its most
sacred rights of life and liberty in the persons of a distant people who never offended him, captivating
and carrying them into slavery in another hemisphere, or to incur miserable death in their
transportation hither. . . . [A]nd . . . he is now exciting those very people to rise in arms among us, and to
purchase that liberty of which he had deprived them, by murdering the people upon whom he also
obtruded them: thus paying off former crimes committed against the liberties of one people, with
crimes which he urges them to commit against the lives of another.

Document 2
Thomas Jefferson, Declaration of Independence, June 1776
NOTE: The following selections were not deleted from the final draft of the Declaration of Independence and are found in the
Declaration of Independence at signing on July 4, 1776.

For cutting off our trade with all parts of the world;

For imposing taxes on us without our consent;

For taking away our charters, abolishing our most valuable laws, and altering fundamentally the forms
of our governments;

He [King George III] has abdicated government here, by declaring us out of his protection and waging
war against us.

A prince, whose character is thus marked by every act which may define a tyrant, is unfit to be the ruler
of a free people. . . .

We, therefore, the representatives of the United States of America, in General Congress assembled,
appealing to the Supreme Judge of the world for the rectitude of our intentions, do, in the name and by
the authority of the good people of these colonies, solemnly publish and declare, That these United
Colonies are, and of right ought to be, FREE AND INDEPENDENT STATES; that they are absolved from all
allegiance to the British crown and that all political connection between them and the state of Great
Britain is, and ought to be, totally dissolved; and that as free and independent states, they have full
power to levy war, conclude peace, contract alliances, establish commerce, and do all other acts and
things which independent states may of right do.

212
Copyright © 2017, 2015, 2011, 2007 Pearson Education, Inc. All rights reserved.
Document 3
French National Assembly, Declaration of the Rights of Man and Citizen, 1789
Therefore the National Assembly recognizes and proclaims, in the presence and under the auspices of
the Supreme Being, the following rights of man and of the citizen. . . .

6. Law is the expression of the general will. Every citizen has a right to participate personally, or
through his representative, in its formation. It must be the same for all, whether it protects or
punishes. All citizens, being equal in the eyes of the law, are equally eligible to all dignities and to
all public positions and occupations, according to their abilities, and without distinction except
that of their virtues and talents.

7. No person shall be accused, arrested, or imprisoned except in the cases and according to the
forms prescribed by law. . . .

13. A common contribution is essential for the maintenance of the public forces and for the cost of
administration. This should be equitably distributed among all the citizens in proportion to their
means.

Document 4
French Engraving, 1789

213
Copyright © 2017, 2015, 2011, 2007 Pearson Education, Inc. All rights reserved.
Document 5
Haitian Constitution of 1801
Art. 3. – There cannot exist slaves on this territory, servitude is therein forever abolished. All men are
born, live and die free and French.

Art. 4. – All men, regardless of color, are eligible to all employment.

Art. 5. – There shall exist no distinction other than those based on virtue and talent, and other
superiority afforded by law in the exercise of a public function. The law is the same for all whether in
punishment or in protection.

Art. 28 – The Constitution nominates the citizen Toussaint-L’Ouverture, Chief General of the army of
Saint-Domingue, and, in consideration for important services rendered to the colony, in the most critical
circumstances of the revolution, and upon the wishes of the grateful inhabitants, he is entrusted the
direction thereof for the remainder of his glorious life.

Document 6
Jean Jacques Dessallines, Announcement of Independence of St. Domingo (Haiti), 1803
In the name of the Black People, and the Men of Color of St. Domingo: The Independence of St.
Domingo is proclaimed. Restored to our primitive dignity, we have asserted our rights; we swear never
to yield them to any power on earth; the frightful veil of prejudice is torn to pieces, be it so forever.
Woe be to them who would dare to put together its bloody tatters. . . .

We have sworn not to listen with clemency towards all those who would dare to speak of slavery; we
will be inexorable, perhaps even cruel, towards all troops who, themselves forgetting the object for
which they have not ceased fighting since 1780, should come from Europe to bring among us death and
servitude. Nothing is too dear, and all means are lawful to men from whom it is wished to tear the first
of all blessings.

Document 7
Simón Bolívar, Advice to the Congress of Angostura (Argentina), 1819
We are not Europeans; we are not Indians; we are but a mixed species of aborigines and Spaniards.
Americans by birth and Europeans by law, we find ourselves engaged in a dual conflict: we are disputing
with the natives for titles of ownership, and at the same time we are struggling to maintain ourselves in
the country that gave us birth against the opposition of the invaders. . . .

Venezuela had, has, and should have a republican government. Its principles should be the sovereignty
of the people, divisions of powers, civil liberty, proscription of slavery, and the abolition of monarchy
and privileges.

We need equality to recast, so to speak, into a unified nation, the classes of men, political opinions, and
public customs. . . .

If the Senate were hereditary rather than elective, it would, in my opinion, be the basis, the tie, the very
soul of our republic. In political storms this body would arrest the thunderbolts of the government and
would repel any violet popular reaction.

214
Copyright © 2017, 2015, 2011, 2007 Pearson Education, Inc. All rights reserved.
Sample Answer

An acceptable thesis would consist of a direct answer to the question concerning the motives of those
calling for revolutions and might include freedom, independence, a fair government, equality before the
law, abolition, and other related factors. The thesis must also include analysis of the historical
complexity of the topic by illustrating relationships among historical evidence, including contradiction,
corroboration and/or qualification. For example, students might write: “Although each of these was an
important factor that contributed to revolutionary upheaval, the desire for independence and equality
for all citizens were the most important reasons for revolution.” The historically defensible claim that
directly answers the question is worth one point and the statement that develops historical complexity
is worth an additional point.

To earn both points in the document analysis category, students will need to relate at least six of the
documents to the thesis and analyze at least four of the documents by addressing historical context,
audience, purpose, or point of view of each document. Document 1 argues that the British have denied
the American colonists life and liberty and was written within the historical context of colonial
restrictions, rising taxes, and limited representation. Document 2 criticizes British trade limitations. In
this document, Jefferson calls the king a tyrant, and it is clear that the Declaration of Independence is
intended for the King of England. Document 3 is from French revolutionaries and calls for equality
before the law, due process, and a government based on the general will of the people. These
revolutionaries were heavily influenced by the Enlightenment, and Enlightenment ideas are seen in the
calls for governance by the general will and equality before the law. Document 4 is a critique of the
social structure in France and the social inequality that accompanied it. Document 4 was created on the
eve of the French Revolution and reflected its social context. Document 5 calls for the abolition of
slavery in Haiti and argues for the extension of equality under the law to those living in Haiti. The
purpose of Document 5 is to create the governmental foundation for the new nation. Document 6 also
discusses the rights of those living in Haiti and vehemently denies any toleration for slavery in Haiti.
Document 6 is a formal declaration of independence and is intended to convey, both to foreign and
domestic parties, the basic principles that the new nation is being founded on. In Document 7, Simón
Bolívar discusses the need to define identity and pursue equality in the new state of Venezuela. As the
leader of an independence movement, he feels the need to defend and define what he has fought for.

Students have numerous opportunities to earn the point for including evidence beyond the documents.
To earn this point, students might include a discussion of specific Enlightenment thinkers and ideas that
contributed to revolutions. Students might also discuss additional causes of the American and French
revolutions beyond the documents, such as French debt and food shortages and salutatory neglect in
America. Furthermore, students might discuss European events that contributed to other Latin
American revolutions, such as Napoleon’s removal of the Spanish king and the French Revolution.

One way students could earn the contextualization point is by discussing the impact and influence of the
ideas of the Enlightenment and the way that the Enlightenment challenged old governments and social
structures. Students might also discuss how European politics and colonial governance of the eighteenth
century—the Seven Years’ War, diplomacy, etc.—created the framework for the “Age of Revolutions.”

The synthesis point could be gained by discussing revolutions for independence in other time periods:
for instance, decolonization in the twentieth century or other revolutions in the twentieth century that
sought rights and/or governmental changes (e.g., the 1917 Russian Revolution, Mao in 1949). Students

215
Copyright © 2017, 2015, 2011, 2007 Pearson Education, Inc. All rights reserved.
could also reference revolutions and revolts in other time periods, such as the Roman Republic’s
overthrow of the Etruscans and the Ming expulsion of the Mongol/Yuan dynasty.

Topic: Causes of revolutionary action in Europe and the Americas


Theme: Development and Interaction of Cultures; State Building, Expansion, and Conflict; Creation,
Expansion, and Interaction of Economic Structures; Development and Transformation of Social
Structures
Learning Objective: CUL–2; CUL–3; CUL–4; CUL–7; SB–1; SB–2; SB–4; SB–7; ECON–7; SOC–1; SOC–2;
SOC–3; SOC–6; SOC–7
Historical Thinking Skill: Analyzing Evidence: Content and Sourcing; Interpretation; Causation;
Periodization
Key Concept: 5.3.I.A; 5.3.I B; 5.3.I.C; 5.3.II; 5.3.III.B; 5.3.III.C

216
Copyright © 2017, 2015, 2011, 2007 Pearson Education, Inc. All rights reserved.
Period 6
Accelerating Global Change and Realignments
(1900 to the Present)
Section I
Multiple-Choice Questions
Questions 6.1–6.3 refer to the image below.

Faisal and Delegation, Peace Conference, Versailles, 1919

6.1. Prince Faisal and the Arab delegation at the Paris peace conference in 1919 would have
__________.
(A) appreciated Woodrow Wilson’s interpretation of self-determination
(B) felt humiliated by broken promises
(C) followed in the footsteps of Kemal Ataturk and formed a modern Arabia
(D) left with control of the Arabian peninsula

Answer: B
Topic: War and Nationalist Movements in the Middle East
Theme: State Building, Expansion, and Conflict
Learning Objective: SB–6.2
Historical Thinking Skill: Interpretation
Key Concept: 6.2.I.C

217
Copyright © 2017, 2015, 2011, 2007 Pearson Education, Inc. All rights reserved.
6.2. Of the following countries, which had the least control over the Middle East in the years
immediately following World War I?
(A) Turkey, which lost influence with the demise of the Ottoman Empire
(B) Russia, which removed itself from the international scene to consolidate its Bolshevik
Revolution within its own borders
(C) France, which was weakened by fighting within its own boundaries
(D) Great Britain, which was unable to retain relations with the region because of the
destruction of its navy

Answer: B
Topic: The War Beyond Europe
Theme: State Building, Expansion, and Conflict
Learning Objective: SB–6.2
Historical Thinking Skill: Analyzing Evidence: Content and Sourcing
Key Concept: 6.2.II.C

6.3. How would most of the people in the photograph have reacted to the Balfour Declaration?
(A) They would have deemed it a humiliation to Arabs everywhere.
(B) They would have seen it as a positive move toward Arab nationalism.
(C) They would have seen it as a win for self-determination.
(D) They would have seen it as essential to the development of a pan-Arab movement.

Answer: A
Topic: War and Nationalist Movements in the Middle East
Theme: State Building, Expansion, and Conflict
Learning Objective: SB–6.2
Historical Thinking Skill: Contextualization
Key Concept: 6.2.I.C

218
Copyright © 2017, 2015, 2011, 2007 Pearson Education, Inc. All rights reserved.
Questions 6.4–6.6 refer the image and passage below.

Advertisement for Mitchell’s Golden Dawn


Cigarettes

Men began to fall one by one . . . . One officer said we were OK, all the machine guns were firing over
our heads. This was so until we passed our own front line and started to cross No Man’s Land. Then
trench machine guns began the slaughter from the [German positions]. Men fell on every side
screaming. Those who were unwounded dare not attend to them, we must press on regardless.
Hundreds lay on the German barbed wire which was not all destroyed and their bodies formed a bridge
for others to pass over and into the German front line.

Michael Kernan, “Day of Slaughter on the Somme,” Washington Post, June 27, 1976

6.4. Who would have most likely seen the above poster?
(A) A parishioner in a church in Berlin
(B) A young man waiting for the subway in London
(C) A Bolshevik fighting in the Red army
(D) A young woman working in a munitions plant near Paris

Answer: B
Topic: A World at War
Theme: State Building, Expansion, and Conflict
Learning Objective: SB–6.2
Historical Thinking Skill: Interpretation
Key Concept: 6.2.IV.A

219
Copyright © 2017, 2015, 2011, 2007 Pearson Education, Inc. All rights reserved.
6.5 Of the following technological elements of warfare, which was not a characteristic of the fighting
in World War I?
(A) blitzkrieg warfare
(B) machine guns
(C) submarines
(D) poison gas

Answer: A
Topic: A World at War
Theme: State Building, Expansion, and Conflict
Learning Objective: SB–6.1
Historical Thinking Skill: Analyzing Evidence: Content and Sourcing
Key Concept: 6.1.III.C

6.6 Which of the following conclusions can be correctly drawn from the tactical, strategic, and
technological aspects of the Great War?
(A) Air power determined the outcome of the war.
(B) Technology helped the defender more than the invader.
(C) The Allied powers were victorious because they controlled the seas.
(D) Germany was doomed because it was forced to fight a two-front war.

Answer: B
Topic: A World at War
Theme: State Building, Expansion, and Conflict
Learning Objective: SB–6.1
Historical Thinking Skill: Synthesis
Key Concept: 6.1.III.C

220
Copyright © 2017, 2015, 2011, 2007 Pearson Education, Inc. All rights reserved.
Questions 6.7–6.9 refer to the passage below.

We Han are now swiftly being caught up in a tidal wave of nationalist revolution, yet the Manchus
continue to discriminate against the Han. They boast that their forefathers conquered the Han because
of their superior unity and military strength and that they intend to retain these qualities so as to
dominate the Han forever. . . . Certainly, once we Han unite, our power will be thousands of times
greater than theirs, and the sources of the nationalist revolution will be assured.

As for the principle of Democracy, it is the foundation of the political revolution. . . . For several
thousand years China has been a monarchical autocracy, a type of political system intolerable to those
living in freedom and equality. A nationalist revolution is not itself sufficient to get rid of such a system.
Think for a moment: When the founder of the Ming dynasty expelled the Mongols and restored Chinese
rule, the nationalist revolution triumphed, but his political system was only too similar to those of the
Han, Tang, and Song dynasties. Consequently, after another three hundred years, foreigners again
began to invade China. This is the result of the inadequacy of the political system, so that a political
revolution is an absolute necessity. . . .

Sun Yat-sen, from a Speech to the United League, Tokyo, 1906

6.7. What was the goal of Sun Yat-sen?


(A) to expel western thinking from Chinese life
(B) to overthrow the Manchu and introduce democracy
(C) to bring communism to China
(D) to replace the Manchu dynasty with the Qing dynasty

Answer: B
Topic: The Fall of the Qing and the Rise of a Chinese Nationalist Alternative
Theme: State Building, Expansion, and Conflict
Learning Objective: SB–6.2
Historical Thinking Skill: Interpretation
Key Concept: 6.2.I.A

6.8. What important ism is reflected in the passage?


(A) nationalism, or a sense of national pride
(B) collectivism, as demonstrated by China’s Great Leap Forward
(C) capitalism, as expressed by Adam Smith
(D) imperialism, as demonstrated by Cecil Rhodes

Answer: A
Topic: The Fall of the Qing and the Rise of a Chinese Nationalist Alternative
Theme: State Building, Expansion, and Conflict
Learning Objective: SB–6.2
Historical Thinking Skill: Interpretation
Key Concept: 6.2.I.A

6.9. With whose ideas would Sun Yat-sen have most agreed?
(A) with Ho Chi-minh, as he saw to free Vietnam from western imperialism

221
Copyright © 2017, 2015, 2011, 2007 Pearson Education, Inc. All rights reserved.
(B) with Chiang Kai-shek, as he fought communism
(C) with Mao Zedong and the power of the peasantry
(D) with Thomas Jefferson and his views on democracy

Answer: D
Topic: The Fall of the Qing and the Rise of a Chinese Nationalist Identity
Theme: State Building, Expansion, and Conflict
Learning Objective: SB–6.2
Historical Thinking Skill: Comparison
Key Concept: 6.2.I.A

222
Copyright © 2017, 2015, 2011, 2007 Pearson Education, Inc. All rights reserved.
Questions 6.10–6.13 refer to the image below.

Russian Children at the Beginning of Stalin’s Collectivization Plan, 1931

6.10. As part of the Soviet Union’s collectivization program, Stalin encouraged __________.
(A) democratically elected regional officials
(B) the importation of workers from China
(C) the exodus of urban workers to rural regions
(D) the development of communal land ownership

Answer: D
Topic: Soviet Experimentation
Theme: State Building, Expansion, and Conflict
Learning Objective: SB–6.3
Historical Thinking Skill: Analyzing Evidence: Content and Sourcing
Key Concept: 6.3.I.A

6.11. Which of the following statements about collectivization is true?


(A) Collectivization resulted in greater gender equality in the Soviet Union.
(B) Collectivization was endorsed by V. I. Lenin.
(C) Collectivization resulted in the death of millions of peasants.
(D) Collectivization shifted land ownership from one elite to another.

Answer: C
Topic: Soviet Experimentation
Theme: Creation, Expansion, and Interaction of Economic Systems
Learning Objective: ECON–6.3
Historical Thinking Skill: Causation
Key Concept: 6.3.I.A

223
Copyright © 2017, 2015, 2011, 2007 Pearson Education, Inc. All rights reserved.
6.12. The most successful aspect of the Soviet collectivization program was
(A) stunning and immediate increases in agricultural production
(B) rural workers moving to the cities to aid in the industrialization process
(C) the export of grains to European countries hurt by the Depression of the 1930s
(D) the freeing of the kulaks to produce on their own farms

Answer: B
Topic: Soviet Experimentation
Theme: Creation, Expansion, and Interaction of Economic Systems
Learning Objective: ECON–6.3
Historical Thinking Skill: Argumentation
Key Concept: 6.3.I.A

6.13. For which audience was the photograph most likely intended?
(A) for farmers seeking increased productivity
(B) for disgruntled industrial workers moving back to the countryside
(C) for potential immigrants from eastern Europe
(D) for members of the Soviet Communist party

Answer: A
Topic: Soviet Experimentation
Theme: Creation, Expansion, and Interaction of Economic Systems
Learning Objective: ECON–6.3
Historical Thinking Skill: Interpretation
Key Concept: 6.3.I.A

224
Copyright © 2017, 2015, 2011, 2007 Pearson Education, Inc. All rights reserved.
Questions 6.14–6.16 refer to the image below.

Photo of Nazi Rally Saluting Adolph Hitler, Mid-1930s

6.14. Which of the following attributes would be least characteristic of the Nazi totalitarian state?
(A) centralized economic planning
(B) appeals to the nationalistic fervor of the German people
(C) disregard for the Versailles Treaty ending World War I
(D) support for trade unions that would empower the working class

Answer: D
Topic: The Rise of Nazism
Theme: State Building, Expansion, and Conflict
Learning Objective: SB–6.2
Historical Thinking Skill: Analyzing Evidence: Content and Sourcing
Key Concept: 6.2.V.C

6.15. Which of the following would characterize Adolph Hitler as a conservative force in German
politics?
(A) his reverence for the German High Command in World War I
(B) his view of Western governments as enlightened
(C) his firm stance against communism and socialism
(D) his love of modern economic systems

Answer: C
Topic: The Rise of Nazism
Theme: State Building, Expansion, and Conflict
Learning Objective: SB–6.2
Historical Thinking Skill: Contextualization
Key Concept: 6.2.V.C

225
Copyright © 2017, 2015, 2011, 2007 Pearson Education, Inc. All rights reserved.
6.16. Of the following leaders, who would Hitler least respect?
(A) Benito Mussolini, because of his love of the fascist state in Italy
(B) Francisco Franco, because of his fight against the communist insurgency in Spain
(C) Getulio Vargas, because of his drive to centralize the government in Brazil
(D) Joseph Stalin, because of his role in the development of a communist state in the Soviet
Union

Answer: D
Topic: New Authoritarianism: The Rise of Fascism; The Spread of Fascism and the Spanish Civil War; The
Vargas Regime in Brazil
Theme: State Building, Expansion, and Conflict
Learning Objective: SB–6.2
Historical Thinking Skill: Comparison
Key Concept: 6.2.V.C

226
Copyright © 2017, 2015, 2011, 2007 Pearson Education, Inc. All rights reserved.
Questions 6.17–6.19 refer to the passage below.

SOURCE: Zenshiro Hoshina, trans. Hikaru Tashima, “Hoshina Memorandum” on the Emperor’s “Sacred Decision [go-seidan],” in
National Security Archive, ed. William Burr, The Atom Bomb and the End of World War II: A Collection of Primary Sources,
National Security Archive Electronic Briefing Book, No. 162, Document 62, http://nsarchive.gwu.edu/NSAEBB/NSAEBB162/.

Despite the bombing of Hiroshima, the Soviet declaration of war, and growing worry about domestic
instability, the Japanese cabinet (whose decisions required unanimity) could not form a consensus to
accept the Potsdam Declaration. Members of the Supreme War Council—“the Big Six”—wanted the
reply to Potsdam to include at least four conditions (e.g., no occupation, voluntary disarmament); they
were willing to fight to the finish. The peace party, however, deftly maneuvered to break the stalemate
by persuading a reluctant emperor to intervene. According to Hasegawa, Hirohito had become
convinced that the preservation of the monarchy was at stake. Late in the evening of 9 August, the
emperor and his advisers met in the bomb shelter of the Imperial Palace.

Zenshiro Hoshina, a senior naval official, attended the conference and prepared a detailed account.
With Prime Minister Suzuki presiding, each of the ministers had a chance to state his view directly to
Hirohito. While Army Minister Anami tacitly threatened a coup (“civil war”), the emperor accepted the
majority view that the reply to the Potsdam declaration should include only one condition not the four
urged by “Big Six.” Nevertheless, the condition that Hirohito accepted was not the one that foreign
minister Togo had brought to the conference. What was at stake was the definition of the kokutai
(national policy). Togo’s proposal would have been generally consistent with a constitutional monarchy
because it defined the kokutai narrowly as the emperor and the imperial household. What Hirohito
accepted, however, was a proposal by the extreme nationalist Kiichiro Hiranuma which drew upon
prevailing understandings of the kokutai: the “mythical notion” that the emperor was a living god. “This
was the affirmation of the emperor’s theocratic powers, unencumbered by any law, based on Shinto
gods in antiquity, and totally incompatible with a constitutional monarchy.” Thus, the Japanese
response to the Potsdam declaration opposed “any demand which prejudices the prerogatives of his
Majesty as a sovereign ruler.” This proved to be unacceptable to the Truman administration.

Zenshiro Hoshina, “Hoshina Memorandum” on the Emperor’s “Sacred Decision [go-seidan],” August 9-10, 1945

6.17. How would you characterize the tone of army minister Anami?
(A) defiance in the face of overwhelming odds
(B) resignation to defeat
(C) disgust at Japanese leadership
(D) fear of enemy (Allied) reprisals

Answer: A
Topic: The Rise and Fall of the Japanese Empire in the Pacific War
Theme: State Building, Expansion, and Conflict
Learning Objective: SB–6.2
Historical Thinking Skill: Interpretation
Key Concept: 6.2.IV.C

6.18. Japan agreed to “unconditional surrender” only after the Allies did what?
(A) allowed the Japanese to retain their emperor in an official capacity
(B) fire-bombed Tokyo

227
Copyright © 2017, 2015, 2011, 2007 Pearson Education, Inc. All rights reserved.
(C) limited the Allied occupation of Japan to a maximum of five years
(D) surrendered control of Irkutsk to Japan

Answer: A
Topic: The Rise and Fall of the Japanese Empire in the Pacific War
Theme: State Building, Expansion, and Conflict
Learning Objective: SB–6.2
Historical Thinking Skill: Historical Causation
Key Concept: 6.2.IV.A

6.19. With whom does the loyalty of the author of the passage lie?
(A) with Japan’s military leaders
(B) with Japan’s emperor
(C) with Japan’s samurai soldiers
(D) with the victorious Allied forces

Answer: B
Topic: The Rise and Fall of the Japanese Empire in the Pacific War
Theme: State Building, Expansion, and Conflict
Learning Objective: SB–6.2
Historical Thinking Skill: Interpretation
Key Concept: 6.2.IV.A

228
Copyright © 2017, 2015, 2011, 2007 Pearson Education, Inc. All rights reserved.
Questions 6.20–6.22 refer to the images below.

Nkrumah of Ghana Nasser of Egypt Gandhi of India Ho of Vietnam

6.20. Independence resulted in the formation of new boundaries in which of the following countries?
(A) in Ghana, which gained land from neighboring Nigeria
(B) in Egypt, which gained the Sinai Peninsula from Israel
(C) in the British colony of India, which became India and East and West Pakistan
(D) in Vietnam, which remained two separate nations

Answer: C
Topic: Nationalism and Decolonization in South and Southeast Asia and Africa
Theme: State Building, Expansion, and Conflict
Learning Objective: SB–6.2
Historical Thinking Skill: Comparison
Key Concept: 6.2.I.B; 6.2.II.A; 6.2.II.C

6.21. To achieve independence from colonial powers, which strategy was used by all of the leaders in
the photographs?
(A) the boycotting of interactions with European nations
(B) the adherence to the communist ideologies of the Soviet Union
(C) the invocation of nationalistic pride in the citizens of their countries
(D) the pursuit of collective socialism in their nation’s economies

Answer: C
Topic: Egypt and the Rise of Nationalism in the Middle East; The Winning of Independence in South and
Southeast Asia; Military Responses: Dictatorships and Revolutions; The War of Liberation Against the
French
Theme: State Building, Expansion, and Conflict
Learning Objective: SB–6.2
Historical Thinking Skill: Analyzing Evidence: Content and Sourcing
Key Concept: 6.2.I.B; 6.2.II.A

6.22. Why were colonies so successful in gaining independence after World War II?
(A) because decolonization was a condition of the peace treaty that ended the war

229
Copyright © 2017, 2015, 2011, 2007 Pearson Education, Inc. All rights reserved.
(B) because European powers felt the need to thank their colonies for the contributions they
made to winning the war
(C) because a wave of support for democratic principles spread across the world
(D) because the newly created United Nations demanded it

Answer: C
Topic: Nationalism and Decolonization in South and Southeast Asia and Africa
Theme: State Building, Expansion, and Conflict
Learning Objective: SB–6.2
Historical Thinking Skill: Causation
Key Concept: 6.2.I.B

230
Copyright © 2017, 2015, 2011, 2007 Pearson Education, Inc. All rights reserved.
Questions 6.23–6.25 refer to the map below.

Soviet and Eastern European Boundaries by 1948

6.23. Which country’s boundaries remained unchanged following World War II?
(A) Poland, whose borders remained as they had been since the reign of Catherine the Great
(B) Finland, which was able to resist territorial incursions by the Soviet Union
(C) Germany, which was able to prevent the sorts of territorial losses it had experienced at the
end of World War I
(D) France, which retained territory it had gained in World War I

Answer: D
Topic: The Cold War
Theme: State Building, Expansion, and Conflict
Learning Objective: SB–6.2
Historical Thinking Skill: Analyzing Evidence: Content and Sourcing
Key Concept: 6.2.IV.D

6.24. As a response to western Europe’s creation of NATO, the Soviet Union and its eastern bloc
satellites formed a similar alliance. What was it?
(A) the European Union

231
Copyright © 2017, 2015, 2011, 2007 Pearson Education, Inc. All rights reserved.
(B) the International Atomic Energy Commission
(C) the Warsaw Pact
(D) the Common Market

Answer: C
Topic: The Cold War
Theme: State Building, Expansion, and Conflict
Learning Objective: SB–6.2
Historical Thinking Skill: Causation
Key Concept: 6.2.IV.D

6.25. Of the following, which was not intended to stop Soviet aggression in post–World War II Europe?
(A) the Brezhnev Doctrine
(B) the Marshall Plan
(C) the European Economic Community
(D) the military buildup in West Germany

Answer: A
Topic: The Cold War; Toward Greater European Unity
Theme: State Building, Expansion, and Conflict; Creation, Expansion, and Interaction of Economic
Systems
Learning Objective: SB–6.2; ECON–6.3
Historical Thinking Skill: Causation
Key Concept: 6.2.IV.D; 6.3.II.B

232
Copyright © 2017, 2015, 2011, 2007 Pearson Education, Inc. All rights reserved.
Questions 6.26–6.28 refer to the image below.

Fidel Castro and His “Barbudos” (Bearded) Guerillas

6.26. The Cuban Revolution was part of a larger revolution in Latin America calling for the end of
__________.
(A) the power of the Catholic Church
(B) economic colonialism by the United States
(C) communist intrusion into Latin America
(D) restrictive trade policies

Answer: B
Topic: The Cuban Revolution: Socialism in the Caribbean
Theme: State Building, Expansion, and Conflict
Learning Objective: SB–6.2
Historical Thinking Skill: Contextualization
Key Concept: 6.2.V.B

6.27. Of the following individuals, who would be most opposed to the success of Castro in Cuba?
(A) a Cuban cigar maker
(B) the owner of a Havana casino
(C) a worker in a Cuban sugar mill
(D) a member of the Sandinista party in Nicaragua

Answer: B
Topic: The Cuban Revolution: Socialism in the Caribbean
Theme: State Building, Expansion, and Conflict

233
Copyright © 2017, 2015, 2011, 2007 Pearson Education, Inc. All rights reserved.
Learning Objective: SB–6.3
Historical Thinking Skill: Comparison
Key Concept: 6.2.V.C

6.28. Identify an example of twentieth-century economic colonialism in Latin America.


(A) the perpetuation of slavery in the Caribbean
(B) U.S. control of sugar production in Cuba
(C) the formation of NAFTA
(D) U.S. dependence on illicit drugs from Latin America

Answer: B
Topic: Latin America After WWII; Institutions of Globalization
Theme: Creation, Expansion, and Interaction of Economic Systems
Learning Objective: ECON–6.3
Historical Thinking Skill: Analyzing Evidence: Content and Sourcing
Key Concept: 6.3.II.E

234
Copyright © 2017, 2015, 2011, 2007 Pearson Education, Inc. All rights reserved.
Questions 6.29–6.31 refer to the table below.

Population of Capital Cities as a Percentage of Total Population in 10 Latin American Nations


Nation Capital 1880 1930 1960 1983 2012
Argentina Buenos Aires 12 20 32 34 32
Brazil Rio de Janeiro 3 4 7 4* 3*
Chile Santiago 6 13 22 37 31
Colombia Bogotá 1 2 8 11 15
Cuba Havana 13 15 18 20 20
Mexico Mexico City 3 5 15 20 16
Panama Panama City 7 16 25 20 12
Peru Lima 3 5 19 27 27
Uruguay Montevideo 12 28 31 40 40
Venezuela Caracas 3 7 20 18 7
*No longer the capital city.

SOURCES: From J. P. Cole, Latin America: An Economic and Social Geography (Washington, D.C.: Butterworth, 1965), 417;
http://www.world-gazetteer.com.

6.29. What is the dominant population trend in Latin American cities between 1983 and 2012?
(A) The urban population increased.
(B) Population fell because of urban pollution.
(C) Diversified economies expanded employment opportunities.
(D) Cities generally declined in the percentage of their populations relative to their country’s
overall population.

Answer: D
Topic: The Movement of People
Theme: Interaction Between Humans and the Environment
Learning Objective: ENV–6.2
Historical Thinking Skill: Patterns of Continuity and Change over Time
Key Concept: 6.2.III.A

6.30. Which region of the world is currently the most urbanized?


(A) Asia
(B) Latin America
(C) Africa
(D) western Europe

Answer: D
Topic: The Movement of People
Theme: Interaction Between Humans and the Environment
Learning Objective: ENV–6.2
Historical Thinking Skill: Analyzing Evidence: Content and Sourcing
Key Concept: 6.2.III.A

6.31. Why did Latin America experience such a dramatic population increase in the last half of the
twentieth century?
(A) Contraceptive measures were banned.

235
Copyright © 2017, 2015, 2011, 2007 Pearson Education, Inc. All rights reserved.
(B) Latin America experienced dramatic increases in immigration from Asia and Africa.
(C) The United States and western Europe denied immigration requests to people from Latin
America.
(D) Mortality rates declined and fertility rates increased in Latin America.

Answer: D
Topic: The Movement of People
Theme: Interaction Between Humans and the Environment
Learning Objective: ENV–6.2
Historical Thinking Skill: Causation
Key Concept: 6.2.III.A

236
Copyright © 2017, 2015, 2011, 2007 Pearson Education, Inc. All rights reserved.
Questions 6.32–6.34 refer to the map below.

The Partition of South Asia

6.32. Which region on the map is claimed by both Pakistan and India?
(A) East Pakistan
(B) Kashmir
(C) Bangladesh
(D) Ceylon

Answer: B
Topic: The Challenges of Independence
Theme: State Building, Expansion, and Conflict
Learning Objective: SB–6.2
Historical Thinking Skill: Analyzing Evidence: Content and Sourcing
Key Concept: 6.2.III.A

6.33. The boundaries between India and the two Pakistans were established by __________.
(A) Great Britain
(B) the United Nations
(C) the Hindu majority in India
(D) the Muslim majority in Pakistan

Answer: A

237
Copyright © 2017, 2015, 2011, 2007 Pearson Education, Inc. All rights reserved.
Topic: The Challenge of Independence
Theme: State Building, Expansion, and Conflict
Learning Objective: SB–6.2
Historical Thinking Skill: Analyzing Evidence: Content and Sourcing
Key Concept: 6.2.III.A

6.34. Which leader argued for a partition of India and the creation of a separate Muslim state after
World War II?
(A) Winston Churchill
(B) Mohandas Gandhi
(C) Muhammad Ali Jinnah
(D) Franklin Roosevelt

Answer: C
Topic: The Winning of Independence in South and Southeast Asia
Theme: State Building, Expansion, and Conflict
Learning Objective: SB–6.2
Historical Thinking Skill: Analyzing Evidence: Content and Sourcing
Key Concept: 6.2.III.A

238
Copyright © 2017, 2015, 2011, 2007 Pearson Education, Inc. All rights reserved.
Questions 6.35–6.37 refer to the chart below.

The Growth of the World Population by Major Geographic Areas, 1930–2000

6.35. Where, in the last half of the twentieth century, was population growth the most dramatic?
(A) South Asia
(B) Africa
(C) Latin America
(D) East Asia

Answer: A
Topic: Parasitic Species and Endangered Ecosystems
Theme: Interaction Between Humans and the Environment
Learning Objective: ENV–6.1
Historical Thinking Skill: Causation
Key Concept: 6.1.I.B

6.36. What factor was most responsible for limiting population growth in the West?
(A) the empowerment of women
(B) industrialization, which increased the size of the middle class
(C) legislation that limited family size
(D) secularization of society from religious beliefs

Answer: B
Topic: The Population Bomb
Theme: Creation, Expansion, and Interaction of Economic Systems
Learning Objective: ECON–6.1
Historical Thinking Skill: Causation
Key Concept: 6.1.I.A
239
Copyright © 2017, 2015, 2011, 2007 Pearson Education, Inc. All rights reserved.
6.37. Population expanded tremendously __________.
(A) in East Asia and South Asia
(B) in North America and the Soviet Union
(C) because a rising middle class is having more children
(D) because South Asia and Africa are becoming more industrialized

Answer: A
Topic: The Population Bomb
Theme: Development and Transformation of Social Structures
Learning Objective: SOC–6.1
Historical Thinking Skill: Contextualization
Key Concept: 6.1.I.A

240
Copyright © 2017, 2015, 2011, 2007 Pearson Education, Inc. All rights reserved.
Questions 6.38–6.40 refer to the passage below.

Brave wives and daughters-in-law, untrammeled by the presence of their menfolk, could voice their own
bitterness . . . encourage their poor sisters to do likewise, and thus eventually bring to the village-wide
gatherings the strength of “half of China” as the more enlightened women, very much in earnest, like to
call themselves. By “speaking pains to recall pains,” the women found that they had as many if not more
grievances than the men, and that given a chance to speak in public, they were as good at it as their
fathers and husbands.

Woman in Post–World War II Communist China

6.38. Who is “half of China?”


(A) Chinese migrants who are moving to cities
(B) Chinese citizens who are becoming literate
(C) Chinese women
(D) Chinese resistors to communist ideology

Answer: C
Topic: “Women Hold Up Half of the Heavens”
Theme: Development and Transformation of Social Structures
Learning Objective: SOC–6.3
Historical Thinking Skill: Interpretation
Key Concept: 6.3.III.A

6.39. With which goals is passage consistent?


(A) with the goals of Mongol rulers
(B) with the goals of the Ching dynasty
(C) with the goals of the Communist Revolution
(D) with the goals of western Europeans in the nineteenth century

Answer: C
Topic: “Women Hold Up Half the Heavens”
Theme: Development and Transformation of Social Structures
Learning Objective: SOC–6.3
Historical Thinking Skill: Contextualization
Key Concept: 6.3.III.A

6.40. The woman whose words appear in the passage would disagree with which traditional Chinese
value?
(A) the mandate of heaven
(B) filial piety
(C) the Middle Kingdom
(D) ancestry worship

Answer: B
Topic: Gender and Family Life

241
Copyright © 2017, 2015, 2011, 2007 Pearson Education, Inc. All rights reserved.
Theme: Development and Transformation of Social Structures
Learning Objective: SOC–6.3
Historical Thinking Skill: Comparison
Key Concept: 6.3.III.A

242
Copyright © 2017, 2015, 2011, 2007 Pearson Education, Inc. All rights reserved.
Questions 6.41–6.43 refer to the map below.

Germany After World War II

6.41. The Berlin Airlift of 1948–1949 __________.


(A) prevented Soviet control of East Germany
(B) linked Hamburg to the Polish Corridor
(C) was a successful response to Soviet aggression
(D) led to the decision to build the Berlin Wall

Answer: C
Topic: Dismantling the Soviet Empire
Theme: State Building, Expansion, and Conflict
Learning Objective: SB–6.2
Historical Thinking Skill: Analyzing Evidence: Content and Sourcing
Key Concept: 6.2.IV.E

6.42. Why was the Berlin Wall built?


(A) to prevent Middle Eastern refugees from entering the country
(B) to discourage trade between East and West Berlin
(C) to prevent the sabotage of Soviet space technology
(D) to prevent East German citizens from defecting to the West

Answer: D

243
Copyright © 2017, 2015, 2011, 2007 Pearson Education, Inc. All rights reserved.
Topic: The New Soviet Empire in Eastern Europe
Theme: State Building, Expansion, and Conflict
Learning Objective: SB–6.2
Historical Thinking Skill: Causation
Key Concept: 6.2.IV.C

6.43. What was a result of the division of Germany during the Cold War?
(A) Both East and West Germany remained economically stagnant.
(B) Germany became a lightning rod of friction between the Soviet Union and the West.
(C) The Warsaw Pact used East Germany as a launching pad into western Europe.
(D) Britain, France, and American troops occupied West Germany for decades.

Answer: B
Topic: The Cold War
Theme: State Building, Expansion, and Conflict
Learning Objective: SB–6.2
Historical Thinking Skill: Historical Causation
Key Concept: 6.2.IV.C

244
Copyright © 2017, 2015, 2011, 2007 Pearson Education, Inc. All rights reserved.
Questions 6.44–6.46 refer to the image below.

Cartoon Commentary on China

6.44. What does the cartoon suggest that China needs to do?
(A) to do more for its growing middle class
(B) to institute democratic reforms
(C) to nationalize the fast-food industry
(D) to attend to a growing obesity problem

Answer: B
Topic: Mao’s Last Campaign and the Fall of the Gang of Four
Theme: Creation, Expansion, and Interaction of Economic Systems
Learning Objective: ECON–6.2
Historical Thinking Skill: Interpretation
Key Concept: 6.2.V.B

6.45. To what is the Chinese economic success of the last 30 years primarily attributable?
(A) to increased agricultural production
(B) to massive foreign investment
(C) to the one-child policy

245
Copyright © 2017, 2015, 2011, 2007 Pearson Education, Inc. All rights reserved.
(D) to the application of capitalist economic principles

Answer: D
Topic: Mao’s Last Campaign and the Fall of the Gang of Four
Theme: Creation, Expansion, and Interaction of Economic Systems
Learning Objective: ECON–6.2
Historical Thinking Skill: Historical Causation
Key Concept: 6.2.V.B

6.46. The reforms of which individual paved the way for China’s recent economic success?
(A) Deng Xiaoping, who introduced market socialism
(B) Chiang Kai-shek, who introduced the Great Leap Forward
(C) Sun Yat-sen, who introduced the five-year-plan concept
(D) Mao Zedong, who introduced the “New Economic Policy”

Answer: A
Topic: Mao’s Last Campaign and the Fall of the Gang of Four
Theme: Creation, Expansion, and Interaction of Economic Systems
Learning Objective: ECON–6.2
Historical Thinking Skill: Analyzing Evidence: Content and Sourcing
Key Concept: 6.2.V.B

246
Copyright © 2017, 2015, 2011, 2007 Pearson Education, Inc. All rights reserved.
Section II
Short-Answer Questions
6.47. Use the table below and your knowledge of world history to answer all parts of the question that
follows.

Index of Industrial Production, 1929–1938 (1929 = 100)


Year Germany Japan Great Britain United States
1929 100.0 100.0 100.0 100.0
1930 85.9 94.8 92.3 80.7
1931 67.6 91.6 83.8 68.1
1932 53.3 97.8 83.5 53.8
1933 60.7 113.2 88.2 63.9
1934 79.8 128.7 98.8 66.4
1935 94.0 141.8 105.8 75.6
1936 106.3 151.1 115.9 88.1
1937 117.2 170.8 123.7 92.2
1938 127.0 174.8 113.4 72.3
SOURCE: League of Nations, Monthly Bulletin of Statistics, vol 20, p 12, pp 51–52. Used with
permission.

(A) Identify and briefly explain one impact that the Great Depression had on non-industrialized
countries that were dependent economically on trade with the industrialized countries
listed on the chart.
(B) Identify and briefly explain one successful policy used by the German and Japanese
governments to lift their countries out of the Great Depression.
(C) Identify and briefly explain one policy of Franklin Roosevelt’s New Deal that continues to
exist in the present.

Sample Answers

(A) Because factory production in industrialized nations was reduced, industrialized countries,
such as those listed in the table, did not demand raw materials typically supplied by non-
industrialized countries. Less demand for raw resources resulted in higher unemployment
and a harsher economic climate in non-industrialized countries.

(B) Both German and Japanese governments centralized industries by controlling production. In
both instances, they stimulated employment and their economies by producing war
matériel. Students may want to talk about the Japanese zaibatsu program and about the
development of industrial giants such as Mitsubishi, in Japan, and the Krupp ironworks, in
Germany.

(C) Students may provide examples of the increased size of government owing to New Deal
programs such as the National Recovery Act (NRA) and the Civilian Conservation Corps
(CCC). Two examples of programs that continue to exist today are the Federal Deposit
Insurance Corporation (FDIC) and the Social Security system.

247
Copyright © 2017, 2015, 2011, 2007 Pearson Education, Inc. All rights reserved.
Topic: The Global Great Depression
Theme: Expansion and Interaction of Economic Systems
Learning Objective: ECON–6.2
Historical Thinking Skill: Interpretation
Key Concept: 6.2.IV.B

248
Copyright © 2017, 2015, 2011, 2007 Pearson Education, Inc. All rights reserved.
6.48. Use the passage below and your knowledge of world history to answer all parts of the question
that follows.

Comrades, in accordance with a decision made by the Standing Committee of the CPC Central
Committee, the party Central Committee and the State Council have convened a meeting here of
cadres from party, government, and army organs at the central and municipal levels, calling on
everyone to mobilize in this emergency and to adopt resolute and effective measures to curb
turmoil in a clear-cut manner, to restore normal order in society, and to maintain stability and
unity in order to insure the triumphant implementation of our reform and open policy and the
program of socialist modernization [applause].

Li Peng (Communist Party Official), Statement Following the Tiananmen Square Demonstration, 1989

(A) Identify and explain one criticism that protesters voiced at the Tiananmen Square
demonstration.
(B) Identify and explain one justification that the Chinese government offered for the manner in
which they repressed the Tiananmen Square demonstration.
(C) Briefly explain how classical Chinese values could be used to justify the actions of either the
government or the Tiananmen Square protesters.

Sample Answers

(A) A growing number of Chinese citizens, who were increasingly aware of the outside world
and of the rising middle class within China, hoped to gain the same rights as citizens in
Western nations. For this reason, protesters wanted a more open and more democratic
system in China and an end to one-party rule.

(B) The government saw the Tiananmen Square demonstration as a threat to its authority and,
therefore, painted the demonstration as anarchic and chaotic. Also, the government wanted
to show the public that Chinese leaders and the Chinese military were working together.

(C) The government could justify its actions by referring to the traditional Chinese value of
respect for authority—a demand of filial piety and a foundation of Confucian thought. The
protesters could refer to the belief that corrupt and undeserving governments, according to
the mandate of heaven, should be overthrown.

Topic: The Spread of Democracy


Theme: State Building, Expansion, and Conflict
Learning Objective: SB–6.2
Historical Thinking Skill: Interpretation; Periodization; Contextualization
Key Concept: 6.2.V.E

249
Copyright © 2017, 2015, 2011, 2007 Pearson Education, Inc. All rights reserved.
6.49. Use the passage below and your knowledge of world history to answer all parts of the question
that follows.

By not allowing men to see their prospective wives following their engagement we cause
Egyptian men to seek European women in marriage. They marry European servants and working
class women thinking they would be happy with them rather than daughters of pashas or beys
away in a “box of chance.” If we do not solve this problem we shall be subject to occupation by
women of the West.

Malah Nasif, Speech to a Moderate Egyptian Nationalist Party, 1909

(A) What is Nasif referring to with her reference to a “box of chance?”


(B) Identify one goal of her audience and explain how Nasif appeals to that goal?
(C) Identify and briefly explain Nasif’s goal in giving the speech.

Sample Answers

(A) The “box of chance” is the unseen, veiled bride on marriage day. Nasif believes that
Egyptian men, by preferring to marry “visible” European women, are polluting the well of
Egyptian nationalism. To Egyptian nationalists, the concept of a blonde-haired, blue-eyed
Egyptian would be anathema.

(B) The moderate nationalist party is seeking to expunge Turkish (Ottoman) and British
influence from Egypt. To Nasif’s audience, Egypt should be for Egyptians. Nasif is hoping to
gain more rights for Egyptian women by appealing to the party’s sense of nationalism. In
essence, she is saying we are all in this together.

(C) Nasif’s goals are those of Egyptian feminism. She is hoping to convince the audience that its
goals and her goals are mutually compatible. By giving Egyptian women more rights and
freedoms, Egypt would strengthen its ability to rule free of foreign interference.

Topic: Egypt and the Rise of Nationalism in the Middle East


Theme: State Building, Expansion, and Conflict
Learning Objective: SB–6.2
Historical Thinking Skill: Interpretation; Contextualization
Key Concept: 6.2.II.A

250
Copyright © 2017, 2015, 2011, 2007 Pearson Education, Inc. All rights reserved.
6.50. Use the passage below and your knowledge of world history to answer all parts of the question
that follows.

It is important to realize the longterm—if crude—truth of Malthus. It could indeed be proved


that without the oil industry, food would have never caught up with world population. Hunger
would be general without fertilizers, weedkillers, pest control, infinitely variable feeding stuffs,
and mechanization, all of which are available only courtesy of the oil industry. There was no hint
of the existence of a “modern petrochemical industry” such as we now enjoy until the end of
WWII. Significantly, the possibility of preventing world hunger has only existed since 1950.

Henry Hobhouse (Historian), The Seeds of Change, 2005

(A) Identify and explain the “truth of Malthus.”


(B) Briefly explain why the board of directors of Exxon would appreciate the statements made
by Hobhouse.
(C) Offer one environmental argument to counter Hobhouse’s position on the oil industry.

Sample Answers

(A) In the late eighteenth century, in his “Essay on the Principle of Population,” Thomas Malthus
suggested that the population of the world would eventually outstrip the capability of the
planet to produce enough food for everyone. The result would be periodic bouts of
worldwide famine, continually reducing world population.

(B) The board of directors of Exxon would appreciate what Hobhouse wrote because Hobhouse
believes that the oil industry has led the way in developing efficient and effective means to
produce more foodstuffs.

(C) Students may want to talk about the greenhouse effect and the resulting changes in
temperature patterns of the oceans as well as temperatures on land. Also, while pesticides
and fertilizers lead to more efficient production of food, they also create crop strains that
are less hardy and unavailable to small farmers. Such chemicals are also poisoning the
environment. With the development of large agribusinesses, small farmers are slowly
getting squeezed out of the market.

Topic: The Global Environment


Theme: Creation, Expansion, and Interaction of Economic Systems; Interaction Between Humans and
the Environment
Learning Objective: ECON–9; ENV–6.3
Historical Thinking Skill: Interpretation; Periodization
Key Concept: 6.3.II.F

251
Copyright © 2017, 2015, 2011, 2007 Pearson Education, Inc. All rights reserved.
Section III
Long Essay Questions
Question 6.51: Analyze changes and continuities in gender roles and family relations in one of the
following regions between 1920 and the present.

Soviet Union—————Western Europe—————Latin America—————India

Sample Answer

In this question, students are asked to analyze changes and continuities in families and relations
between men and women from 1920 to the present in one of the above regions of the world. To gain
the thesis point, students must address all parts of the question, qualifying change and continuity in one
of the four regions mentioned above within the timeframe identified. Qualifying categories denoting
change in the post–World War I societies of each of these regions could be industrialization,
urbanization, feminism, the increasing impact of the middle class, or the development of gender
equality through the ideals of communism. Qualifying categories denoting continuity could be
traditional values, maintenance of a patriarchal society, and the dominance of rural over urban beliefs
and behaviors.

Points gained through the use of evidence shall be determined by the amount of evidence used. To gain
the maximum two points, the student will need to use at least five pieces of evidence. If the student
opts to talk about changes and continuities in the Soviet Union, changes mentioned could include the
discouragement of religious influence, the increase in universal education for both males and females,
the impact of urbanization and industrialization on the nuclear family, the decline in birth and death
rates, and growing gender equality in the professions. Under continuities, students could speak to
women maintaining an important position in the hard labor force throughout this time period and the
reality that women were still expected to fulfill traditional family roles while increasing their role in the
professions.

In western Europe, there was a concerted movement towards women’s rights and female involvement
in the professions that had a dramatic impact on family relations. Women received the right to vote,
played a larger role in advanced education, and secured a greater stake in jobs outside the home. These
changes were due, in part, to a rising middle class and industrialization. Early success led to feminist
protests demanding equality. Writers such as Friedan and Beauvoir called for laws giving women rights
to divorce, to equal pay, and to abortion (although a continuity is that such laws were only sporadically
enforced). A major shift in the West was that more and more middle-class women chose professions
over children, resulting in a decline in the birth rate. That which didn’t change in western European
countries was the continuance of inequality in pay and of discrimination in professional advancement.

In Latin America, change came more slowly. Traditional male attitudes about the role of women being
domestic continued. Industrialization and urbanization seemed to be the major engines of change. More
women entered the workforce, and women slowly gained the right to vote. Laws granting more rights to
women were passed but not necessarily enforced. Education reforms lagged behind those in the West,
and women were only grudgingly accepted into elected positions. Throughout this time, income
disparity between the sexes persisted, and women continued to be plagued by the “glass ceiling.”

252
Copyright © 2017, 2015, 2011, 2007 Pearson Education, Inc. All rights reserved.
It is in India that the least amount of change was seen. Gender roles and family relations were least
impacted by global urbanization and industrialization during this time period. Though women had the
right to vote, laws passed to ensure gender equality were largely ignored, as were divorce laws.
Attempts were made to enable women greater educational and occupational opportunities, but
throughout the timeframe women regarded themselves as second-class citizens. Early marriage and
large families still seemed to be the norm in much of the subcontinent of India. In fact, of the four
regions mentioned, India was the only region where males outnumbered females (attributable to
malnourishment).

In this question, the targeted historical thinking skill of continuity and change is accomplished by
providing specific evidence to support the impact of traditional values, urbanization, protests, and
industrialization, for which two points can be gained.

There are many ways the synthesis point can be earned. One way is by relating the question to a
previous era. In China, students could refer to filial piety and to footbinding of the classical and Qing
periods. Another way to obtain the synthesis point would be to concentrate on a different academic
discipline such as government. Although women in India may not have experienced as great a change
overall, women like Indira Gandhi have been political leaders. This fact could be connected to Margaret
Thatcher in Great Britain, Angela Merkel in Germany, or Eva Peron in Argentina. In all regions, with India
possibly excepted, reference could be made to gender and family relations in pre-industrialized rural
societies. Also, while being careful not to confuse economics with the thesis, students could speak to the
roles of industrialization, global economics, and communication and their impact on the twin issues of
gender equality and family relations. Additionally, the geography of urbanization in regional migration
(or the role of religion in continuity) could also be developed. This question offers the student many
options.

Topic: Gender Roles and Family Relations (The Soviet Union); Culture and Society in the West; Slow
Changes in Women’s Roles (Latin America); The Subordination of Women and the Nature of Feminist
Struggle in the Post-Classical Era
Theme: Development and Transformation of Social Structures
Learning Objective: SOC 6.3
Historical Thinking Skill: Patterns of Continuity and Change over Time
Key Concept: 6.3.III

253
Copyright © 2017, 2015, 2011, 2007 Pearson Education, Inc. All rights reserved.
Question 6.52: Select one of the revolutions below and analyze how the country was changed and not
changed by the revolution.

Mexico, 1910—————Russia, 1917—————Iran, 1917

Sample Answer

This question is asking students to select one of the above revolutions and to provide evidence and
analysis that show how that revolution changed and did not change the country. The absence of a time
frame indicates that students can discuss impact up to the present. To gain the thesis point, the student
needs to select a revolution and identify qualifiers that can be linked to specific evidence as to how the
revolution was and was not a change agent in the country.

To gain two evidential points, students will need to provide a minimum of six pieces of evidence, at least
one piece of evidence coming from each category. Before the 1910 revolution, Mexico was
characterized by corruption, stifled dissent, foreign intervention, and unequal land distribution.
Evidence could be shown to indicate that the first three characteristics of Mexican society remained in
force well after the revolution had ended. Other factors that remained constant before and after the
revolution that students could discuss include treatment of the indigenous and one-party rule (the PRI).
Fear of German intervention during World War I and the escapades of Pancho Villa perpetuated U.S.
involvement in Mexico in the years immediately after the revolution. Changes include land reform,
through land ownership that was transferred from haciendas to communal ownership, and the
discouragement of cults of personality (caudillos), through the limiting of presidential terms to one six-
year term of office.

While continuity dominated revolution in Mexico, change was definitely in charge in the Russian
Revolution of 1917. In this revolution, society, economics, politics, demography, and geography were
impacted. Because Bolshevik Russia withdrew from World War I, it lost large chunks of territory, sizably
reducing its western boundary. The capital was also moved from St. Petersburg to Moscow. The most
dramatic change brought about by the revolution, politically, was to end the monarchy and replace it
with one-party rule. Economically, it replaced a capitalistic system with communism. This entailed the
collectivization of the means of production and the development of a centrally controlled system of
quotas. Unlike Russia, the Soviet Union became more of a meritocracy, recognizing and developing
talent across socio-economic and gender lines. There was certainly much change in this revolution.
However, there were also many constants. Though the monarchy was eliminated and the aristocracy
dissolved, the Soviet Union replaced a plutocracy with an oligarchy. Before and after the revolution, the
Russians remained the dominant ethnic group.

The third revolution that students can select to write about is the Iranian Revolution. Change in Iran
brought on by revolution were dramatic. Leaders of this revolution found a faithful following when they
called for a religious purification of their society and a rejection of Western ideals. Women quickly re-
adopted the veil, and schools for women quickly closed. Iran changed from secular leadership to a
theocracy. Also, pro-democratic reforms that preceded the revolution were quickly dissolved. However,
both before and after the revolution, there was a dislike for the West. This dislike was fomented in the
early 1950s when the West openly sabotaged democratic elections in Iran that appeared to have
communist leanings.

Using the above information effectively, as evidence supporting the thesis, will gain students two points.

254
Copyright © 2017, 2015, 2011, 2007 Pearson Education, Inc. All rights reserved.
The synthesis point can be gained by connecting the revolution discussed to another revolution.
Comparisons could be drawn between the Russian Revolution and revolutions in Cuba (1959) or China
(1949). Perhaps students could attempt to compare the Mexican Revolution of 1910 with a revolution in
the same region 90 years earlier. Most of the information provided in the sample answer speaks to
internal changes and continuities; the synthesis point could also be gathered by looking at how outside
factors changed and didn’t change as a result of the revolution.

Topic: (Mexico): Culture and Politics in Post-Revolutionary Mexico; (Russia): Revolution in Russia:
Liberalism to Communism; Stabilization of Russia’s Communist Regime; (Iran) Iran: Religious Revivalism
and the Rejection of the West)
Theme: State Building, Expansion, and Conflict
Learning Objective: SB–6.2
Historical Thinking Skill: Patterns of Continuity and Change over Time
Key Concept: 6.2.II.D

255
Copyright © 2017, 2015, 2011, 2007 Pearson Education, Inc. All rights reserved.
Section IV
Document-Based Questions
Directions: The following question is based on the accompanying documents. The documents have been
edited for the purpose of this exercise.

In your response, you should do the following:

• State a relevant thesis that directly addresses all parts of the question.

• Support the thesis or a relevant argument with evidence from all, or all but one, of the
documents.

• Source and analyze the significance of at least four of the documents on the following basis:
intended audience, author’s purpose, historical context, format or medium, and/or point of view.

• Develop context by relating your argument to broader historical events or processes.

• Synthesize the elements above into a persuasive essay that extends your argument by connecting
it to:

• a different historical period or geographical region


• another course theme or historical approach that is not the focus of the essay (such as
political, social, economic, or intellectual history)
• another perspective from a different academic discipline (such as economics, art history,
anthropology, or government)

256
Copyright © 2017, 2015, 2011, 2007 Pearson Education, Inc. All rights reserved.
Question 6.53: To what extent did the Cold War affect the goals of leaders of decolonization
movements and new nations in the 1950s and 1960s?

Historical Background: The Cold War conflict between the United States and the Soviet Union, and their
respective allies, emerged in the late 1940s; it was reflected in movements like Ho Chi Minh’s communist
revolt against French colonialism in Vietnam and Castro’s 1959 revolution in Cuba.

Document 1
Manifesto of the Laodong Party, 1951
[Note: By this time China and the Soviet Union had recognized the Party’s regime in Vietnam, but it was still struggling for
control.]

The main task of the Viet Nam Laodong Party now is: To unite and lead the working class, the working
masses and the entire people of Viet Nam in their struggle to wipe out the French colonialists and
defeat the American interventionists; to bring the liberation war of the Viet Nam people to complete
victory, thereby making Viet Nam a genuinely independent and united country. . . . In the field of
external affairs, the Viet Nam Laodong Party recommends: The Viet Nam people must actively support
the national liberation movements of oppressed peoples; unite closely with the Soviet Union, China and
other people’s democracies; form close alliances with the peoples of France and the French colonies so
as to contribute to the antiimperialist struggle to defend world peace and democracy!

Document 2
President Sukarno’s Opening Remarks at the Bandung Conference, 1955
No task is more urgent than that of preserving peace. Without peace our independence means little.
The rehabilitation and upbuilding of our countries will have little meaning. Our revolutions will not be
allowed to run their course. . . .

What can we do? We can do much! We can inject the voice of reason into world affairs. We can
mobilise all the spiritual, all the moral, all the political strength of Asia and Africa on the side of peace.
Yes, we! We, the peoples of Asia and Africa, 1,400,000,000 strong, far more than half the human
population of the world, we can mobilise what I have called the Moral Violence of Nations in favour of
peace. We can demonstrate to the minority of the world which lives on the other continents that we,
the majority, are for peace, not for war.

257
Copyright © 2017, 2015, 2011, 2007 Pearson Education, Inc. All rights reserved.
Document 3
Jawaharlal Nehru’s First Speech, 1956
The preservation of peace forms the central aim of India’s policy. It is in the pursuit of this policy that we
have chosen the path of nonalignment in any military or like pact of alliance [including Cold War
alliances]. Nonalignment does not mean passivity of mind or action, lack of faith or conviction. It does
not mean submission to what we consider evil. It is a positive and dynamic approach to such problems
that confront us. We believe that each country has not only the right to freedom but also to decide its
own policy and way of life. Only thus can true freedom flourish and a people grow according to their
own genius.

We believe, therefore, in nonaggression and noninterference by one country in the affairs of another
and the growth of tolerance between them and the capacity for peaceful coexistence. We think that by
the free exchange of ideas and trade and other contacts between nations each will learn from the other
and truth will prevail. We therefore endeavor to maintain friendly relations with all countries, even
though we may disagree with them in their policies or structure of government. We think that by this
approach we can serve not only our country but also the larger causes of peace and good fellowship in
the world.

Document 4
Fidel Castro’s Second Declaration of Havana, 1962
Since the end of the Second World War, the Latin American nations are becoming pauperized
constantly. The value of their per capita income falls. The dreadful percentages of child death rate do
not decrease, the number of illiterates grows higher, the peoples lack employment, land, adequate
housing, schools, hospitals, communication systems and the means of subsistence. On the other hand,
North America investments exceed 10 billion dollars. Latin America, moreover, supplies cheap raw
materials and pays high prices for manufactured articles. Like the first Spanish conquerors, who
exchanged mirrors and trinkets with the Indians for silver and gold, so the United States trades with
Latin America. . . .

The duty of every revolutionary is to make revolution. We know that in America and throughout the
world the revolution will be victorious. But revolutionaries cannot sit in the doorways of their homes to
watch the corpse of imperialism pass by. The role of Job does not behoove a revolutionary. Each year by
which America’s liberation may be hastened will mean millions of children rescued from death, millions
of minds freed for learning, infinitudes of sorrow spared the peoples.

258
Copyright © 2017, 2015, 2011, 2007 Pearson Education, Inc. All rights reserved.
Document 5
Charles DeGaulle’s Fourth Press Conference, April 11, 1961
France does not have to be at all sorry for what she has achieved overseas . . . . I have said it often and I
repeat: it constitutes a great human accomplishment which—notwithstanding certain abuses and errors
despite all the endless spouting of all sorts of worthless demagogues—will forever be a credit to France.
But how many things have changed today.

Now our great national ambition is our own national progress, constituting a real source of power and
influence. Now the modern era permits us, compels us, to undertake a vast development. Now for this
development to succeed we must first of all employ the means and resources at our disposal on our
own behalf, in our own country. All the more so as we need these means and resources to ensure our
own defense against the greatest imperialism that the world has ever known—the imperialism of the
Soviet Union. We also need these means to win out in the tremendous economic, technical and social
struggle now under way between the forces of humanity and the forces of slavery. . . .

Some people say, “What would happen to these territories if France withdrew? They would
straightaway fall into misery and chaos, until Communism took over.” That is no doubt what would
happen to them; but then we would no longer have any duty toward them other than to pity them.

Some people say also, “Either the Soviet Union or the United States—or both at once—would try to take
France’s place in the territories from which she withdrew.” My answer is: I wish both of them a lot of
fun.

259
Copyright © 2017, 2015, 2011, 2007 Pearson Education, Inc. All rights reserved.
Sample Answer

An acceptable thesis would consist of one or more congruent sentences in the opening paragraph that
present a plausible argument for the relationship between the Cold War and the decolonization
movement during the 1950s and 1960s. Students may organize the documents around aligned and
nonaligned ex-colonies or around ex-colonies that were nonaligned and ex-colonies that were pro-
communist.

To achieve all four points in the evidence category of the rubric, the student will need to relate at least
four of the documents to the thesis and analyze each by addressing historical context, audience,
purpose, or point of view of each document. In Document 1, the audience is likely (plausibly) to be the
people of Vietnam, while the audience in Document 2 is likely to be newly independent nations in Asia
and Africa. The tone of Document 3 would seem to have the effect of calming the apprehensions of the
people of India. In Document 4, Castro aggressively urges unity of all Latin America against North
American imperialism. DeGaulle, in Document 5, assesses the positive impact that France has had upon
her now-independent colonies.

There are plenty of outside examples students could use to support their thesis. Considering only Africa,
they could cite Algeria, Nasser in Egypt, South Africa, Ghana, or any of several new nations along the
West African coast. One way the contextualization point could be earned is by commenting on how
some nonaligned emerging countries attempted to play both sides of the Cold War against each other as
they sought aid to improve their countries’ circumstances. Another way to gain the contextualization
point would be to address the role of the United Nations in decolonization.

The synthesis point could be gained by comparing the decolonization process in the 1950s and 1960s
with the Latin American decolonization process in the first half of the nineteenth century.

Topic: Nationalism and Decolonization in South and Southeast Asia and Africa; The Cuban Revolution:
Socialism in the Caribbean
Theme: State Building, Expansion, and Conflict
Learning Objective: SB–6.2
Historical Thinking Skill: Causation
Key Concept: 6.2.II.A; 6.2.IV.D

260
Copyright © 2017, 2015, 2011, 2007 Pearson Education, Inc. All rights reserved.
Question 6.54: Assess the relative importance of common ingredients or individual dynamics in
explaining the causes of late twentieth– and early twenty-first–century outbreaks of ethnic conflict.

Historical Background: After the initial collapses of Eastern bloc communist governments in 1989–1990,
Serbian Communist Slobodan Milosevic was elected president of Serbia, a state within Yugoslavia. Civil
war existed in Yugoslavia between 1991–1999.

Wales has been part of the British kingdom since the fourteenth century. In the 1990s, a Welsh
nationalist party called Plaid Cymru began calling for Welsh independence.

When Rwanda achieved independence from the Belgians in 1962, hundreds of thousands of the Tutsi, an
ethnic group favored by the Belgians but a minority of the population, fled to neighboring Uganda,
fearing the majority Hutu. Supporters of Hutu President Habyarimana routinely killed opposition Tutsi
and Hutu politicians within Rwanda. On April 6, 1994, a plane carrying Habyarimana was shot down by
unknown assailants. The shooting down of the plane was used by a small group of Habyarimana’s
associates to justify systematically killing opponents and, particularly, Tutsi.

Document 1
Indictment of Slobodan Milosevic, United Nations, New York, 2002
[F]rom 1987 until late 2000, Slobodan Milosevic was the dominant political figure in Serbia and the
Federal Republic of Yugoslavia. It is alleged that Slobodan Milosevic, acted alone and in the joint
criminal enterprise in the following ways:

(a) He exerted effective control over the elements of the Yugoslav People’s Army (“JNA”) and the
Yugoslav Army (“VJ”) which participated in the planning, preparation, facilitation and execution of the
forcible removal of the majority of non-Serbs, principally Bosnian Muslims and Bosnian Croats, from
large areas of Bosnia and Herzegovina. . . .

(d) He participated in the planning and preparation of the takeover of municipalities in Bosnia and
Herzegovina and the subsequent forcible removal of the majority of non-Serbs. He provided the
financial, material and logistical support for such a takeover. . . .

(g) He controlled, manipulated or otherwise utilized Serbian state run media to spread false messages of
ethnically based attacks by Bosnian Muslims and Croats against Serbs intended to create an atmosphere
of fear and hatred among Serbs living in Serbia, Croatia and Bosnia and Herzegovina which contributed
to the forcible removal of the majority of non-Serbs.

261
Copyright © 2017, 2015, 2011, 2007 Pearson Education, Inc. All rights reserved.
Document 2
T. D. Allman, “Serbia’s Bloody War,” Pamphleteer’s Press, 1993
“The Serbs came in the night,” a neighbor whispers in German. “The explosion was at one.” They put
dynamite in the sanctuary of the mosque and around the minaret. When the minaret collapsed, the
explosion blew out the back wall of their house. The mother and father were killed instantly. “The
children were sleeping upstairs with their grandmother,” he adds, “and survived.” They fled. No one
know where. . . .

After inspecting the debris of the mosque, which was really just a neighborhood chapel, we drive over
to see the Roman Catholic church the Serbs blew up. . . . The church is a tangle of cement and iron
girders now, but the steeple stands—or rather leans—twisted, pock-marked, scorched by the explosion.
...

It’s getting late now, and people scurry past us into their houses, refusing to answer questions. I knock
on several doors. Finally, a Serb opens his door. His house faces the side of the church. Windows in his
house were shattered. “Blowing up the church was a bad thing to do,” he says. He explains why: “Serbs
live around here. Serb property was damaged. Serb people were hurt.”

262
Copyright © 2017, 2015, 2011, 2007 Pearson Education, Inc. All rights reserved.
Document 3
Plaid Cymru, A [Welsh] Manifesto for the European Parliament Elections, 2004
Plaid Cymru wishes Wales to be a positive role model as a nation of proactive and influential Europeans.
With our two Plaid Cymru MEPs [Members of the European Parliament], Jill Evans and Eurig Wyn, Wales
has achieved a prominent place in Brussels. As a result significant steps have been taken towards
improving the conditions and rights of people in Wales, across Europe and further afield. . . .

Plaid Cymru is the only political party in Wales that has consistently represented the values and
interests of Wales on national and international issues.

Plaid Cymru represents the distinctive interests of Welsh farmers, not large ranchers in South East
England. Welsh farmers’ interests are what drive our policy interventions in agriculture. We do not
follow the British government’s line on the war in Iraq or other foreign affairs issues. Instead, we tell the
world what Wales thinks on such highly important matters.

We do not bow to the interests of the London stock exchange at the expense of Welsh manufacturing
jobs. Plaid Cymru is free to truly represent Wales in Europe. . . .

We supported the move to include a reference to respecting the rich cultural and linguistic diversity of
the Union in the text of the draft European Constitution [which reads:]

We will:

1. Continue to fight for the rights of minority language speakers in the European Union;
2. Continue working to ensure a legal basis to support minority languages in Europe in order to
facilitate European funding;
3. Continue fighting to mainstream minority language matters in European programs;
4. Continue raising the profile of the Welsh language in Europe;
5. Continue building alliances with other European stateless nations and minority communities to
advance the cause of minority language rights.

263
Copyright © 2017, 2015, 2011, 2007 Pearson Education, Inc. All rights reserved.
Document 4
National Assembly for Wales, “Your Guide to the Assembly,” 2006
In July, 1997, the Government established a White Paper, A Voice for Wales, which outlined proposals
for devolution in Wales [transfer of power to govern from the English Parliament to a Welsh National
Assembly]. These proposals were endorsed in the referendum of 18 September 1997. Parliament passed
the Government of Wales Act 1998, which established the National Assembly for Wales, and the
National Assembly for Wales (Transfer of Functions) Order 1999, which enabled the transfer of the
devolved powers and responsibilities from the Secretary of State for Wales to the Assembly to take
place on 1 July 1999. Subsequently many acts of Parliament have given new powers to the Assembly.

Wales remains part of the UK and the Secretary of State for Wales and Members of Parliament (MPs)
from Welsh constituencies continue to have seats in Westminster. Laws passed by Parliament in
Westminster still apply to Wales.

The Assembly has considerable power to develop and implement power within a range of areas:
agriculture, ancient monuments and historic buildings, culture, economic development, education and
training, the environment, health and health services, highways, housing, industry, local government,
social services, sport and leisure, tourism, town and country planning, transport and roads, [and] the
Welsh language.

Document 5
Bureau of Intelligence and Research, Declassified Confidential Memo on Genocide in Rwanda, April 26, 1994
Heavy fighting yesterday continued in Kigali after the government failed to show up for weekend peace
talks in Tanzania and the Rwandan Patriotic Front (RPF—Tutsi) did not appear in Zaire for separate talks
to which it apparently was not invited, according to press reports. Both sides announced unilateral,
conditional ceasefire declarations that were to take effect yesterday.

The ICRC [International Committee of the Red Cross] delegate for Africa is certain at least 100,000
Rwandans have been killed since April 6, believes the actual number is closer to 300,000, and notes ICRC
personnel in country think the total could be 500,000, according to Mission Geneva. The ICRC is
concerned that the situation could worsen, citing some Hutu extremists who speak of a “final solution”
to eliminate all Tutsis. The ICRC and other NGOs have withdrawn all their workers for the country,
except for a small number in Kigali.

Comment: the butchery shows no sign of ending. The inability of either side to conquer the other—and
the intensity of ethnic slaughter—make it increasingly probable Rwanda will divide into zones controlled
by the RPF in the north and east and by Hutu forces linked to the interim government in the northwest,
the south and most of Kigali. Though a ceasefire may be possible in the days ahead, Hutu hardliners
ascendant behind government lines totally distrust the RPF and will reject any interim political
settlement based on sharing power with the rebels.

264
Copyright © 2017, 2015, 2011, 2007 Pearson Education, Inc. All rights reserved.
Document 6
Human Rights Watch, Report on Genocide in Rwanda, 1999
[Note: Human Rights Watch is a privately funded, nongovernmental, nonpartisan, so-called watchdog group, with international
offices in North America and Europe.]

President Juvenal Habyarimana, nearing the end of two decades in power, was losing popularity among
Rwandans when the RPF [Tutsi Rwandan Patriotic Front] attacked from Uganda on October 1, 1990. At
first Habyarimana did not see the rebels as a serious threat, although they stated their intention to
remove him as well as to make possible the return of hundreds of thousands of Rwandan [Tutsi]
refugees who had lived in exile for a generation. The president and his close colleagues decided,
however, to exaggerate the RPF threat as a way to pull dissident Hutu back to his side and they began
portraying Tutsi inside Rwanda as RPF collaborators. For three and a half years, this elite worked to
redefine the population of Rwanda into “Rwandans,” meaning those who backed the president, and the
“ibyitso” or “accomplices of the enemy,” meaning the Tutsi minority and Hutu opposed to him. . . .

This leaves 5–7,000 Tutsi killed, and represents the annihilation of about 77percent of the population
registered as Tutsi. Using other data from Butare prefecture, our researchers computed an estimate loss
of 75 percent of the Tutsi population in that prefecture. Based on these preliminary data, we would
conclude that at least half a million persons were killed in the genocide, a loss that represented about
three quarters of the Tutsi population in Rwanda.

Sample Answer

To receive the thesis point, students need to present a series of congruent sentences in the opening or
closing paragraph (preferably opening) that groups common ingredients and individual dynamics of
different ethnic conflicts around the world toward the end of the twentieth and beginning of the
twenty-first centuries. Students can group common elements by looking at either causes or
characteristics or both. Keep in mind, however, that this is a causation question. If causation is
investigated, students may want to investigate common “powder kegs” or common “sparks.” If focusing
on characteristics, students may want to look at levels of violence. To gain the maximum four points in
evidence analysis, it is always wise to use all documents by relating the evidence back to the thesis and
addressing the intended audience, author’s point of view or purpose, or placing the document in the
context of what was then happening. For instance, Document 1, in essence, is calling for the arrest of
Milosevic. In Documents 2, 3, and 4, students may want to speak to the intended audiences for the
documents that reside outside the effected region, particularly Western democracies. Documents 4 and
5, in particular, have the tone of hoping to wake up the West to the atrocities of the region. The fact
that the source for Document 6 is headquartered in North America suggests that its intended audience
is English-speaking nations.

The contextualization point may be earned by showing how ethnic conflicts were not unique to these
regions. Students could reference current and recent conflicts in Sudan, Mexico, Indonesia, Spain,
Turkey, Syria, and Cambodia, as well as the more historic “Final Solution” imposed by the Nazis during
World War II.

The synthesis point can perhaps be most safely earned by addressing the role of geography or religion in
fanning the flames of ethnic discord: water in the Middle East as well as Sunni vs. Shi’a in the same
region.

265
Copyright © 2017, 2015, 2011, 2007 Pearson Education, Inc. All rights reserved.
Topic: Nationalism and New Religious Currents
Theme: State Building, Expansion, and Conflict
Learning Objective: SB–6.2
Historical Thinking Skill: Causation
Key Concept: 6.2.II.B

266
Copyright © 2017, 2015, 2011, 2007 Pearson Education, Inc. All rights reserved.
 

Credits  
 
 
Period  1  
 
MCQ  1.1–1.3:   The  Spread  of  Agriculture:  W.  H.  McNeil,  A  History  of  Human  Community,  Third  Edition  1990,  p.  17.  Pearson  
Education:  Upper  Saddle  River,  New  Jersey.  
MCQ  1.4–1.6:   Excavation  of  the  Ancient  Settlement  at  Çatal  Hüyük:  Marion  Bull  /  Alamy  Stock  Photo  
MCQ  1.4–1.6:   Neolithic  Woman  Spinning  a  Dried  Yucca  Stalk  Against  a  Fire-­‐Starter:  Reeda  L.  Peel  
MCQ  1.7–1.9:   Statue  Known  to  the  West  as  the  Sphinx  and  to  the  Arabs  as  the  Father  of  Terror:  Ian  Stewart/Shutterstock  
MCQ  1.7–1.9:   Sumerian  Clay  Tablet  with  Cuneiform  Characters:  DeAgostini  /Superstock  
MCQ  1.7–1.9:   Nineveh:  Philip  and  Elizabeth  De  Bay/Getty  Images  
MCQ  1.10–1.12:   Statue  Known  to  the  West  as  the  Sphinx  and  to  the  Arabs  as  the  Father  of  Terror:  orlandin/Shutterstock  
MCQ  1.10–1.12:   Early  Chinese  Script  on  Stone:  New  York  Public  Library  Digital  Collection  
MCQ  1.10–1.12:   Capture  of  Babylon:  Falkenstein/Bildagentur-­‐online  Historical  Collect.  /  Alamy  Stock  Photo  
SAQ  1.16:   Basalt  Olmec  Head:  Anthropology  Museum,  Veracruz,  Jalapa,  Mexico/Werner  Forman  /  Art  Resource,  NY  
DBQ  1.18:   The  Great  Wall:  Nancy  McKenna/  Photo  Researchers,  Inc.  
DBQ  1.18:   Sources  of  Chinese  Tradition,  Second  Edition.  Volume  1.  Edited  by  William  Theodore  de  Bary  and  Irene  
Bloom.  Copyright  ©  1999  Columbia  University  Press.  Reprinted  by  permission  of  Columbia  University  
Press.  
DBQ  1.18:   Bronze  Axe  Head  from  the  Shang  Dynasty:  Werner  Forman  Archive  Heritage  Images/Newscom  
DBQ  1.18:   Bronze  Vessel  from  the  Shang  Dynasty:  Martha  Avery/Asian  Art  &  Archaeology,  Inc/Corbis  Images  
 
Period  2  
 
MCQ  2.1–2.4:   Eighteenth-­‐Century  Artist’s  Rendering  of  Confucius,  Laozi,  and  the  Buddha:  Art  Archive/Superstock  
MCQ  2.5–2.7:   Battle  Between  Alexander  the  Great  and  King  Darius,  Roman  Mosaic,  House  of  Faun,  Macedonia:  World  
History  Archive  /  Alamy  Stock  Photo  
MCQ  2.11–2.13:   A  Roman  Warship:  Roman  civilization,  Bireme  war  ship,  relief  from  Temple  of  Fortuna  Primigenia  at  
Palestrina,  Lazio  Region,  Italy  /  De  Agostini  Picture  Library  /  Bridgeman  Images  
MCQ  2.14–2.16:   Poems  by  Two  Buddhist  Nuns:  Used  by  permission  from  Uma  Chakravarti  and  Kumkum  Roy.  
MCQ  2.21–2.25:   The  Great  Wall  of  China:  Nancy  McKenna/Science  Source  
MCQ  2.32–2.34:   Icon  of  the  Emperor  Constantine  Accompanied  by  the  Bishops  of  the  First  Council  of  Nicaea:  Heritage  Image  
Partnership  Ltd  /  Alamy  Stock  Photo  
SAQ  2.35:   Seventh-­‐Century  Buddhist  Pilgrim:  Lebrecht  Music  and  Arts  Photo  Library  /  Alamy  Stock  Photo  
 
Period  3  
 
MCQ  3.1–3.4:   Conversion  of  the  Russia’s  King  Vladimir  I,  988:  Academy  of  Science,  St.  Petersburg,  Russia  /  Erich  Lessing  /  
Art  Resource,  NY  
MCQ  3.5–3.8:   One  of  the  Earliest  Known  Artistic  Renderings  of  a  Ninth-­‐Century  Arab  Army  at  War:  Biblioteca  Nacional,  
Madrid,  Spain  /  Werner  Forman  /  Art  Resource,  NY  
MCQ  3.9–3.13:   Illustration  of  the  Great  Aztec  City-­‐State  of  Tenochtitlan:  Museo  Nacional  de  Antropologia,  Mexico  City,  
Mexico  /  Sean  Sprague/Mexicolore  /  Bridgeman  Images  
MCQ  3.26–3.28:   New  Farming  Techniques  Developed  as  the  Importance  of  Rice  Increased  in  China  During  the  Song  Dynasty:  
bpk  Bildagentur  /  Museum  fuer  Asiatische  Kunst,  Staatliche  Museen,  Berlin,  Germany/Juergen  Liepe./  Art  
Resource,  NY  
MCQ  3.35–3.37:   Arab  Scientists  Testing  and  Working  with  a  Wide  Variety  of  Instruments:  Granger,  NYC  —  All  rights  reserved.  
MCQ  3.41–3.43:   Painting  of  Dante,  by  Domenico  di  Michelina,  Which  Hangs  in  the  Cathedral  of  Florence,  1465:  Duomo,  
Florence,  Italy  /  Scala  /  Art  Resource,  NY    
MCQ  3.44–3.46:   The  Golden  Pavilion,  One  of  the  Great  Architectural  Treasures  of  Japan:  Free  Agents  Limited/Corbis  Images    
MCQ  3.47–3.50:   Night  Attack  on  the  Sanjo  Palace  (from  the  Scrolls  of  the  Events  of  the  Heiji  Era):  Heritage  Image  Partnership  
Ltd/Alamy  Stock  Photo  &  Fenollosa-­‐Weld  Collection/Museum  of  Fine  Arts,  Boston  

267  
Copyright  ©  2017,  2015,  2011,  2007  Pearson  Education,  Inc.  All  rights  reserved.  
 

MCQ  3.47–3.50:   The  Knight  and  the  Landsknecht,  by  Albrecht  Dürer,  c.  1497:  Heritage  Image  Partnership  Ltd  /  Alamy  Stock  
Photo  
MCQ  3.51:   X-­‐Ray  of  Bound  Feet,  China,  Between  1890  and  1923:  Photo  Researchers,  Inc  /  Alamy  Stock  Photo  
SAQ  3.55:   Persian  Miniature  of  a  Prince  Submitting  to  Chinggis  Khan:  Granger,  NYC  —  All  rights  reserved  
DBQ  3.58:   Painting  of  Dante,  by  Domenico  di  Michelina,  Which  Hangs  in  the  Cathedral  of  Florence,  1465:  Duomo,  
Florence,  Italy  /  Scala  /  Art  Resource,  NY  
DBQ  3.59:   Group  of  Stone  Sculptures,  Longmen  Caves,  China:  The  Metropolitan  Museum  of  Art,  New  York,  NY,  USA  /  
Art  Resource,  NY  
 
Period  4  
 
MCQ  4.1–4.3:   The  Port  of  Lisbon,  Portugal:  Service  Historique  de  la  Marine,  Vincennes,  France  /  Bridgeman  Images  
MCQ  4.10–4.12:   Jesuits  in  Chinese  Dress  at  the  Chinese  Emperor’s  Court:  Private  Collection  /  Bridgeman  Images    
MCQ  4.13–4.15:   Silver  Mine  at  Potosí:  Photo  Researchers,  Inc  /  Alamy  Stock  Photo  
MCQ  4.16–4.18:   Collection  of  Taxes  from  the  Russian  Peasantry:  Sovfoto/Eastfoto  
MCQ  4.22–4.24:   Suleymaniye  Mosque  and  Golden  Horn,  Istanbul,  Turkey:  ruzgar344/Shutterstock  
MCQ  4.22–4.24:   Taj  Mahal  at  Agra:  Tretyakov  Gallery,  Moscow,  Russia  /  Scala  /  Art  Resource,  NY  
MCQ  4.28–4.30:   Caricature  of  John  Tetzel,  Famous  Indulgence  Preacher:  INTERFOTO  /  Alamy  Stock  Photo  
MCQ  4.31–4.34:   The  Ottoman  Siege  of  Constantinople,  1453:  Bettmann/Getty  Images  
MCQ  4.38–4.40:   Nineteenth  Engraving  of  the  Cubicles  for  Imperial  Civil  Service  Exams  in  Beijing:  Peter  N  Stearns/Charles  
Scribner's  Sons  
MCQ  4.38–4.40:   Wives  and  Concubines  of  Ming  Emperors  Depicted  in  a  Scene  of  Court  Life:  British  Museum,  London,  Great  
Britain  /  The  Trustees  of  the  British  Museum  /  Art  Resource,  NY  
MCQ  4.41–4.43:   Cathedral  of  St.  Basil  the  Blessed,  Red  Square,  Moscow:  bunyos/Fotolia  
MCQ  4.41–4.43:   Cartoon  Lampooning  Peter  the  Great’s  Order  to  the  Nobility  to  Cut  Off  Their  Beards:  akg-­‐images  
MCQ  4.44–4.45:   Door  Panels,  Florence  Baptistery,  Italy  (left):  akg-­‐images  /  Rabatti  -­‐  Domingie/Newscom  
MCQ  4.44–4.45:   Door  Panels,  Florence  Baptistery,  Italy  (right):  akg-­‐images  /  Rabatti  -­‐  Domingie/Newscom  
MCQ  4.44–4.45:   The  Palace  of  Versailles:  Chateaux  de  Versailles  et  de  Trianon,  Versailles,  France  /  RMN-­‐Grand  Palais  /  Art  
Resource,  NY  
SAQ  4.48:   French  Engraving,  Haitian  Sugar  Plantation,  Eighteenth  Century:  INTERFOTO  /  Alamy  Stock  Photo  
SAQ  4.49:   Hanging  Scroll  Depicting  “A  Meeting  of  Japan,  China  and  the  West”:  Minneapolis  Institute  of  Arts,  MN,  USA  
/  Bridgeman  Images  
DBQ  4.53:   Henricus  Martellus  (German  Cartographer),  World  Map,  c.  1489:  Granger,  NYC  —  All  rights  reserved.  
DBQ  4.53:   Diego  Duran  (Spanish  Artist),  Hernando  Cortés  Besieged  by  Aztec  Warriors,  Sixteenth  Century:  Codex  Duran:  
Pedro  de  Alvarado  (c.1485–1541)  companion-­‐at-­‐arms  of  Hernando  Cortes  (1485–1547)  besieged  by  Aztec  
warriors  (vellum),  Duran,  Diego  (16th  century)  /  Biblioteca  Nacional,  Madrid,  Spain/The  Bridgeman  Art  
Library  
 
Period  5  
 
MCQ  5.4–5.6:   American  Anti-­‐Imperialist  Cartoon:  Sarin  Images  /  Granger,  NYC  —  All  rights  reserved  
MCQ  5.12–5.14:   Cartoon  from  Puck,  August  15,  1900:  Granger,  NYC  —  All  rights  reserved.  
MCQ  5.24–5.26:   A  Political  Print:  Library  of  Congress  Prints  and  Photographs  Division  [LC-­‐USZC4-­‐5913]  
MCQ  5.27–5.30:   Painting  of  the  Tennis  Court  Oath:  Everett  -­‐  Art/Shutterstock  
MCQ  5.31–5.33:   Cartoon  from  Punch,  1906:  Mary  Evans  Picture  Library/Alamy  Stock  Images  
MCQ  5.43–5.45:   Theodore  Roosevelt,  Panama,  c.  1904:  Library  of  Congress  Prints  and  Photographs  Division  [LC-­‐DIG-­‐stereo-­‐
1s02353]  
MCQ  5.46–5.48:   Emmeline  Pankhurst:  Granger,  NYC  —  All  rights  reserved.  
SAQ  5.60:   Engraving  of  British  Opium  Factory,  Nineteenth  Century:  Contraband  Collection  /  Alamy  Stock  Photo  
DBQ  5.66:   French  Engraving,  1789:  Josse  Christophel  /  Alamy  Stock  Photo  
 
Period  6  
 
MCQ  6.1–6.3:   Faisal  and  Delegation,  Peace  Conference,  Versailles,  1919:  Bettmann/Getty  Images  

268  
Copyright  ©  2017,  2015,  2011,  2007  Pearson  Education,  Inc.  All  rights  reserved.  
 

MCQ  6.4–6.6:   Advertisement  for  Mitchell’s  Golden  Dawn  Cigarettes:  Private  Collection  /  Peter  Newark  Military  Pictures  /  
Bridgeman  Images  
MCQ  6.10–6.13:   Russian  Children  at  the  Beginning  of  Stalin’s  Collectivization  Plan,  1931:  Sovfoto/Eastfoto  
MCQ  6.14–6.16:   Photo  of  Nazi  Rally  Saluting  Adolph  Hitler,  Mid-­‐1930s:  World  History  Archive/Alamy  Stock  Photo  
MCQ  6.20–6.22:   Nkrumah  of  Ghana:  Bettmann/Getty  Images  
MCQ  6.20–6.22:   Nasser  of  Egypt:  Gamal  Abdel  Nasser/Getty  Images  
MCQ  6.20–6.22:   Gandhi  of  India:  Ullstein  Bild  /  Granger,  NYC  —  All  rights  reserved.    
MCQ  6.20–6.22:   Ho  of  Vietnam:  World  History  Archive  /  Alamy  Stock  Photo  
MCQ  6.26–6.28:   Fidel  Castro  and  His  “Barbudos”  (Bearded)  Guerillas:  Bettmann/Getty  Images  
MCQ  6.29–6.31:   Population  of  Capital  Cities  as  a  Percentage  of  Total  Population  in  10  Latin  American  Nations  Based  on  J.  P.  
Cole,  Latin  America:  An  Economic  and  Social  Geography  (1965),  417  
MCQ  6.35–6.37:   The  Growth  of  the  World  Population  by  Major  Geographic  Areas,  1930–2000:  Encyclopedia/Corbis  Images  
MCQ  6.44–6.46:   Cartoon  Commentary  on  China:  John  Deering/Arkansas  Democrat-­‐Gazette  
DBQ  6.54:   From  Fighting  Hard  for  Wales:  A  Manifesto  for  the  European  Parliament  Elections  2004.  Copyright  ©  2004  
by  Plaid  Cymru.  Used  by  permission.  
 

269  
Copyright  ©  2017,  2015,  2011,  2007  Pearson  Education,  Inc.  All  rights  reserved.  

Das könnte Ihnen auch gefallen